Peds Exam 2 TB

Réussis tes devoirs et examens dès maintenant avec Quizwiz!

Which statement best describes a subdural hematoma? a. Bleeding occurs between the dura and the skull. b. Bleeding occurs between the dura and the cerebrum. c. Bleeding is generally arterial, and brain compression occurs rapidly. d. The hematoma commonly occurs in the parietotemporal region.

b. Bleeding occurs between the dura and the cerebrum. A subdural hematoma is bleeding that occurs between the dura and the cerebrum as a result of a rupture of cortical veins that bridge the subdural space. An epidural hemorrhage occurs between the dura and the skull, is usually arterial with rapid brain concussion, and occurs most often in the parietotemporal region.

Which interventions should the nurse include in the plan of care for the infant awaiting surgical closure of a myelomeningocele sac? a. Open to air b. Covered with a sterile, moist, nonadherent dressing c. Reinforcement of the original dressing if drainage noted d. A diaper secured over the dressing

b. Covered with a sterile, moist, nonadherent dressing Before surgical closure, the myelomeningocele is prevented from drying by the application of a sterile, moist, nonadherent dressing over the defect. The moistening solution is usually sterile normal saline. Dressings are changed frequently (every 2 to 4 hours), and the sac is closely inspected for leaks, abrasions, irritation, and any signs of infection. The sac must be carefully cleansed if it becomes soiled or contaminated. The original dressing would not be reinforced but changed as needed. A diaper is not placed over the dressing because stool contamination can occur.

Parents of a school-age child with hemophilia ask the nurse, "Which sports are recommended for children with hemophilia?" Which sports should the nurse recommend? (Select all that apply.) a. Soccer b. Swimming c. Basketball d. Golf e. Bowling

b. Swimming d. Golf e. Bowling Because almost all persons with hemophilia are boys, the physical limitations in regard to active sports may be a difficult adjustment, and activity restrictions must be tempered with sensitivity to the child's emotional and physical needs. Use of protective equipment, such as padding and helmets, is particularly important, and noncontact sports, especially swimming, walking, jogging, tennis, golf, fishing, and bowling, are encouraged. Contact sports such as soccer and basketball are not recommended.

Which best describes how preschoolers react to the death of a loved one? a. The preschooler is too young to have a concept of death. b. A preschooler is likely to feel guilty and responsible for the death. c. Grief is acute but does not last long at this age. d. Grief is usually expressed in the same way in which the adults in the preschooler's life are expressing grief.

ANS: B Because of egocentricity, the preschooler may feel guilty and responsible for the death. Preschoolers usually have some understanding of the meaning of death. Death is seen as a departure or some kind of sleep and they have no understanding of the permanence of death. None of the other statements accurately describe the usually preschoolers reaction to death.

In which condition are all the formed elements of the blood simultaneously depressed? a. Aplastic anemia b. Sickle cell anemia c. Thalassemia major d. Iron deficiency anemia

a. Aplastic anemia Aplastic anemia refers to a bone marrow-failure condition in which the formed elements of the blood are simultaneously depressed. Sickle cell anemia is a hemoglobinopathy in which normal adult hemoglobin is partly or completely replaced by abnormal sickle hemoglobin. Thalassemia major is a group of blood disorders characterized by deficiency in the production rate of specific hemoglobin globin chains. Iron deficiency anemia results in a decreased amount of circulating red cells.

The adolescent is admitted to the hospital in sickle cell crisis with a pain level of 10/10. The physician orders: Morphine sulfate 5 mg IV q 2 hr prn Medication on hand: morphine sulfate 10 mg/mL Calculate how many ml of morphine sulfate will be given IV.

0.5 mL

A parent of a newborn asks the nurse why a heel stick is being done on the baby to test for phenylketonuria (PKU). Which response by the nurse is the most appropriate? 1. "This screening is required and detection can be done before symptoms develop." 2. "The infant has high-risk characteristics." 3. "Because the infant was born by cesarean, this test is necessary." 4. "Because the infant was born by vaginal delivery, this test is recommended."

1. "This screening is required and detection can be done before symptoms develop." Screening for phenylketonuria is required by law in every state. It is not done according to high-risk characteristics or type of delivery.

) Parents of a child in the pediatric intensive care unit (PICU) have been experiencing shock and disbelief regarding their situation. Which statement by the parents indicates they are moving forward into the next stage of coping? 1. "Why not me instead of my child?" 2. "It is hard for me to have others take care of my child." 3. "I feel like life is suspended in time." 4. "I am glad I can help with his care."

1. "Why not me instead of my child?" The parents initially enter the stage of shock and disbelief. Asking "Why not me instead of my child?" shows they are moving into the next stage, which is anger and disbelief. Having feelings about others caring for their child is the third stage of deprivation and loss. The feeling of being suspended in time is the fourth stage, which is anticipatory guidance.

The school nurse is providing care to a school-age client who experienced a sprain of the right ankle on the playground. Which intervention is appropriate for the nurse to implement for this client? 1. Apply ice to the extremity 2. Apply a warm, moist pack to the extremity 3. Perform passive range of motion to the extremity 4. Lower the extremity to below the level of the heart

1. Apply ice to the extremity For the first 24 hours of a sprain, rest, ice, compression, and elevation should be used. Therefore, the nurse should apply ice to the extremity.

The nurse is working with a child with a chronic condition. The nurse observes that over time, the parents have experienced a pattern of periodic grieving alternating with denial. What are the parents currently experiencing based on this assessment finding? 1. Chronic sorrow 2. Compassion fatigue 3. Dysfunctional parenting 4. Pathological grieving

1. Chronic sorrow Parents experience chronic sorrow as they grieve when their child does not meet developmental milestones or participate in activities of "normal" children. The time between periods of grieving may be times of parental denial, which allows the family to function. Compassion fatigue is experienced by caregivers as their ability to feel compassion is exhausted. Dysfunctional parenting involves inadequately meeting the needs of children. Pathological grieving results when persons do not move through the stages of grief to resolution.

The mother of a child with a heart defect is questioning the nurse about the child's diuretic. When teaching the mother about the medication, what should the emphasis from the nurse? 1. Close monitoring of output 2. The digitalization process 3. The possibility that pulses in the child might be weak 4. The child's increased appetite

1. Close monitoring of output It is important to monitor the output of the child on a diuretic to determine effectiveness of the drug. Digitalization pulses are not associated with diuretics. The child will usually have a decreased appetite.

The nurse is providing care to an adolescent client who is dying. Which assessment findings indicate the client is experiencing a decrease in peripheral circulation? Select all that apply. 1. Cool skin 2. Mottled appearance 3. Cheyne-Stokes respirations 4. Increased agitation 5. Increased urine output

1. Cool skin 2. Mottled appearance A client who is experiencing decreased peripheral circulation will have cool, mottled skin. While Cheyne-Stokes respirations may indicate death is approaching, this is not indicative of a decrease in peripheral circulation. Increased agitation indicates decreased perfusion to the brain. A client will not experience increased urine output near the end life.

The school nurse completes an assessment of a school-age client to determine the services this child will need in the classroom. The client is a newly diagnosed with type I diabetes mellitus. Based on this information, which special healthcare need category is the most appropriate? 1. Dependent on medication or special diet 2. Dependent on medical technology 3. Increase use of healthcare services 4. Functional limitations

1. Dependent on medication or special diet A child recently diagnosed with type I diabetes mellitus with no other medical diagnoses would be placed in the dependent on medication or special diet category. The other categories of care are not appropriate for this client.

A pediatric client is hospitalized with a severe case of impetigo contagiosa. Which antibiotic does the nurse anticipate the healthcare provider will order for this client? 1. Dicloxacillin (Pathocil) 2. Rifampin (Rifadin) 3. Sulfamethoxazole and trimethoprim (Bactrim) 4. Metronidazole (Flagyl)

1. Dicloxacillin (Pathocil) A systemic antibiotic will be given for severe impetigo because it is a bacterial infection. Dicloxacillin is used in treatment of skin and soft-tissue infections. It is specific for treating staphylococcal infections. Rifampin is an antitubercular agent, sulfamethoxazole and trimethoprim are used as a prophylaxis against Pneumocystis carinii pneumonia (PCP), and metronidazole is used to treat anaerobic and protozoic infections.

The nurse is performing the initial assessment of a child newly diagnosed Kawasaki disease. Which symptoms would the nurse expect to assess with this child? 1. Dry, swollen, fissured lips 2. Nonpalpable lymph nodes 3. Conjunctivitis with exudates 4. Cyanosis of the hands and feet

1. Dry, swollen, fissured lips Dry, swollen, fissured lips are symptoms of Kawasaki disease. Lymph nodes can be palpable, conjunctivitis is present but without exudates, and hands and feet are typically erythematous.

Which stressor is common in the hospitalized toddler with a chronic disorder? Select all that apply. 1. Fear of painful procedures 2. Self-concept 3. Interruption of normal routines 4. Unfamiliarity of caregivers 5. Isolation

1. Fear of painful procedures 3. Interruption of normal routines 4. Unfamiliarity of caregivers

The nurse admits a child with a ventricular septal defect (VSD) to the unit. Which nursing diagnosis for this child is the most appropriate? 1. Impaired Gas Exchange Related to Pulmonary Congestion Secondary to the Increased Pulmonary Blood Flow 2. Deficient Fluid Volume Related to Hyperthermia Secondary to the Congenital Heart Defect 3. Acute Pain Related to the Effects of a Congenital Heart Defect 4. Hypothermia Related to Decreased Metabolic State

1. Impaired Gas Exchange Related to Pulmonary Congestion Secondary to the Increased Pulmonary Blood Flow Because of the increased pulmonary congestion, Impaired Gas Exchange would be an appropriate nursing diagnosis. Ventricular septal defects do not cause pain, fever, or deficient fluid volume.

The nurse is providing education to a pediatric client diagnosed with diabetes. The client will be playing soccer over the summer. Which change in the client's management will the nurse explore during this education session? 1. Increased food intake 2. Decreased food intake 3. Increased need for insulin 4. Decreased risk of insulin reaction

1. Increased food intake Increased physical activity requires adequate caloric intake to prevent hypoglycemia, so food intake should be increased. Increased activity would not require decreased food intake, and it would not result in a decreased risk of insulin reaction. Exercise causes the insulin to be used more efficiently, so increased insulin would not be needed.

A child comes to the clinic for an assessment 20 days post-bone marrow transplant. Which system should receive the highest priority during the nursing assessment? 1. Integumentary 2. Gastrointestinal 3. Respiratory 4. Cardiovascular

1. Integumentary The skin is most commonly affected in graft-versus-host disease after a transplant. A pruritic, macular papular rash and a blistering, burning sensation can occur. The other systems are important to assess, but are not the highest priority.

A school-age client is admitted to the pediatric intensive care unit (PICU) in critical condition after a motor vehicle accident. Which intervention should be implemented at this time? 1. Maintain consistent caregivers. 2. Turn the lights off at night. 3. Keep alarm levels low. 4. Consult the hospital play therapist.

1. Maintain consistent caregivers. The intensive care environment is fast-paced, overwhelming, and frightening. Maintaining consistent caregivers is invaluable in developing a familiar and trusting relationship with the child. Turning off the lights in an intensive care environment is not feasible. Keeping alarm levels low could increase risk of injury if an alarm is not heard by staff. Consulting the play therapist is not appropriate at this time.

The nurse is caring for an adolescent client diagnosed with rheumatoid arthritis. Which nonpharmacological measure to reduce joint pain is most appropriate for the nurse to recommend to this client? 1. Moist heat 2. Elevation of extremity 3. Massage 4. Immobilization

1. Moist heat Moist heat can promote relief of pain and decrease joint stiffness. Elevation of extremity would not have an effect on reducing pain in rheumatoid arthritis. Massage of extremities should be avoided because of a potential risk for emboli. Immobilization can lead to contractures, and range of motion to the involved joint should be maintained.

The nurse is providing care to a school-age client admitted to the emergency department following a motor vehicle crash. The client is exhibiting symptoms of hypovolemic shock. Which nursing interventions are appropriate for this client? Select all that apply. 1. Monitor hemoglobin and hematocrit. 2. Monitor liver enzymes. 3. Administer oxygen, as needed. 4. Administer a dextrose solution. 5. Monitor blood glucose.

1. Monitor hemoglobin and hematocrit. 3. Administer oxygen, as needed. 5. Monitor blood glucose. Nursing care for a client experiencing hypovolemic shock is aimed at monitoring the child's condition and response to clinical therapy. It is appropriate for the nurse to monitor hemoglobin, hematocrit, and blood glucose. The nurse will also administer oxygen. The nurse will administer large volumes of crystalloid fluids (normal saline or lactated Ringer's), not dextrose. It is not necessary to monitor liver enzymes for this client.

The nurse in the long-term care clinic is reviewing the charts of a group of children with chronic physical, psychological, functional, and social limitations. Which conditions are most likely to lead to chronic limitations? Select all that apply. 1. Near drowning 2. Congenital heart defect 3. Sinusitis 4. Fetal insult when the mother contracted rubella in the first trimester of pregnancy 5. Sepsis contracted as a neonate

1. Near drowning 2. Congenital heart defect 4. Fetal insult when the mother contracted rubella in the first trimester of pregnancy 5. Sepsis contracted as a neonate All of these conditions or events except sinusitis can leave a child with a permanent chronic condition.

A child with human immunodeficiency virus (HIV) also has oral candidiasis. Which type of mouth care solution will the nurse teach the child to use? 1. Normal saline 2. Listerine 3. Scope 4. Viscous lidocaine

1. Normal saline The mouth care should be with a non-alcohol base. Normal saline can keep the child's lips and mouth moist. Listerine and Scope are commercial mouth rinses that can have an alcohol base and cause drying of the membranes. Viscous lidocaine causes numbing and could depress the gag reflex in a younger child.

A child recently diagnosed with aplastic anemia is being prepared for discharge. When planning support for the family, which service should the nurse plan to include in the discharge plan? 1. Referrals to support groups and social services 2. Short-term support 3. Genetic counseling 4. Nutrition counseling

1. Referrals to support groups and social services Families require support in dealing with a child who has a life-threatening disease. They should be referred to support groups for counseling, if indicated, and to social services. The support will be long term in nature. Aplastic anemia is not a genetically transmitted disease. Nutrition counseling is not a priority and may or may not be needed with aplastic anemia.

An adolescent client who is diagnosed with Duchenne muscular dystrophy is seen in the clinic for a routine health visit. Which nursing diagnosis is the priority for this client? 1. Risk for Impaired Mobility Related to Hypertrophy of Muscles 2. Risk for Infection Related to Altered Immune System 3. Risk for Impaired Skin Integrity Related to Paresthesia 4. Risk for Altered Comfort Related to Effects of the Illness

1. Risk for Impaired Mobility Related to Hypertrophy of Muscles Nursing care for muscular dystrophy (MD) focuses on promoting independence and mobility for this progressive, incapacitating disease. Risk for Infection, Risk for Impaired Skin Integrity, and Risk for Altered Comfort are not as high a priority as Risk for Impaired Mobility.

A preschool-age child has just had a moderate reaction to latex. When teaching the parents about latex allergy, the nurse should inform the parents of what common household items that contain latex? Select all that apply. 1. Rubber bands 2. Sneakers 3. Toothbrushes 4. Big Wheel® tricycle 5. Water toys

1. Rubber bands 2. Sneakers 3. Toothbrushes 5. Water toys Rubber bands, sneakers, toothbrushes, and water toys are household items that might contain latex. A Big Wheel® tricycle is plastic and does not contain latex.

A child is admitted with infective endocarditis. Which nursing intervention is most appropriate for this child? 1. Start an intravenous line. 2. Place the child in contact isolation. 3. Place the child on seizure precautions. 4. Assist with a lumbar puncture.

1. Start an intravenous line. Infective endocarditis is treated with intravenous antibiotics for 2 to 8 weeks. It is not contagious, so the child is not placed in contact isolation. Seizures are not a risk of infective endocarditis. A lumbar puncture is not a diagnostic test done for infective endocarditis.

The nurse is performing an admission assessment on an infant diagnosed with hydrocephalus and a malfunctioning shunt. Which assessment findings should the nurse expect? Select all that apply. 1. Vomiting 2. Fever 3. Irritability 4. Poor appetite 5. Decreased level of consciousness

1. Vomiting 2. Fever 3. Irritability 4. Poor appetite Signs of shunt malfunction in infants are nonspecific and include irritability, vomiting, poor appetite, disordered sleep, and fever. Older children with shunt malfunction may have a headache, nausea, vomiting, and decreased level of consciousness.

A child with human immunodeficiency virus is started on sulfamethoxazole and trimethoprim (Bactrim) for Pneumocystis carinii pneumonia (PCP) prophylaxis. The recommended dose is based on the trimethoprim (TMP) component and is 15 to 20 mg TMP/kg/day in divided doses every 6 to 8 hours. The child weighs 6.8 kg. The highest dose of TMP the child can receive a day is ________. Round your answer to the nearest whole number.

136 6.8 kg (the child's weight) is multiplied by 20 mg. This yields the answer, which is 136 mg a day.

A child with the diagnosis of Wiskott-Aldrich syndrome has been ordered an IV infusion of gamma globulin. The child weighs 20 pounds. The healthcare provider orders: gamma globulin 2 g/kg IV over 12 hours. Calculate how many grams of gamma globulin will be given IV.

18 g

A child recently had a heart transplant and the nurse teaches the parents the importance of administering cyclosporine A. Which statement by the parents indicates an appropriate understanding of the teaching session? 1. "Cyclosporin A reduces serum-cholesterol level." 2. "Cyclosporin A prevents rejection." 3. "Cyclosporin A treats hypertension." 4. "Cyclosporin A treats infections."

2. "Cyclosporin A prevents rejection." Cyclosporin A is given to prevent rejection. Lovastatin is given to reduce serum-cholesterol level, calcium channel blockers may be used to treat hypertension, and an antibiotic may be given to treat an infection.

Which statement made by a parent during a well-child visit would cause the nurse to suspect the child has cerebral palsy? 1. "My 6-month-old baby is rolling from back to prone now." 2. "My 3-month-old seems to have floppy muscle tone." 3. "My 8-month-old can sit without support." 4. "My 10-month-old is not walking."

2. "My 3-month-old seems to have floppy muscle tone." Children with cerebral palsy are delayed in meeting developmental milestones. The infant with hypotonia is showing a clinical manifestation of cerebral palsy. A baby rolls over from back to prone at 6 months, sits without support at 8 months, and walks at 12 months.

Which intervention is considered supportive care for a family whose infant has died from sudden infant death syndrome (SIDS)? 1. Interviewing parents to determine the cause of the SIDS incident 2. Allowing parents to hold, touch, and rock the infant 3. Sheltering parents from the grief by not giving them any personal items of the infant, such as footprints 4. Advising parents that an autopsy is not necessary

2. Allowing parents to hold, touch, and rock the infant The parents should be allowed to hold, touch, and rock the infant, giving them a chance to say good-bye to their baby. The other options are nontherapeutic. The death of an infant without a known medical condition is an indication for an autopsy.

The nurse is caring for a pediatric client in Bryant skin traction. Which nursing intervention is most appropriate for this client? 1. Remove the adhesive traction straps daily to prevent skin breakdown. 2. Check the traction frequently to ensure that proper alignment is maintained. 3. Place the child in a prone position to maintain good alignment. 4. Move the child as infrequently as possible to maintain traction.

2. Check the traction frequently to ensure that proper alignment is maintained. The traction apparatus should be checked frequently to ensure that proper alignment is maintained. The adhesive straps should not be removed. The child should be placed in a supine position, and frequent repositioning is necessary to prevent complications of immobility.

A child is ready for discharge after surgery for a myelomeningocele repair. Before discharge, the nurse works with the parents to establish a catheterization schedule to prevent urinary tract infection. With what frequency should the nurse instruct the parents to catheterize the child? 1. Every 1 to 2 hours 2. Every 3 to 4 hours 3. Every 6 to 8 hours 4. Every 10 to 12 hours

2. Every 3 to 4 hours To decrease the incidence of bladder or urinary tract infections, catheterization should occur every 3 to 4 hours.

A school-age client diagnosed with diabetes insipidus (DI) is admitted to the pediatric unit. Which laboratory value does the nurse anticipate for this client based on the diagnosis? 1. Hyperglycemia 2. Hypernatremia 3. Hypercalcemia 4. Hypoglycemia

2. Hypernatremia In all forms of diabetes insipidus, serum sodium can increase to pathologic levels, so hypernatremia can occur and should be treated. The glucose level is not affected, so hypoglycemia or hyperglycemia is not caused by the diabetes insipidus. Hypercalcemia (high calcium) does not occur with this endocrine disorder.

The nurse explains to the parents of a child with a severe burn that wearing of an elastic pressure garment (Jobst stocking) during the rehabilitative stage can help with the prevention of which complication? 1. Poor circulation 2. Hypertrophic scarring 3. Pain 4. Formation of thrombus in the burn area

2. Hypertrophic scarring During the rehabilitation stage, Jobst stockings or pressure garments are used to reduce development of hypertrophic scarring and contractures.

The adolescent is seen in the clinic for a consultation to treat severe acne. The adolescent has tried other medications, but the acne has not been responsive. The nurse knows that what medication is the most effective for this client with severe acne? 1. Oral contraceptives 2. Isotretinoin 3. Antibiotics 4. Benzoyl peroxide

2. Isotretinoin Isotretinoin is reserved for severe acne that is not responsive to other therapies.

The nurse is caring for a child with disseminated intravascular coagulation (DIC). Which nursing intervention is a priority for this child? 1. Frequent ambulation 2. Maintenance of skin integrity 3. Monitoring of fluid restriction 4. Preparation for x-ray procedures

2. Maintenance of skin integrity Impairment of skin integrity can lead to bleeding in DIC. The child with DIC should be placed on bed rest. Fluids need to be monitored but will not be restricted, and DIC is not diagnosed with x-ray examination but by serum lab studies.

Which nursing interventions would be best for the nursing diagnosis of Powerlessness Related to Relinquishing Control to the Healthcare Team? Select all that apply. 1. Provide a primary nursing care model. 2. Prepare the child in advance for procedures. 3. Provide optimal pain relief. 4. Explain procedures in developmentally appropriate terms. 5. Incorporate home rituals when possible.

2. Prepare the child in advance for procedures. 4. Explain procedures in developmentally appropriate terms. 5. Incorporate home rituals when possible. Preparation in advance—and in terms that are developmentally appropriate—and incorporating home rituals provide some degree of control, and might reduce the feeling of powerlessness. Providing a primary nursing care model will help decrease anxiety, and providing pain relief will decrease pain.

A pediatric client sustains a minor burn. When teaching the family the treatment for this burn, the nurse would teach that the client's diet should be high in which substance? 1. Fats 2. Protein 3. Minerals 4. Carbohydrates

2. Protein Parents should be taught that management of a minor burn requires a high-calorie, high-protein diet. This is necessary to meet the increased nutritional requirements of healing.

The nurse is caring for the adolescent with systemic lupus erythematosus (SLE). What nursing diagnoses would the nurse address? Select all that apply. 1. Activity intolerance 2. Risk for impaired skin integrity 3. Body image disturbed 4. Ineffective breathing pattern 5. Risk for infection

2. Risk for impaired skin integrity 3. Body image disturbed 5. Risk for infection Nursing diagnoses that may apply to the adolescent with SLE are: risk for impaired skin integrity, risk for activity intolerance, disturbed body image, risk for infection, acute pain, and ineffective family therapeutic regimen management.

A toddler-age client has a tonic-clonic seizure while in a crib in the hospital. The client's jaw is clamped. Which nursing action is the priority? 1. Place a padded tongue blade between the child's jaws. 2. Stay with the child and observe the respiratory status. 3. Prepare the suction equipment. 4. Restrain the child to prevent injury.

2. Stay with the child and observe the respiratory status. During a seizure, the nurse remains with the child, watching for complications. The child's respiratory rate should be monitored. Be sure nothing is placed in the child's mouth during a seizure. Suction equipment should already be set up at the bedside before a seizure begins. The child should not be restrained during a seizure.

A child is on a ventilator in the pediatric intensive care unit (PICU). Which nursing intervention would best meet the psychosocial needs of this child? 1. Allow the parents to remain at the bedside. 2. Touch and talk to the child often. 3. Provide the child with a blanket from home. 4. Provide consistent caregivers.

2. Touch and talk to the child often. Touch and verbal exchanges will aid in psychosocial support. The other responses provide a sense of security.

A child diagnosed with hemophilia plans on participating in a bicycling club. Which recommendation by the nurse is the most appropriate? 1. Consider a swim club instead of the bicycling club. 2. Wear kneepads, elbow pads, and a helmet while bicycling. 3. Participate only in the social activities of the club. 4. Not join the club.

2. Wear kneepads, elbow pads, and a helmet while bicycling. Children with hemophilia should be encouraged to participate in noncontact sports activities. Bicycling is an excellent option and is recommended along with swimming. The child should always use kneepads, elbow pads, and a helmet when participating in a physical sport. Participating only in the social aspects of the club would not encourage physical activity. Discouraging a child from joining a club would not foster growth and development.

The nurse is administering packed RBCs to a child with sickle cell disease (SCD). The nurse is monitoring for a transfusion reaction and knows it is most likely to occur during which time frame? 1. Six hours after the transfusion is given 2. Within the first 20 minutes of administration of the transfusion 3. At the end of the administration of the transfusion 4. Never; children with SCD do not have reactions.

2. Within the first 20 minutes of administration of the transfusion Blood reactions can occur as soon as the blood transfusion begins or within the first 20 minutes. The nurse should remain with the child for the first 20 minutes of the transfusion.

The student nurse is learning a lesson about communicable diseases and how they are spread. On a quiz the next day the nurse uses the information learned in this lesson and demonstrates learning. For a communicable disease to occur what factors must be in place? Select all that apply. 1. Antibodies 2. Toxoid 3. Pathogen 4. Transmission 5. Host

3. Pathogen 4. Transmission 5. Host For a communicable disease to occur, three factors need to be in place: an infectious agent or pathogen, means of transmission, and a host. This is not a factor needed for communicable disease to occur.

A child is prescribed Didanosine (Videx), a nucleoside reverse transcriptase inhibitor, for human immunodeficiency virus (HIV). Which lab value will the nurse monitor closely for this child? 1. Potassium 2. Sodium 3. RBC count 4. Glucose

3. RBC count Didanosine (Videx) causes bone-marrow suppression with resulting anemia. RBC counts are monitored at least monthly for changes. Potassium and sodium are electrolytes, and glucose is a laboratory test for checking diabetes. Didanosine (Videx) does not affect these values.

A nurse begins an infusion of intravenous immune globulin (IVIG) to a child who has combined immunodeficiency disease. Which assessment finding indicates that the nurse should stop the infusion? 1. A mild headache 2. Clear yellow urine 3. Severe shaking, chills, and fever 4. Complaints of being "thirsty"

3. Severe shaking, chills, and fever Hypersensitivity reaction can be seen with IVIG. The infusion should be started slowly and increased if there is no reaction. Shaking, chills, and fever can indicate a reaction. A mild headache is an adverse side effect of IVIG but not a severe reaction. Thirst is not an indication of a reaction. Voiding clear yellow urine is a normal finding.

A toddler is started on digoxin (Lanoxin) for cardiac failure. Which is the initial symptom the nurse would assess if the child develops digoxin (Lanoxin) toxicity? 1. Lowered blood pressure 2. Tinnitus 3. Ataxia 4. A change in heart rhythm

4. A change in heart rhythm An early sign of digoxin (Lanoxin) toxicity is a change in heart rhythm. Digoxin (Lanoxin) toxicity does not cause lowered blood pressure, tinnitus (ringing in the ears), or ataxia (unsteady gait).

Siblings of a client in pediatric intensive care unit (PICU) are preparing to visit their brother, who was hit by a car while riding his bike. Which intervention by the nurse will assist the siblings in preparing for the visit? 1. Spend time developing a relationship with the siblings. 2. Have the parents go with the siblings when they visit. 3. Encourage the siblings to talk to a social worker before seeing their brother. 4. Explain what the siblings will hear and see when they visit.

4. Explain what the siblings will hear and see when they visit. Explaining what the siblings will hear and see when they visit will best prepare them for the visit with their brother. The other responses are good ways to help alleviate stress but won't help prepare the siblings for the visit.

A child who has beta-thalassemia is receiving numerous blood transfusions. The child is also receiving deferoxamine (Desferal) therapy. The parents ask how the deferoxamine will help their child. Which rationale does the nurse use when responding to the parents? 1. It prevents blood transfusion reactions. 2. It stimulates RBC production. 3. It provides vitamin supplementation. 4. It prevents iron overload.

4. It prevents iron overload. Iron overload can be a side effect of a hypertransfusion therapy. Deferoxamine (Desferal) is an iron-chelating drug, which binds excess iron so it can be excreted by the kidneys. It does not prevent blood-transfusion reactions, stimulate RBC production, or provide vitamin supplementation.

The child has just been diagnosed with osteosarcoma, and the nurse is teaching the family regarding this type of cancer. The nurse knows that instruction has been successful when the family states that osteosarcoma is common in which age group? 1. Infants 2. Toddlers 3. Preschool-age children 4. School-age children 5. Adolescents

5. Adolescents Osteosarcoma's peak incidence is during the rapid growth years, at age 13 for girls and 14 for boys.

A goiter is an enlargement or hypertrophy of which gland? a. Thyroid b. Adrenal c. Anterior pituitary d. Posterior pituitary

ANS: A A goiter is an enlargement or hypertrophy of the thyroid gland. Goiter is not associated with the adrenals or the anterior and posterior pituitaries.

What effect does immobilization have on the cardiovascular system? a. Venous stasis b. Increased vasopressor mechanism c. Normal distribution of blood volume d. Increased efficiency of orthostatic neurovascular reflexes

ANS: A Because of decreased muscle contraction, the physiologic effects of immobilization include venous stasis. This can lead to pulmonary emboli or thrombi. A decreased vasopressor mechanism results in orthostatic hypotension, syncope, hypotension, decreased cerebral blood flow, and tachycardia. An altered distribution of blood volume is found, with decreased cardiac workload and exercise tolerance. Immobilization causes a decreased efficiency of orthostatic neurovascular reflexes, with an inability to adapt readily to the upright position and pooling of blood in the extremities in the upright position.

The parents of a child born with disabilities ask the nurse for advice about discipline. The nurse's response should be based on what knowledge concerning discipline? a. Appropriate disciple is essential for the child. b. It may be too difficult to implement appropriate discipline for a special-needs child. c. Discipline is not needed unless the child becomes problematic. d. Discipline is best achieved with punishment for misbehavior.

ANS: A Discipline is essential for the children with disabilities. It provides boundaries within which to test their behavior and teaches them socially acceptable behaviors. It is not too difficult to implement discipline with a special-needs child. The nurse should teach the parents ways to manage the child's behavior before it becomes problematic. Punishment is not effective in managing behavior.

Which term is used to describe a type of fracture that does not produce a break in the skin? a. Simple b. Compound c. Complicated d. Comminuted

ANS: A If a fracture does not produce a break in the skin, it is called a simple or closed fracture. A compound or open fracture is one with an open wound through which the bone protrudes. A complicated fracture is one in which the bone fragments damage other organs or tissues. A comminuted fracture occurs when small fragments of bone are broken from the fractured shaft and lie in the surrounding tissue. These are rare in children.

Which statement appropriately describes a neuroblastoma? a. It is considered to be a "silent" tumor. b. Early diagnosis is usually possible because of the obvious clinical manifestations. c. It is the most common brain tumor in young children. d. It is the most common benign tumor in young children.

ANS: A Neuroblastoma is a silent tumor with few symptoms. In more than 70% of cases, diagnosis is made after metastasis occurs, with the first signs caused by involvement in the nonprimary site. In only 30% of cases is diagnosis made before metastasis. Neuroblastomas are the most common malignant extracranial solid tumors in children. The majority of tumors develop in the adrenal glands or the retroperitoneal sympathetic chain. They are not benign; they metastasize.

A nurse is conducting a staff in-service on childhood cancers. Which is the primary site of osteosarcoma? a. Femur b. Humerus c. Pelvis d. Tibia

ANS: A Osteosarcoma is the most frequently encountered malignant bone cancer in children. The peak incidence is between ages 10 and 25 years. More than half occur in the femur. After the femur, most of the remaining sites are the humerus, tibia, pelvis, jaw, and phalanges.

A young child has just injured an ankle at school. In addition to calling the child's parents, what is the most appropriate immediate action by the school nurse? a. Apply ice b. Observe for edema and discoloration c. Encourage child to assume a comfortable position d. Obtain parental permission for administration of acetaminophen or aspirin

ANS: A Soft-tissue injuries should be iced immediately. In addition to ice, the extremity should be rested, be elevated, and have compression applied. Observing for edema and discoloration, encouraging the child to assume a comfortable position, and obtaining parental permission or administration of acetaminophen or aspirin are not immediate priorities.

The nurse is taking care of an adolescent with osteosarcoma. The parents ask the nurse about treatment. The nurse should make which accurate response about treatment for osteosarcoma? a. Treatment usually consists of surgery and chemotherapy. b. Amputation of the affected extremity is rarely necessary. c. Intensive irradiation is the primary treatment. d. Bone marrow transplantation offers the best chance of long-term survival.

ANS: A The optimal therapy for osteosarcoma is a combination of surgery and chemotherapy. Intensive irradiation and bone marrow transplantation are usually not part of the therapeutic management.

The parents of a child who has just died ask to be left alone so that they can rock their child one more time. In response to their request, what intervention should the nurse implement? a. Grant their request. b. Assess why they feel that this is necessary. c. Discourage this because it will only prolong their grief. d. Kindly explain that they need to say good-bye to their child now and leave.

ANS: A The parents should be allowed to remain with their child after the death. The nurse can remove all of the tubes and equipment and offer the parents the option of preparing the body. This is an important part of the grieving process and should be allowed if the parents desire it. It is important for the nurse to ascertain if the family has any special needs. None of the other options adequately meet the parent's need to grieve.

A cure is no longer possible for a young child with cancer. The nursing staff recognizes that the goal of treatment must shift from cure to palliation. Which is an important consideration at this time? a. The family is included in the decision to shift the goals of treatment. b. The decision must be made by the health professionals involved in the child's care. c. The family needs to understand that palliative care takes place in the home. d. The decision should not be communicated to the family because it will encourage a sense of hopelessness.

ANS: A When the child reaches the terminal stage, the nurse and physician should explore the family's wishes. The family should help decide what interventions will occur as they plan for their child's death. None of the other options address the parent's need to be involved effectively in their child's care.

Which describe avoidance behaviors a parent may exhibit when learning that his or her child has a chronic condition? (Select all that apply.) a. Refuses to agree to treatment b. Shares burden of disorder with others c. Verbalizes possible loss of child d. Withdraws from outside world e. Punishes self because of guilt and shame

ANS: A, D, E A parent who refuses to agree to treatment, withdraws from the outside world, and punishes self because of guilt and shame is exhibiting avoidance coping behaviors. A parent who shares the burden of disorder with others and verbalizes possible loss of child is exhibiting approach coping behaviors.

The nurse should expect to assess which clinical manifestations in an adolescent with Cushing's syndrome? (Select all that apply.) a. Hyperglycemia b. Hyperkalemia c. Hypotension d. Cushingoid features e. Susceptibility to infections

ANS: A, D, E In Cushing's syndrome, physiologic disturbances seen are cushingoid features, hyperglycemia, susceptibility to infection, hypertension, and hypokalemia.

What is an advantage to using a fiberglass cast instead of a plaster cast? a. Is less expensive b. Dries rapidly c. Molds closely to body parts d. Has a smooth exterior

ANS: B A synthetic casting material dries in 5 to 30 minutes as compared with a plaster cast, which takes 10 to 72 hours to dry. Synthetic casts are more expensive. Plaster casts mold closer to body parts. Synthetic casts have a rough exterior, which may scratch surfaces.

A child is diagnosed with juvenile hypothyroidism. The nurse should expect to assess which symptoms associated with hypothyroidism? (Select all that apply.) a. Weight loss b. Sleepiness c. Diarrhea d. Puffiness around the eyes e. Spare hair

ANS: B, D, E A child diagnosed with juvenile hypothyroidism will display sleepiness; dry, periorbital puffiness; and spare hair growth. Weight loss and diarrhea are signs of hyperthyroidism.

To help the adolescent deal with diabetes, the nurse must consider which characteristic of adolescence? a. Desire to be unique b. Preoccupation with the future c. Need to be perfect and similar to peers d. Need to make peers aware of the seriousness of hypoglycemic reactions

ANS: C Adolescence is a time when the individual wants to be perfect and similar to peers. Having diabetes makes adolescents different from their peers. Adolescents do not wish to be unique; they desire to fit in with the peer group and are usually not future oriented. Forcing peer awareness of the seriousness of hypoglycemic reactions would further alienate the adolescent with diabetes since the peer group would likely focus on the differences.

What is a common clinical manifestation of juvenile hypothyroidism? a. Insomnia b. Diarrhea c. Dry skin d. Accelerated growth

ANS: C Dry skin, mental decline, and myxedematous skin changes are associated with juvenile hypothyroidism. Children with hypothyroidism are usually sleepy. Constipation is associated with hypothyroidism. Decelerated growth is common in juvenile hypothyroidism.

The parents of a child who has just been diagnosed with type 1 diabetes ask about exercise. The nurse should provide the parents with what information to address the child's safety needs? a. Exercise will increase blood glucose. b. Exercise should be restricted. c. Extra snacks are needed before exercise. d. Extra insulin is required during exercise.

ANS: C Exercise lowers blood glucose levels, which can be compensated for by extra snacks. Exercise is encouraged and not restricted unless indicated by other health conditions. Extra insulin is contraindicated because exercise decreases blood glucose levels.

What is the initial method of treating osteomyelitis? a. Joint replacement b. Bracing and casting c. Intravenous antibiotic therapy d. Long-term corticosteroid therapy

ANS: C Osteomyelitis is an infection of the bone, most commonly caused by Staphylococcus aureus. The treatment of choice is antibiotics delivered intravenous and then possibly by the oral route. Joint replacement, bracing and casting, and long-term corticosteroids are not indicated for infectious processes.

A 16 year old diagnosed with a chronic illness has recently become rebellious and is taking risks such as missing doses of his medication. What information should the nurse provide the parents to help explain their child's behavior? a. The child at this age requires more discipline. b. At this age, children need more socialization with peers. c. This behavior is seen as a normal part of adolescence. d. This is how the child is asking for more parental involvement in managing stress.

ANS: C Risk taking, rebelliousness, and lack of cooperation are normal parts of adolescence. If the parents increase the amount of discipline, he will most likely be more rebellious. Socialization with peers should be encouraged as a part of adolescence. It is a normal part of adolescence during which the young adult is establishing independence.

Which type of traction uses skin traction on the lower leg and a padded sling under the knee? a. Dunlop b. Bryant's c. Russell d. Buck's extension

ANS: C Russell traction uses skin traction on the lower leg and a padded sling under the knee. The combination of longitudinal and perpendicular traction allows realignment of the lower extremity and immobilizes the hips and knees in a flexed position. Dunlop traction is an upper-extremity traction used for fractures of the humerus. Bryant's traction is skin traction with the legs flexed at a 90-degree angle at the hip. Buck's extension traction is a type of skin traction with the legs in an extended position. It is used primarily for short-term immobilization, before surgery with dislocated hips, for correcting contractures, or for bone deformities such as Legg-Calvé-Perthes disease.

A school-age child diagnosed with leukemia experienced severe nausea and vomiting when receiving chemotherapy for the first time. Which is the most appropriate nursing action to prevent or minimize these reactions with subsequent treatments? a. Encourage drinking large amounts of favorite fluids. b. Encourage child to take nothing by mouth (remain NPO) until nausea and vomiting subside. c. Administer an antiemetic at least 30 minutes before chemotherapy begins. d. Administer an antiemetic as soon as child has nausea

ANS: C The most beneficial regimen to minimize nausea and vomiting associated with chemotherapy is to administer the antiemetic 30 minutes to an hour before the chemotherapy is begun. The goal is to prevent anticipatory symptoms. Drinking fluids will add to the discomfort of the nausea and vomiting. Waiting until nausea and vomiting subside will help with this episode, but the child will have the discomfort and be at risk for dehydration. Administering an antiemetic as soon as the child has nausea does not prevent anticipatory nausea.

A child diagnosed with hypoparathyroidism is receiving vitamin D therapy. The parents should be advised to watch for which sign of vitamin D toxicity? a. Headache and seizures b. Physical restlessness and voracious appetite without weight gain c. Weakness and lassitude d. Anorexia and insomnia

ANS: C Vitamin D toxicity can be a serious consequence of therapy. Parents are advised to watch for signs including weakness, fatigue, lassitude, headache, nausea, vomiting, and diarrhea. Renal impairment is manifested through polyuria, polydipsia, and nocturia. Headaches may be a sign of vitamin D toxicity, but seizures are not. Physical restlessness and a voracious appetite with weight loss are manifestations of hyperthyroidism. Anorexia and insomnia are not characteristic of vitamin D toxicity.

The nurse is caring for a school-age child with hyperthyroidism (Graves' disease). Which clinical manifestations should the nurse monitor that may indicate a thyroid storm? (Select all that apply.) a. Constipation b. Hypotension c. Hyperthermia d. Tachycardia e. Vomiting

ANS: C, D, E A child with a thyroid storm will have severe irritability and restlessness, vomiting, diarrhea, hyperthermia, hypertension, severe tachycardia, and prostration.

What should the nurse identify as major fears in the school-age child who is hospitalized with a chronic illness? (Select all that apply.) a. Altered body image b. Separation from peer group c. Bodily injury d. Mutilation e. Being left alone

ANS: C, D, E Bodily injury, mutilation, and being left alone are all major fears of the school age. Altered body image and separation from peers are major fears in the adolescent.

The nurse is preparing a child for possible alopecia from chemotherapy. Which intervention should be included? a. Explain to child that hair usually regrows in 1 year. b. Advise child to expose head to sunlight to minimize alopecia. c. Explain to child that wearing a hat or scarf is preferable to wearing a wig. d. Explain to child that when hair regrows, it may have a slightly different color or texture.

ANS: D Alopecia is a side effect of certain chemotherapeutic agents. When the hair regrows, it may be a different color or texture. The hair usually grows back within 3 to 6 months after cessation of treatment. The head should be protected from sunlight to avoid sunburn. Children should choose the head covering they prefer.

Which symptom is considered a cardinal sign of diabetes mellitus? a. Nausea b. Seizures c. Impaired vision d. Frequent urination

ANS: D Hallmarks of diabetes mellitus are glycosuria, polyuria, and polydipsia. Nausea and seizures are not clinical manifestations of diabetes mellitus. Impaired vision is a long-term complication of the disease.

Which is the most appropriate response to a school-age child who asks if she can talk to her dying sister? a. "You need to speak loudly so she can hear you." b. "Holding her hand would be better because at this point she can't hear you." c. "Although she can't hear you, she can feel your presence so sit close to her." d. "Even though she will probably not answer you, she can still hear what you say to her."

ANS: D Hearing is the last sense to cease before death. Talking to the dying child is important both for the child and for the family. There is no evidence that the dying process decreases hearing acuity; therefore, the sister should speak at a normal volume. The sibling should be encouraged to speak to the child, as well as sit close to the bed and hold the child's hand.

The parent of a child diagnosed with diabetes mellitus asks the nurse when urine testing will be necessary. The nurse should explain that urine testing is necessary for which reason? a. Glucose is needed before administration of insulin. b. Glucose is needed 4 times a day. c. Glycosylated hemoglobin is required. d. Ketonuria is suspected.

ANS: D Urine testing is still performed to detect evidence of ketonuria. Urine testing for glucose is no longer indicated for medication administration because of the poor correlation between blood glucose levels and glycosuria. Glycosylated hemoglobin analysis is performed on a blood sample.

Match the child's concept of death with their behavioral response. A. Infant B. Toddler C. Preschool-age child D. School-age child E. Adolescent 1. Understands difference between temporary separation and death. 2. Senses emotions of caregivers and altered routines. 3. Capable of understanding death, recognizes all people and self will die. 4. No understanding of true concept of death. 5. Believes death is temporary and the person will return.

Answer: 1/D, 2/A, 3/E, 4/B, 5/C School-age child—Understands difference between temporary separation and death. Infant—Senses emotions of caregivers, and altered routines. Adolescent—Capable of understanding death, recognizes all people and self will die. Toddler—No understanding of true concept of death. Preschool-age child— Believes death is temporary and the person will return.

A school-aged child is admitted with pneumococcal meningitis. The child weighs 44 pounds. The physician orders: ceftriaxone (Rocephin) 50 mg/kg/dose IV every 12 hours three times and then every 24 hours. Calculate how many mg/dose of ceftriaxone the child will receive and then calculate mL/hr to infuse via pump. Supply on hand is: a premix of ceftriaxone 1 g/50 mL, administer over 30 minutes.

Answer: 1000 mg/dose; 100 mL/hr Explanation: The child will receive 1000 mg/dose of ceftriaxone, then 100 mL/hr to infuse via pump.

A child is prescribed cephalexin for treatment of cellulitis. The child weighs 15 kg. The pediatrician orders: cephalexin 40 mg/kg/day PO, give twice a day. Medication on hand: 250 mg/5 mL Calculate how many mLs the nurse must draw up for each dose.

Explanation: 6 mL

Which type of seizure may be difficult to detect? a. Absence b. Generalized c. Simple partial d. Complex partial

a. Absence Absence seizures may go unrecognized because little change occurs in the child's behavior during the seizure except for a period of unconsciousness lasting less than 10 seconds. Generalized, simple partial, and complex partial seizures all have clinical manifestations that are observable.

Which test is never performed on a child who is awake? a. Oculovestibular response b. Doll's head maneuver c. Funduscopic examination for papilledema d. Assessment of pyramidal tract lesions

a. Oculovestibular response The oculovestibular response (caloric test) involves the instillation of ice water into the ear of a comatose child. The caloric test is painful and is never performed on a child who is awake or one who has a ruptured tympanic membrane. Doll's head maneuver, funduscopic examination, and assessment of pyramidal tract lesions can be performed on children who are awake.

In which situation is there the greatest risk that a newborn infant will have a congenital heart defect (CHD)? a. Trisomy 21 detected on amniocentesis b. Family history of myocardial infarction c. Father has type 1 diabetes mellitus d. Older sibling born with Turner's syndrome

a. Trisomy 21 detected on amniocentesis The incidence of congenital heart disease is approximately 50% in children with trisomy 21 (Down syndrome). A family history of congenital heart disease, not acquired heart disease, increases the risk of giving birth to a child with CHD. Infants born to mothers who are insulin dependent have an increased risk of CHD. Infants identified as having certain genetic defects, such as Turner's syndrome, have a higher incidence of CHD.

A school-age child is admitted in vaso-occlusive sickle cell crisis. The child's care should include which intervention? (Select all that apply.) a. Correction of acidosis b. Adequate hydration c. Pain management d. Administration of heparin e. Replacement of factor VIII

b. Adequate hydration c. Pain management The management of crises includes adequate hydration, minimizing energy expenditures, pain management, electrolyte replacement, and blood component therapy if indicated. The acidosis will be corrected as the crisis is treated. Heparin and factor VIII are not indicated in the treatment of vaso-occlusive sickle cell crisis.

The nurse is admitting a school-age child with suspected Guillain-Barré syndrome (GBS). Which nursing intervention is a priority in the care for this child? a. Monitoring intake and output b. Assessing respiratory efforts c. Placing on a telemetry monitor d. Obtaining laboratory studies

b. Assessing respiratory efforts Treatment of GBS is primarily supportive. In the acute phase, patients are hospitalized because respiratory and pharyngeal involvement may require assisted ventilation, sometimes with a temporary tracheotomy. Treatment modalities include aggressive ventilatory support in the event of respiratory compromise, administration of intravenous immunoglobulin (IVIG), and sometimes steroids; plasmapheresis and immunosuppressive drugs may also be used. Monitoring intake and output, telemetry monitoring, and obtaining laboratory studies may be part of the plan of care but are not the priority.

Which finding should cause the nurse to suspect a diagnosis of spastic cerebral palsy? a. Tremulous movements at rest and with activity b. Positive Babinski reflex c. Writhing, uncontrolled, involuntary movements d. Clumsy, uncoordinated movements

b. Positive Babinski reflex Spastic cerebral palsy, the most common type of cerebral palsy, will manifest with persistent primitive reflexes, positive Babinski reflex, ankle clonus, exaggerated stretch reflexes, and eventual development of contractures. The child's muscles are very tight and any stimuli may cause a sudden jerking movement. Tremulous movements are characteristic of rigid/tremor/atonic cerebral palsy. Slow, writhing, uncontrolled, involuntary movements occur with athetoid or dyskinetic cerebral palsy. Clumsy movements, loss of coordination, equilibrium, and kinesthetic sense occur in ataxic cerebral palsy.

A nurse is teaching nursing students the physiology of congenital heart defects. Which defect results in decreased pulmonary blood flow? a. Atrial septal defect b. Tetralogy of Fallot c. Ventricular septal defect d. Patent ductus arteriosus

b. Tetralogy of Fallot Tetralogy of Fallot results in decreased blood flow to the lungs. The pulmonic stenosis increases the pressure in the right ventricle, causing the blood to go from right to left across the interventricular septal defect. Atrial and ventricular septal defects and patent ductus arteriosus result in increased pulmonary blood flow.

What is the initial goal for the treatment of secondary hypertension? a. Weight control and diet b. Treating the underlying disease c. Administration of digoxin d. Administration of â-adrenergic receptor blockers

b. Treating the underlying disease Identification of the underlying disease should be the first step in treating secondary hypertension. Weight control and diet are nonpharmacologic treatments for primary hypertension. Digoxin is indicated in the treatment of congestive heart failure.

What is an important nursing consideration when suctioning a young child who has had heart surgery? a. Perform suctioning at least every hour. b. Suction for no longer than 30 seconds at a time. c. Administer supplemental oxygen before and after suctioning. d. Expect symptoms of respiratory distress when suctioning.

c. Administer supplemental oxygen before and after suctioning. If suctioning is indicated, supplemental oxygen is administered with a manual resuscitation bag before and after the procedure to prevent hypoxia. Suctioning should be done only as indicated, not on a routine basis. The child should be suctioned for no more than 5 seconds at one time. Symptoms of respiratory distress are to be avoided by using the appropriate technique.

Which defect results in increased pulmonary blood flow? a. Pulmonic stenosis b. Tricuspid atresia c. Atrial septal defect d. Transposition of the great arteries

c. Atrial septal defect Atrial septal defect results in increased pulmonary blood flow. Blood flows from the left atrium (higher pressure) into the right atrium (lower pressure) and then to the lungs via the pulmonary artery. Pulmonic stenosis is an obstruction to blood flowing from the ventricles. Tricuspid atresia results in decreased pulmonary blood flow. Transposition of the great arteries results in mixed blood flow.

The nurse is caring for a neonate with suspected meningitis. Which clinical manifestations should the nurse prepare to assess if meningitis is confirmed? (Select all that apply.) a. Headache b. Photophobia c. Bulging anterior fontanel d. Weak cry e. Poor muscle tone

c. Bulging anterior fontanel d. Weak cry e. Poor muscle tone Assessment findings in a neonate with meningitis include bulging anterior fontanel, weak cry, and poor muscle tone. Headache and photophobia are signs seen in an older child

Which type of fracture describes traumatic separation of cranial sutures? a. Basilar b. Compound c. Diastatic d. Depressed

c. Diastatic Diastatic skull fractures are traumatic separations of the cranial sutures. A basilar fracture involves the basilar portion of the frontal, ethmoid, sphenoid, temporal, or occipital bone. A compound fracture has the bone exposed through the skin. A depressed fracture has the bone pushed inward, causing pressure on the brain.

Clinical manifestations of increased intracranial pressure (ICP) in infants are: (Select all that apply.) a. Low-pitched cry b. Sunken fontanel c. Drowsiness d. Irritability e. Distended scalp veins f. Increased blood pressure

c. Drowsiness d. Irritability e. Distended scalp veins Drowsiness, irritability, and distended scalp veins are signs of increased ICP in infants. Low-pitched cry, sunken fontanel, and increased blood pressure are not clinical manifestations associated with ICP in infants.

What is the initial clinical manifestation of generalized seizures? a. Being confused b. Feeling frightened c. Losing consciousness d. Seeing flashing lights

c. Losing consciousness Loss of consciousness is a frequent occurrence in generalized seizures and is the initial clinical manifestation. Being confused, feeling frightened, and seeing flashing lights are clinical manifestations of a complex partial seizure.

A nurse is conducting discharge teaching to parents about the care of their infant after cardiac surgery. The nurse instructs the parents to notify the physician if what conditions occur? (Select all that apply.) a. Respiratory rate of 36 breaths/minute at rest b. Appetite slowly increasing c. Temperature above 37.7° C (100° F) d. New, frequent coughing e. Turning blue or bluer than normal

c. Temperature above 37.7° C (100° F) d. New, frequent coughing e. Turning blue or bluer than normal The parents should be instructed to notify the physician after their infant's cardiac surgery for a temperature above 37.7° C (100° F); new, frequent coughing; and any episodes of the infant turning blue or bluer than normal. A respiratory rate of 36 breaths/minute at rest for an infant is within normal expectations, and it is expected that the appetite will increase slowly.

José is 4 year old. Preoperative teaching for a 4-year-old child scheduled for a cardiac catheterization should be done with what primary consideration in mind? a. Directed at his parents because he is too young to understand. b. Detailed in regard to the actual procedures so he will know what to expect. c. Done several days before the procedure so that he will be prepared. d. Adapted to his level of development so that he can understand.

d. Adapted to his level of development so that he can understand. Preoperative teaching should always be directed at the child's stage of development. The caregivers also benefit from the same explanations. The parents may ask additional questions, which should be answered, but the child needs to receive the information based on developmental level. This age-group does not understand in-depth descriptions. Preschoolers should be prepared close to the time of the cardiac catheterization.

Which clinical manifestation should the nurse expect when a child diagnosed with sickle cell anemia experiences an acute vaso-occlusive crisis? a. Circulatory collapse b. Cardiomegaly, systolic murmurs c. Hepatomegaly, intrahepatic cholestasis d. Painful swelling of hands and feet, painful joints

d. Painful swelling of hands and feet, painful joints A vaso-occlusive crisis is characterized by severe pain in the area of involvement. If in the extremities, painful swelling of the hands and feet is seen; if in the abdomen, severe pain resembles that of acute surgical abdomen; and if in the head, stroke and visual disturbances occur. Circulatory collapse results from sequestration crises. Cardiomegaly, systolic murmurs, hepatomegaly, and intrahepatic cholestasis result from chronic vaso-occlusive phenomena.

A child is diagnosed with sickle cell disease. The parents are unsure how their child contracted the disease. Which explanation by the nurse is the most appropriate? 1. "Both the mother and the father have the sickle cell trait." 2. "The mother has the trait, but the father doesn't." 3. "The father has the trait, but the mother doesn't." 4. "The mother has sickle cell disease, but the father doesn't have the disease or the trait."

1. "Both the mother and the father have the sickle cell trait." Sickle cell disease is an autosomal recessive disorder; both parents must have the trait in order for a child to have the disease.

A 5-year-old child is admitted to the hospital with increased intracranial pressure after a motor vehicle struck the child. The child weighs 15 kg. The neurosurgeon orders: Mannitol 0.5 g/kg/10 minutes IV first, followed by Mannitol 0.25 g/kg IV every 4 hours. Medication on hand: Mannitol 100 g/500mL D5W. Calculate how many mL/hr to set the IV pump to infuse the Mannitol ordered every 4 hours.

18.75 or 18.8 mL/hr

An adolescent is admitted to the intensive care unit (ICU) with diabetic ketoacidosis. The client weighs 115 pounds. The healthcare provider orders: Regular insulin 0.15 units/kg bolus via IVF, then regular insulin 0.1 units/kg/hr in 0.9 percent NSS Medication on hand: 250 mL 0.9 percent NSS with 250 units of regular insulin. Calculate the mL/hr for the continuous infusion of regular insulin at 0.1 unit/kg/hr in 0.9 percent NSS.

7.4 mL/hr

When assessing the child with osteogenesis imperfecta, the nurse should expect to observe clinical feature? a. Discolored teeth b. Below-normal intelligence c. Increased muscle tone d. Above-average stature

ANS: A Children with osteogenesis imperfecta have incomplete development of bones, teeth, ligaments, and sclerae. Teeth are discolored because of abnormal enamel. Despite their appearance, children with osteogenesis imperfecta have normal or above-normal intelligence. The child with osteogenesis imperfecta has weak muscles and decreased muscle tone. Because of compression fractures of the spine, the child appears short.

The nurse is admitting a child with a Wilms' tumor. Which is the initial assessment finding associated with this tumor? a. Abdominal swelling b. Weight gain c. Hypotension d. Increased urinary output

ANS: A The initial assessment finding with a Wilms' (kidney) tumor is abdominal swelling. Weight loss, not weight gain, may be a finding. Hypertension occasionally occurs with a Wilms' tumor. Urinary output is not increased, but hematuria may be noted.

A boy with leukemia screams whenever he needs to be turned or moved. Which is the most probable cause of this pain? a. Edema b. Bone involvement c. Petechial hemorrhages d. Changes within the muscles

ANS: B The invasion of the bone marrow with leukemic cells gradually causes weakening of the bone and a tendency toward fractures. As leukemic cells invade the periosteum, increasing pressure causes severe pain. Edema, petechial hemorrhages, and changes within the muscles would not cause severe pain.

Which is often administered to prevent or control hemorrhage in a child diagnosed with cancer? a. Nitrosoureas b. Platelets c. Whole blood d. Corticosteroids

ANS: B Most bleeding episodes can be prevented or controlled with the administration of platelet concentrate or platelet-rich plasma. Nitrosoureas, whole blood, and corticosteroids would not prevent or control hemorrhage.

The feeling of guilt that the child "caused" the disability or illness is especially critical in which child? a. Toddler b. Preschooler c. School-age child d. Adolescent

ANS: B Preschoolers are most likely to be affected by feelings of guilt that they caused the illness/disability or are being punished for wrongdoings. Toddlers are focused on establishing their autonomy. The illness will foster dependency. The school-age child will have limited opportunities for achievement and may not be able to understand limitations. Adolescents are faced with the task of incorporating their disabilities into their changing self-concept.

Diabetes insipidus is a disorder of which organ? a. Anterior pituitary b. Posterior pituitary c. Adrenal cortex d. Adrenal medulla

ANS: B The principal disorder of posterior pituitary hypofunction is diabetes insipidus. The anterior pituitary produces hormones such as growth hormone, thyroid-stimulating hormone, adrenocorticotropic hormone, gonadotropin, prolactin, and melanocyte-stimulating hormone. The adrenal cortex produces aldosterone, sex hormones, and glucocorticoids. The adrenal medulla produces catecholamines.

What is an appropriate nursing intervention when caring for a child in traction? a. Remove adhesive traction straps daily to prevent skin breakdown. b. Assess for tightness, weakness, or contractures in uninvolved joints and muscles. c. Provide active range-of-motion exercises to affected extremity 3 times a day. d. Keep child in one position to maintain good alignment.

ANS: B Traction places stress on the affected bone, joint, and muscles. The nurse must assess for tightness, weakness, or contractures developing in the uninvolved joints and muscles. The adhesive straps should be released/replaced only when absolutely necessary. Active, passive, or active with resistance exercises should be carried out for the unaffected extremity only. Movement is expected with children. Each time the child moves, the nurse should check to ensure that proper alignment is maintained.

Which immunization should be considered carefully before being given to a child receiving chemotherapy for cancer? a. Tetanus vaccine b. Inactivated poliovirus vaccine c. Diphtheria, pertussis, tetanus (DPT) d. Measles, rubella, mumps

ANS: D The vaccine used for measles, mumps, and rubella is a live virus and can result in an overwhelming infection. Tetanus vaccine, inactivated poliovirus vaccine, and diphtheria, pertussis, tetanus (DPT) are not live virus vaccines.

What should the nurse consider when preparing a school-age child and the family for heart surgery? a. Not showing unfamiliar equipment b. Letting child hear the sounds of an electrocardiograph monitor c. Avoiding mentioning postoperative discomfort and interventions d. Explaining that an endotracheal tube will not be needed if the surgery goes well

b. Letting child hear the sounds of an electrocardiograph monitor The child and family should be exposed to the sights and sounds of the intensive care unit. All positive, nonfrightening aspects of the environment are emphasized. The child should be shown unfamiliar equipment, and its use should be demonstrated on a doll. Carefully prepare the child for the postoperative experience, including intravenous lines, incision, and endotracheal tube.

The nurse is providing an educational session for parents with children diagnosed with iron deficiency anemia. Which statements will the nurse include educate about the normal functions of RBCs? Select all that apply. 1. "RBCs transport oxygen from the lungs to the tissue." 2. "RBCs transport carbon dioxide to the lungs." 3. "RBCs protect the body against bacterial invaders." 4. "RBCs form hemostatic plugs to stop bleeding." 5. "RBCs are responsible for psychosocial development."

1. "RBCs transport oxygen from the lungs to the tissue." 2. "RBCs transport carbon dioxide to the lungs." The normal function of RBCs includes transporting oxygen from the lungs to the tissue and transporting carbon dioxide to the lungs. WBCs protect the body against bacterial invaders. Platelets form hemostatic plugs to stop bleeding. RBCs are not directly responsible for psychosocial development.

The nurse is working in an adolescent medical clinic. What can the nurse anticipate when comparing adolescents in the clinic with chronic conditions to their peers? 1. A high level self-esteem 2. A concern for their parents 3. An altered body image 4. A decreased concern about their appearance

3. An altered body image As adolescents develop a sense of identity, they are focused on themselves and the present. They have a heightened concern about their appearance but may have inaccurate assessments of their body image and low self-esteem when comparing their bodies with those of their peers.

The nurse is assessing a child who was just admitted to the hospital for observation after a head injury. What is the most essential part of the nursing assessment to detect early signs of a worsening condition? a. Posturing b. Vital signs c. Focal neurologic signs d. Level of consciousness

d. Level of consciousness The most important nursing observation is assessment of the child's level of consciousness. Alterations in consciousness appear earlier in the progression of head injury than do alterations of vital signs or focal neurologic signs. Neurologic posturing indicates neurologic damage. Vital signs and focal neurologic signs are later signs of progression when compared with level-of-consciousness changes.

With a group of new parents, the nurse is reviewing treatment for viral illnesses such as influenza. Which statement by the parents indicates appropriate understanding of the teaching session? 1. "Some over-the-counter medications contain aspirin." 2. "Acetaminophen is good for treatment of fevers in young children." 3. "I can use ibuprofen as needed when my child has aches and pains." 4. "Aspirin is acceptable if my child does not have a virus."

1. "Some over-the-counter medications contain aspirin." Reye syndrome is a serious consequence of aspirin use in children with viral illnesses. Over-the-counter medications should be checked to see whether they contain aspirin before being used. Aspirin is avoided in children. Ibuprofen and acetaminophen are acceptable to use in children.

An HIV-positive mother states she is relieved after the birth of her child to hear that the child is HIV-negative. Which response by the nurse is the most appropriate? 1. "Symptoms could still appear over the next 2 years." 2. "You took good care of yourself, so your child did not get HIV." 3. "We will assess for signs of pneumonia to be sure." 4. "The test will be repeated in 1 week to verify the negative status."

1. "Symptoms could still appear over the next 2 years." Symptoms of HIV could still manifest within the first 2 years. An infant is retested 1 to 2 months after the initial negative result. The HIV-positive mother can infect the newborn regardless of how well she takes care of herself once she is HIV-positive. There is no reason to assess for signs of pneumonia if the newborn is HIV-negative.

The nurse prepares a DTaP (diphtheria, tetanus toxoid, and acellular pertussis) immunization for a 6-month-old infant. To administer this injection safely, the nurse chooses which needle, size and length, injection type, and injection site? 1. 25-gauge, 5/8-inch needle; IM (intramuscular); anterolateral thigh. 2. 22-gauge, 1/2-inch needle; IM (intramuscular); ventrogluteal. 3. 25-gauge, 5/8-inch needle; ID (intradermal); deltoid. 4. 25-gauge, 3/4-inch needle; SQ (subcutaneous); anterolateral thigh.

1. 25-gauge, 5/8-inch needle; IM (intramuscular); anterolateral thigh. The dose of DTaP is 0.5 cc or 0.5 mL, to be given with a 22 to 25-gauge, 5/8- to 3/4-inch needle; IM (intramuscularly). The only safe intramuscular injection site for a 6-month-old infant is the anterolateral thigh.

The nurse has admitted a child with tricuspid atresia. The nurse would expect which initial lab result? 1. A high hemoglobin 2. A low hematocrit 3. A high WBC count 4. A low platelet count

1. A high hemoglobin The child's bone marrow responds to chronic hypoxemia by producing more RBCs to increase the amount of hemoglobin available to carry oxygen to the tissues. This occurs in cases of cyanotic heart defects such as tricuspid atresia. Therefore, the hematocrit would not be low, the WBC count would not be high (unless an infection were present), and the platelets would be normal.

The nurse is admitting an infant diagnosed with supraventricular tachycardia. Which intervention is the priority for this infant? 1. Apply ice to the face. 2. Perform Valsalva's maneuver. 3. Administer a beta blocker. 4. Prepare for cardioversion.

1. Apply ice to the face. Supraventricular tachycardia episodes are initially treated with vagal maneuvers to slow the heart rate when the infant is stable. In stable infants, the application of ice or iced saline solution to the face can reduce the heart rate. The infant is not capable of performing Valsalva's maneuver. Calcium channel blockers, not beta blockers, are the drugs of choice. Cardioversion is used in an urgent situation, but is not typically the initial treatment.

It is important that parents of adolescents with special needs transition care of the adolescent so they can learn to make good decisions on their own. Which items are considered transitional needs? Select all that apply. 1. Attending school 2. Discussing sexual matters 3. Letting most friends know of the medical condition 4. Socialization beyond the family 5. To write his or her own individualized healthcare plan

1. Attending school 2. Discussing sexual matters 4. Socialization beyond the family Transitional needs toward independence include attending school, discussion of sexual matters, and socialization beyond the family. The other areas are not transitional needs.

The family and school-age child are at the healthcare clinic for immunizations. The nurse takes the time to talk with the child and family about reducing the transmission of infection. What practices should the nurse suggest for the family? Select all that apply. 1. Do not share dishes, utensils, and cups. 2. Sanitize toys every week with Lysol. 3. Use alcohol-based hand sanitizer with the child after eating and toileting. 4. Cough or sneeze into cloth tissue 5. Dispose of diapers in a closed container.

1. Do not share dishes, utensils, and cups. 5. Dispose of diapers in a closed container. Teach families to reduce transmission of infection among family members with the following practices: use disposable tissues and dispose immediately after using, wash hands thoroughly with soap/water after all contact with diapers/tissues/mucous, sneeze/cough into elbow, wash hands with soap/water after eating and toileting, do not share dishes/utensils/cups, wash hands thoroughly before preparing food and again several times during the preparation process, use soapy warm water to wash dishes/cutting boards, wipe counters/surfaces that are used for diaper changes or that the child touches with disinfectant, make sure diaper changing area is well away from food prep areas, dispose of diapers in closed containers. This is a practice that the nurse should suggest for the family.

A child is admitted with a diagnosis of early localized Lyme disease. Which clinical manifestations would the nurse expect to find on the initial assessment of this client? Select all that apply. 1. Erythema 5 to 15 cm in diameter 2. Hyperactivity 3. Cranial nerve palsies 4. Fever 5. Headache

1. Erythema 5 to 15 cm in diameter 4. Fever 5. Headache Erythema, fever, and headache are signs/symptoms in the early localized stage of Lyme disease. Cranial nerve palsies are seen in the early disseminated stage of the disease. Malaise, rather than hyperactivity, is seen with this disease.

The nurse is teaching parents how to prevent a sickle cell crisis in the child with sickle cell disease. Which precipitating factors to a sickle cell crisis will the nurse include in the explanation? Select all that apply. 1. Fever 2. Dehydration 3. Regular exercise 4. Altitude 5. Increased fluid intake

1. Fever 2. Dehydration 4. Altitude Fever, dehydration, and altitude are precipitating factors contributing to a sickle cell crisis. Regular exercise and increased fluid intake are recommended activities for a child with sickle cell disease and will not contribute to a sickle cell crisis.

The nurse is providing care to a school-age client with neutropenia. Which clinical manifestations does the nurse anticipate when assessing this client? Select all that apply. 1. Fever 2. Fatigue 3. Tachycardia 4. Hypertension 5. Tachypnea

1. Fever 2. Fatigue 3. Tachycardia 5. Tachypnea A school-age client who is diagnosed with neutropenia, or a decrease in WBCs, will likely exhibit fever, fatigue, tachycardia, and tachypnea (as a result of congestive heart failure). The nurse would not anticipate that the client will exhibit hypertension as a result of the diagnosis.

The nurse educator is teaching a group of nursing students about the endocrine system. Which statements are appropriate for the educator to include in the teaching session? Select all that apply. 1. Gonadotropin-releasing hormone stimulates the anterior pituitary to produce LH and FSH. 2. Growth hormone regulates linear bone growth and growth of all tissues. 3. Antidiuretic hormone regulates urine concentration by the kidneys. 4. Thyroid hormone regulates serum calcium levels and phosphorus excretion. 5. Parathyroid hormone regulates metabolism of cells and body heat production.

1. Gonadotropin-releasing hormone stimulates the anterior pituitary to produce LH and FSH. 2. Growth hormone regulates linear bone growth and growth of all tissues. 3. Antidiuretic hormone regulates urine concentration by the kidneys. All statements are correct except the statements regarding the thyroid hormone and the parathyroid hormone. The thyroid hormone regulates metabolism of the cells and body heat production, not the parathyroid hormone. The parathyroid hormone regulates serum calcium levels and phosphorus excretion.

A young school-age child is in the pediatric intensive-care unit (PICU) with a fractured femur and head trauma. The child was not wearing a helmet while riding his new bicycle on the highway and collided with a car. Which nursing diagnoses may be appropriate for this family? Select all that apply. 1. Guilt Related to Lack of Child Supervision and Safety Precautions 2. Family Coping: Compromised, Related to the Critical Injury of the Child 3. Parental Role Conflict Related to Child's Injuries and PICU Policies 4. Knowledge Deficit Related to Home Care of Fractured Femur 5. Anger Related to Feelings of Helplessness

1. Guilt Related to Lack of Child Supervision and Safety Precautions 2. Family Coping: Compromised, Related to the Critical Injury of the Child 3. Parental Role Conflict Related to Child's Injuries and PICU Policies 5. Anger Related to Feelings of Helplessness All of these nursing diagnoses except Knowledge Deficit are possible in this situation. Although planning for discharge begins with admission, it is too early to begin teaching the parents about home care. The astute and experienced PICU nurse is prepared to recognize current problems and intervene appropriately.

There are many healthcare needs of children with chronic conditions. What nursing strategy would best help parents with continuity of care? 1. Include the family and older child in decision making. 2. Assist the family in gaining transportation to healthcare appointments. 3. Provide the family with resources such as social services. 4. Recognize and respect the cultural needs of the family.

1. Include the family and older child in decision making. Continuity of care involves the family and child's participation in their health care. Access to transportation involves access to care, not continuity. Providing resources such as social services is related to comprehensiveness of care, not to continuity. Recognizing and respecting cultural needs are part of the degree to which healthcare services, not continuity of care, are provided.

A 5-year-old sibling of a 9-year-old child with cystic fibrosis tells the nurse, "I wish I had a breathing disease, too." The nurse knows the parents strive to spend quality time with each child and with both children together. What is the sibling currently experiencing? 1. Jealousy 2. Isolation 3. Loneliness 4. Anger

1. Jealousy The child with cystic fibrosis has something the younger child does not have. Cystic fibrosis brings the affected child more attention from others. Even if parents strive to spend more time with siblings of ill children, the well-child will be jealous because the situation can never be equal. The 5-year-old child does not understand the complications of the disease and only sees the 9-year-old child treated differently. Siblings of ill children may experience loneliness, isolation, or anger; but the child's comment does not support these feelings.

A toddler-age client is in end-stage renal failure. Which nursing intervention will assist this child most? 1. Maintain the child's normal routines. 2. Explain body changes that will take place. 3. Encourage friends to visit. 4. Allow the child to talk about the illness.

1. Maintain the child's normal routines. A toddler has no real concept of death, but does sense changes in routine and parent behavior. Maintaining normal routines is the best intervention to assist this child. A toddler will not understand the body changes; this approach would be more appropriate for a school-age child. Encouraging friends to visit and allowing the child to talk about the illness are more appropriate for older children.

The nurse is providing care for an adolescent client who is experiencing pain related to a sickle cell crisis. Which medication does the nurse prepare to administer to this client? 1. Morphine sulfate 2. Meperidine 3. Acetaminophen 4. Ibuprofen

1. Morphine sulfate The pain during a sickling crisis is severe, and morphine is needed for pain control around the clock or by patient-controlled analgesia (PCA). Meperidine is not used for pain control for clients with sickle cell pain crisis because it could cause seizures. Acetaminophen or ibuprofen is used for mild pain and would not be effective for the severe pain experienced by a child in sickle cell pain crisis.

The nurse is planning activities for a toddler with a birth injury of a torn brachial plexus that resulted in muscle atrophy and weakness of his right arm. Which nursing intervention is most appropriate for this client? 1. Offering the toddler a choice of clothing 2. Asking the toddler if he would like to take his medicine 3. Dressing the toddler 4. Feeding the toddler

1. Offering the toddler a choice of clothing Toddlers are developing autonomy, self-control, and independence. Offering the toddler a choice contributes to their sense of autonomy. However, taking medicine is not within the toddler's realm of choice. Dressing and feeding the toddler does not encourage independence and will eventually cause frustration for both parent and toddler. The toddler must learn how to do these activities despite the physical limitations of the right arm.

The emergency-room nurse receives a preschool-age child who was hit by a car. Which nursing interventions are a priority for this child? Select all that apply. 1. Performing a rapid head-to-toe assessment 2. Recording the parents' insurance information 3. Assessing airway, breathing, and circulation 4. Asking the parents about organ donation 5. Asking the parents if anyone witnessed the accident

1. Performing a rapid head-to-toe assessment 3. Assessing airway, breathing, and circulation Assessing airway, breathing, and circulation and performing a rapid head-to-toe assessment are the priority nursing interventions. Asking the parents about organ donation is insensitive until the extent of the child's injuries is known. Recording insurance information is necessary but should never come before lifesaving assessment and intervention. Detailed information about the accident is helpful in determining the child's point of impact with the car and mechanism of injury, but this is not the initial priority.

An infant with tetralogy of Fallot is having a hypercyanotic episode ("tet" spell). Which nursing interventions are appropriate for the nurse to implement for this infant? Select all that apply. 1. Place the child in knee-chest position. 2. Draw blood for a serum hemoglobin. 3. Administer oxygen. 4. Administer morphine and propranolol intravenously as ordered. 5. Administer Benadryl as ordered.

1. Place the child in knee-chest position. 3. Administer oxygen. 4. Administer morphine and propranolol intravenously as ordered. When an infant with tetralogy of Fallot has a hypercyanotic episode, interventions should be geared toward decreasing the pulmonary vascular resistance. Therefore, the nurse would place the infant in knee-chest position (to decrease venous blood return from the lower extremities), and administer oxygen, morphine, and propranolol (to decrease the pulmonary vascular resistance). The nurse would not draw blood until the episode had subsided, because unpleasant procedures are postponed. Benadryl is not appropriate for this child.

The nurse is planning care for a preschool-age client who has cerebral palsy (CP). Which interventions are appropriate for this client? Select all that apply. 1. Providing heath supervision 2. Collaborating with physical therapy 3. Assisting with planning educational services 4. Prescribing medication for spasticity 5. Promoting growth and development

1. Providing heath supervision 2. Collaborating with physical therapy 3. Assisting with planning educational services 5. Promoting growth and development Appropriate interventions for the nurse who is providing care to a client with a chronic condition include providing health supervision, collaborating with other specialties, assisting with planning educational services, and promoting growth and development. It is outside the scope of nursing practice to prescribe medication. The nurse could, however, administer prescribed medications if appropriate.

The family has just been informed by the healthcare provider that their newborn is diagnosed with a congenital heart defect, Tetralogy of Fallot (TOF). The family tells the nurse that the healthcare provider told them that TOF is comprised of several defects, and they ask the nurse what the defects are. What will the nurse tell the family? Select all that apply. 1. Pulmonary stenosis 2. Coarctation of the aorta 3. Right ventricular hypertrophy 4. Ventral septal defect 5. Overriding aorta

1. Pulmonary stenosis 3. Right ventricular hypertrophy 4. Ventral septal defect 5. Overriding aorta Four defects are involved with TOF include: pulmonary stenosis, right ventricular hypertrophy, ventral septal defect, and overriding aorta.

A child who has undergone a hematopoietic stem cell transplantation (HSCT) is ready for discharge. Which items will the nurse include in the discharge teaching for this child and family? Select all that apply. 1. Recognize the signs of graft-versus-host disease. 2. Return the child to school within six weeks. 3. Practice good handwashing. 4. Avoid obtaining influenza vaccinations. 5. Avoid live plants and fresh vegetables.

1. Recognize the signs of graft-versus-host disease. 3. Practice good handwashing. 5. Avoid live plants and fresh vegetables. A child who is preparing for discharge after a HSCT will require specific interventions to decrease the risk of contracting communicable illnesses. Appropriate teaching points include: recognizing the signs of graft-versus-host disease; practicing good handwashing; and avoiding live plants and fresh vegetables. The child will require home schooling for 6 to 12 months. The child and family members should be encouraged to obtain yearly influenza vaccinations.

A nurse is planning care for a child with human immunodeficiency virus (HIV). Which nursing diagnosis is the highest priority for this child? 1. Risk for Infection 2. Risk for Fluid-Volume Deficit 3. Ineffective Thermoregulation 4. Ineffective Tissue Perfusion, Peripheral

1. Risk for Infection A child with HIV is at risk for a myriad of bacterial, viral, fungal, and opportunistic infections because of the effect of the virus on the immune system. Risk for Fluid-Volume Deficit, Ineffective Thermoregulation, and Ineffective Tissue Perfusion, Peripheral would not be priority problems with this disease process.

The nurse is providing care to a school-age client diagnosed with idiopathic thrombocytopenic purpura (ITP). Which nursing diagnosis is the priority for this client? 1. Risk for Injury 2. Ineffective Breathing Pattern 3. Nausea 4. Fluid-Volume Deficit.

1. Risk for Injury ITP is the most common bleeding disorder in children, so risk for injury (bleeding) is the priority nursing diagnosis. The disease process does not usually cause ineffective breathing patterns, nausea, or fluid-volume deficits.

A novice nurse in the newborn intensive care unit (NICU) has just performed postmortem care on a premature infant who passed away. The novice nurse asks to be excused near the end of the shift. Which interventions can be implemented to support this nurse? Select all that apply. 1. Schedule additional education on bereavement care 2. Ask a seasoned nurse to talk with the novice nurse 3. Tell the nurse it is OK to grieve with the family 4. Recommend that the nurse transfer to another unit 5. Assign the nurse to stable clients only

1. Schedule additional education on bereavement care 2. Ask a seasoned nurse to talk with the novice nurse 3. Tell the nurse it is OK to grieve with the family Appropriate interventions for this nurse include scheduling additional education on bereavement care, asking a seasoned nurse to talk about the situation with the novice nurse, and telling the nurse it is OK to grieve with the family. Recommending a transfer and assigning the nurse to only stable clients are not appropriate interventions to support the novice nurse.

The nurse is providing care to a toddler client who is diagnosed with osteogenesis imperfecta. Which nursing intervention is appropriate for this client? 1. Support of the trunk and extremities when moving 2. Traction care 3. Cast care 4. Postop spinal surgery care

1. Support of the trunk and extremities when moving With osteogenesis imperfecta, nursing care focuses on preventing fractures. Because the bones are fragile, the entire body must be supported when the child is moved. Traction, casts, and spinal surgery are not routinely done for osteogenesis.

The nurse is caring for the 5-year-old just diagnosed with von Willebrand disease after a tooth extraction with increased bleeding. The family asks the nurse how the signs and symptoms of von Willebrand disease are manifested. What will the nurse tell the family? Select all that apply. 1. Decreased partial thromboplastin time 2. Factor VI deficiency 3. Frequent nosebleeds 4. Bleeding from mucous membranes 5. Frequent bruising

3. Frequent nosebleeds 4. Bleeding from mucous membranes 5. Frequent bruising Characteristic manifestations are prolonged and excessive mucocutaneous bleeding, in children this is exhibited through gingival bleeding, epistaxis, menorrhagia, bruising, and minor wounds or lacerations.

A child undergoing chemotherapeutic treatment for cancer is being admitted to the hospital for fever of 102 degrees F and possible sepsis. Cultures, antibiotics, and acetaminophen (Tylenol) along with bed rest have been ordered for this child. Place the following steps in order from first to last. 1 Administer the antibiotics. 2 Administer the acetaminophen (Tylenol). 3 Obtain the cultures. 4 Ensure the child has bed rest.

2, 3, 1, 4 Give acetaminophen (Tylenol) first to decrease discomfort and reduce fever. Obtain the cultures next because management of infections is critical, and since a child on chemotherapy has lowered immune status, unusual agents can be identified. Cultures can help identify the causative agents before treatment is started. Give the antibiotics next, as an infection can seriously impact the child who is receiving chemotherapy. Finally, provide comfort followed by bed rest to allow the child to rest.

The nurse is providing care to a preschool-age client who is diagnosed with acquired immune deficiency syndrome (AIDS). In planning the client's care, which vaccine is inappropriate for the client to receive? 1. Diphtheria and tetanus toxoids and acellular pertussis vaccine (DTaP) 2. Haemophilus influenzae type B (HIB conjugate vaccine) 3. Varicella vaccine 4. Hepatitis B vaccine (Hep B)

3. Varicella vaccine A child with an immune disorder should not be immunized with a live varicella vaccine because of the risk of contracting the disease. DTaP, HIB, and hepatitis B vaccinations are not live vaccines and should be given on schedule.

The nurse is examining a 12-month-old who is brought to the clinic for persistent diaper rash. The nurse finds perianal inflammation with bright red scaly plaques and small papules. Satellite lesions are also present. What is the most likely cause of this client's diaper rash? 1. Impetigo (staph) 2. Candida albicans (yeast) 3. Urine and feces 4. Infrequent diapering

2. Candida albicans (yeast) Candida albicans is frequently the underlying cause of severe diaper rash. When a primary or secondary infection with Candida albicans occurs, the rash has bright red scaly plaques with sharp margins. Small papules and pustules may be seen, along with satellite lesions. Even though diaper dermatitis can be caused by impetigo, urine and feces, and infrequent diapering, the lesions and persistent characteristics are common for Candida.

A child is admitted to the neonatal intensive care unit (NICU). The parents are concerned because they cannot stay for long hours to visit. Which statement made by the nurse is most appropriate? 1. "One of you might take a leave of absence to be here more." 2. "Parents often feel this way; would you be interested in talking with others who have experienced having a child in the NICU?" 3. "Perhaps the grandparents can make the visits for you." 4. "Why can't you visit after work every day?"

2. "Parents often feel this way; would you be interested in talking with others who have experienced having a child in the NICU?" "Parents often feel this way; would you be interested in talking with others who have experienced having a child in the NICU?" is therapeutic; it focuses on feelings and offers support to the parents. The other options do not focus on how the parents feel and attempt to solve the issue rather than allow for the parents to deal with their feelings and form solutions.

A parent brings her school-age child to the clinic because the child has a temperature of 100.2°F. The child remains active without other symptoms. Which statement by the nurse to the parents is most appropriate? 1. "Take the child's temperature every 2 hours and call the clinic if it reaches 102°F or above." 2. "Unless the fever bothers the child, it is best to let the natural body defenses respond to the infection." 3. "Keep the child warm, because shivering often occurs with fever." 4. "Alternate acetaminophen and ibuprofen to help keep the fever down and keep the child comfortable."

2. "Unless the fever bothers the child, it is best to let the natural body defenses respond to the infection." Fever is the body's response to an infection, and is not a disease. Allowing the body's natural defenses (fever) to fight the infection is best. The fever is treated if the child is uncomfortable from effects of the fever, such as body aches, headache, and so on. Taking the child's temperature more than every 4 to 6 hours is unnecessary. The child should be dressed for comfort. Light clothing is recommended. Alternating acetaminophen and ibuprofen is not recommended.

A nurse is caring for four pediatric clients in the hospital. Which client should the nurse refer for play therapy? 1. An adolescent with asthma 2. A preschool-age child with a fractured femur 3. A school-age child having an appendectomy 4. An infant with sepsis

2. A preschool-age child with a fractured femur Play therapy is often used with preschool and school-age children who are experiencing anxiety, stress, and other specific nonpsychotic mental disorders. In this case, the child who experiences a condition that requires longer hospitalization and recovery, such as a fracture of the femur, should be referred for play therapy. The adolescent with asthma, the school-age child having an appendectomy, and the infant with sepsis do not have as high a need for play therapy as the preschool child with a broken bone.

A 24-hour urine collection for vanillylmandelic acid (VMA) has been ordered on a child suspected of having neuroblastoma. When is the most appropriate time for the nurse to begin the collection? 1. At 0700 2. After the next time the child voids 3. At bedtime 4. When the order is noted

2. After the next time the child voids A 24-hour urine collection is started after the child voids. That specimen is not saved, but all subsequent specimens in that 24-hour period should be collected. It would not be an accurate collection of 24 hours of urine if the collection began at 0700, bedtime, or when the order is noted.

A nurse is administering an intramuscular vaccination to an infant diagnosed with Wiskott-Aldrich syndrome (WAS). Which reaction is the infant more at risk for due to the diagnosis of WAS? 1. Pain at injection site 2. Bleeding at injection site 3. Redness and swelling at injection site 4. Mild rash at injection site

2. Bleeding at injection site Wiskott-Aldrich syndrome is characterized by thrombocytopenia, with bleeding tendencies appearing during the neonatal period. The syndrome would not put the child at higher risk for pain, redness, swelling, or rash at the injection site.

e nurse is teaching a prenatal class about infant care. Under which circumstances should the nurse emphasize that parents should call their healthcare provider immediately? Select all that apply. 1. Child 4 months old, received a DTaP immunization yesterday, and has a temperature of 38.0°C (100.4°F) 2. Child under 3 months old and has a temperature over 40.1°C (104.2°F) 3. Child difficult to awaken and has a pulsing fontanel 4. Child has purple spots on the skin and is lethargic. 5. Child has a stiff neck and has been irritable for three days.

2. Child under 3 months old and has a temperature over 40.1°C (104.2°F) 3. Child difficult to awaken and has a pulsing fontanel 4. Child has purple spots on the skin and is lethargic. 5. Child has a stiff neck and has been irritable for three days. Infants under 3 months of age have limited ability to develop antibodies to fight infection, and a fever as high as 40.1°C indicates a serious infection. Difficulty to awaken and a pulsing fontanel, purple spots on the skin and lethargy, a stiff neck and irritability for 3 days in infants and children of any age may indicate meningitis. A mild fever of 38.0°C (100.4°F) in the 4-month-old who received a DTaP immunization yesterday is incorrect because the mild fever is expected as the body develops antibodies in response to antigens in the immunization.

The child and family come to the clinic requesting information about causes of cardiac defects. The father has high incidence of cardiac defects in his family, and the child is frequently cyanotic around the lips. What causes should the nurse tell the family about? Select all that apply. 1. Decreased maternal age 2. Chromosomal abnormalities 3. Fetal exposure to maternal drugs 4. Maternal viral infections 5. Maternal metabolic disorders

2. Chromosomal abnormalities 3. Fetal exposure to maternal drugs 4. Maternal viral infections 5. Maternal metabolic disorders Cardiac defects may result from fetal exposure to maternal drugs, increased maternal age, chromosomal abnormalities, maternal viral infections, maternal metabolic disorders, and multifactorial genetic factors.

An infant returns from surgery for correction of bilateral congenital clubfeet. The infant has bilateral long-leg casts. The toes on both feet are edematous, but there is color, sensitivity, and movement to them. Which action by the nurse is the most appropriate? 1. Call the healthcare provider to report the edema. 2. Elevate the legs on pillows. 3. Apply a warm, moist pack to the feet. 4. Encourage movement of toes.

2. Elevate the legs on pillows. The legs should be elevated on a pillow for 24 hours to promote healing and help with venous return. Some amount of swelling can be expected, so it would not be appropriate to notify the healthcare provider, especially if the color, sensitivity, and movement remain normal to the toes. Ice should be applied, not heat. An infant would not be able to follow directions to move toes, and in this case it would not be as effective as elevating the legs on pillows.

In working with parents of children with chronic diseases, the nurse is concerned with helping the parents to protect themselves from compassion fatigue. Which activities are appropriate for the nurse to encourage? Select all that apply. 1. Sleeping more than 9 hours per 24-hour period 2. Exercising 3. Fostering social relationships 4. Developing a hobby 5. Moving away

2. Exercising 3. Fostering social relationships 4. Developing a hobby Exercising, fostering social relationships, and developing a hobby all contribute to physical, spiritual, social, and mental rest and restoration. Sleeping more than the body requires and moving away are avoidance behaviors that do not address exhaustion from overwhelming caregiving responsibilities.

A mother brings her 4-month-old infant in for a routine checkup and vaccinations. The mother reports that the infant was exposed to a brother who has the flu. Which action by the nurse is most appropriate based on these assessment findings? 1. Withhold the vaccinations. 2. Give the vaccinations as scheduled. 3. Withhold the DTaP vaccination but give the others as scheduled. 4. Give the infant the flu vaccination but withhold the others.

2. Give the vaccinations as scheduled. Recent exposure to an infectious disease is not a reason to defer a vaccine. There is no reason to withhold any of the vaccinations due at this time. The flu vaccination would not routinely be given to a 4-month-old.

The nurse is preparing to discharge an infant with a congenital heart defect. The infant will be cared for at home by the parents until surgery. Which items will the nurse include in the discharge teaching for this infant and family? Select all that apply. 1. Allow the infant to feed for 60 minutes. 2. Hold the infant at a 45-degree angle. 3. Encourage frequent hand hygiene. 4. Notify the health care provider for fever. 5. Pump the breasts and feed with a bottle if weight gain is an issue.

2. Hold the infant at a 45-degree angle. 3. Encourage frequent hand hygiene. 4. Notify the health care provider for fever. 5. Pump the breasts and feed with a bottle if weight gain is an issue. Children are often managed at home until surgery. The parents should hold the infant at a 45-degree angle to decrease tachypnea. The parents should also encourage frequent hand hygiene to decrease the risk of infection. It is important to notify the health care provider for a fever, as the infant will be at risk for dehydration and digoxin toxicity. If the mother is breastfeeding and the infant is losing weight, the mother should be encouraged to pump the milk and feed the infant from a bottle, but each feeding should be limited to 30 minutes. Tube feedings may be needed for this infant to conserve calories expenditure.

The nurse is providing care to an adolescent child who is at risk for developing adult-onset cardiovascular disease. Which teaching points will decrease the adolescent's risk? Select all that apply. 1. Encourage a decrease in smoking. 2. Limit fat intake to 20 to 35 percent of intake. 3. Encourage participation in vigorous exercise for at least 30 minutes. 4. Maintain a normal weight. 5. Include high-fat dairy products in the daily diet.

2. Limit fat intake to 20 to 35 percent of intake. 3. Encourage participation in vigorous exercise for at least 30 minutes. 4. Maintain a normal weight. Teaching points that will decrease the adolescent's risk of developing adult-onset cardiovascular disease include: limiting fat intake to 20 to 35 percent of total daily intake; encouraging the participation in vigorous exercise at least 30 minutes each day; and maintaining a normal weight. The adolescent and family members should be encouraged to stop smoking, not just to decrease smoking. The family should be educated to include low-fat dairy products in the daily diet.

A family actively participates in school functions. One of the children is paraplegic and requires a wheelchair for mobility. Which process does the nurse determine the family is working on based on these assessment findings? 1. Stagnation 2. Normalization 3. Isolation 4. Interaction

2. Normalization The family is normalizing life with the children through activities. The family is not staying at home because one member cannot walk; rather, the family is moving on to full participation in life. The family is interacting with others through the process of normalization.

The nurse is providing care to a school-age client with a documented immunodeficiency who is admitted to the general pediatric unit for intravenous medication administration. Which interventions are appropriate for this client? Select all that apply. 1. Institute droplet precautions. 2. Place in a positive-pressure room. 3. Avoid live vaccines. 4. Perform frequent handwashing. 5. Recommend fresh fruits brought in by the family.

2. Place in a positive-pressure room. 3. Avoid live vaccines. 4. Perform frequent handwashing. Pediatric clients with documented immunodeficiency require specific interventions to decrease their risk for developing infections while in the hospital environment. Appropriate interventions for this client include a positive-pressure room, avoiding live vaccines, and meticulous handwashing from staff and visitors. This client would require standard precautions, not droplet precautions. Because of the risk of infection with fresh fruit, the family would not be allowed to bring this to the client during their hospital stay.

The nurse is providing care for a pediatric client who has a third-degree circumferential burn of the right arm. Which nursing diagnosis is the priority for this client? 1. Risk for Infection 2. Risk for Altered Tissue Perfusion 3. Risk for Altered Nutrition: Less than Body Requirements 4. Impaired Physical Mobility

2. Risk for Altered Tissue Perfusion When the burn is circumferential, blood flow can become restricted due to edema and result in tissue hypoxia; therefore, the priority diagnosis is Risk for Altered Tissue Perfusion to the Extremity. Infection, Nutrition, and Mobility would have second priority in this case.

An adolescent client has a stiff neck, a headache, a fever of 103 degrees Fahrenheit, and purpuric lesions noted on the legs. Although the adolescent's physical needs take priority at the present time, the nurse can expect which to be the most significant psychological stressor for this adolescent? 1. Separation from parents and home 2. Separation from friends and permanent changes in appearance 3. Fear of painful procedures and bodily mutilation 4. Fear of getting behind in schoolwork

2. Separation from friends and permanent changes in appearance Adolescents are developing their identity and rely most on their friends. They are concerned about their appearance and how they look compared to their peers. Separation from parents and home is the main psychological stressor for infants and toddlers. Preschool-age children fear pain and bodily mutilation. School-age children are developing a sense of industry and fear getting behind in schoolwork.

A child with a brain tumor is admitted to the pediatric intensive care unit (PICU) after brain surgery to remove the tumor. Which postoperative order would the nurse question? 1. Antibiotics 2. Sodium levels every 24 hours 3. Anticonvulsants 4. Hourly intake and output

2. Sodium levels every 24 hours Antibiotics, anticonvulsants, and hourly intake and output are appropriate orders. Serum sodium levels should be done every 4 to 6 hours, not every 24 hours.

A nurse working in a pediatric clinic is responsible for monitoring and maintaining the vaccinations on site. Which actions are appropriate for this nurse to implement? Select all that apply. 1. Fluctuate refrigerator and freezer temperatures each day. 2. Store vaccines in the center of the unit. 3. Check and record the temperature of the unit twice each day. 4. Have a plan for power outages. 5. Place bottles of water in each unit to help keep temperatures consistent.

2. Store vaccines in the center of the unit. 3. Check and record the temperature of the unit twice each day. 4. Have a plan for power outages. 5. Place bottles of water in each unit to help keep temperatures consistent. Appropriate interventions for the nurse to implement in order to maintain the potency of vaccines include storing the vaccines in the center of the unit, checking and recording the temperature of the storage unit twice a day, having a plan for power outages, and placing bottles of water in each unit to help keep temperatures consistent. The temperature of the refrigerator and freezer should be consistent and not fluctuate.

The nurse is teaching the parents of a group of cardiac patients. Which teaching guideline will the nurse include for any child who has undergone cardiac surgery? 1. The child should be restricted from most play activities. 2. The child should be evaluated to determine if prophylactic antibiotics for dental, oral, or upper-respiratory-tract procedures are necessary. 3. The child should not receive routine immunizations. 4. The child can be expected to have a fever for several weeks following the surgery.

2. The child should be evaluated to determine if prophylactic antibiotics for dental, oral, or upper-respiratory-tract procedures are necessary. Parents should be taught that the child may need prophylactic antibiotics for some dental procedures, according to the American Heart Association, to prevent endocarditis. The child should live a normal and active life following repair of a cardiac defect. Immunizations should be provided according to the schedule, and any unexplained fever should be reported.

The nurse has set up a group discussion for several families with chronically ill children. The nurse informs these parents that they may face which ethical issue? 1. Normalization 2. Withholding and refusal of treatment 3. Repeated hospital admissions 4. Lack of proper dietary needs

2. Withholding and refusal of treatment Withholding and refusal of treatment is an ethical issue involving the life and quality of life of the child. Normalization is a family process of adaptation as the family members cope with daily life with their child. Lack of dietary needs is not an ethical issue, nor is repeated hospital admissions.

The nurse is caring for a pediatric client who sustained a severe burn. Determine the order of what would be done for this child when the medical team arrives on the scene: 1. Start intravenous fluids. 2. Provide for relief of pain. 3. Establish an airway. 4. Place a Foley catheter.

3, 1, 2, 4 The first step in burn care is to ensure that the child has an airway, is breathing, and has a pulse. Due to the severity of the burn, establishing IV access and starting resuscitation fluids would be next, followed by addressing the area of pain and inserting a Foley catheter

The nurse is teaching family members how to care for their infant in a Pavlik harness to treat congenital developmental dysplasia of the hip. Which statement will the nurse include in the teaching session? 1. "Apply lotion or powder to minimize skin irritation." 2. "Put clothing over the harness for maximum effectiveness of the device." 3. "Check at least 2 or 3 times a day for red areas under the straps." 4. "Place a diaper over the harness, preferably using a thin, superabsorbent, disposable diaper."

3. "Check at least 2 or 3 times a day for red areas under the straps." The brace should be checked 2 or 3 times for red areas under the straps. Lotion or powder can contribute to skin breakdown. A light layer of clothing should be under the brace, not over. The diaper should also be under the brace.

At the conclusion of teaching parents about cerebral palsy, the nurse asks, "What is your hope for your toddler with cerebral palsy?" Which reply from a parent best indicates an understanding of a realistic achievement for the child? 1. "I hope my child qualifies for the Winter Olympics like I did." 2. "I hope my child just enjoys life." 3. "I hope my child will attend our neighborhood school." 4. "I hope my child is liked and accepted by other children."

3. "I hope my child will attend our neighborhood school." Expecting a child with cerebral palsy to do well in the local school is a realistic hope that the child can possibly achieve. A child with cerebral palsy does not have the gross motor skills to qualify for the Olympics; thus, this is unrealistic. A hope for the child to enjoy life is realistic, but is not an achievement for the child. A hope that the child is liked and accepted by other children is realistic, but this hope is also dependent on other children.

The hospital has just provided its nurses with information about biologic threats and terrorism. After completing the course, a group of nurses is discussing its responsibility in relation to bioterrorism. Which statement by the nurse indicates a correct understanding of the concepts presented? 1. "It is important to separate clients according to age and illness to prevent the spread of disease." 2. "It is important to dispose blood-contaminated needles in the lead-lined container." 3. "I will notify the Centers for Disease Control (CDC) if a large number of persons with the same life-threatening infection present to the emergency room." 4. "I will initiate isolation precautions for a hospitalized client with methicillin-resistant staphylococcus aureus (MRSA)."

3. "I will notify the Centers for Disease Control (CDC) if a large number of persons with the same life-threatening infection present to the emergency room." The CDC must be contacted to investigate the source of serious infections and to determine if a bioterrorist threat exists. Separating clients according to age and illness to prevent the spread of disease will do nothing to stop terrorism. Proper disposal of blood-contaminated needles in the sharps container and initiating isolation precautions for a hospitalized client with methicillin-resistant staphylococcus aureus (MRSA) are appropriate nursing actions but do not relate to bioterrorism.

The nurse is discussing ways to treat fever in the home environment to a group of parents in the community. Which statement is appropriate for the nurse to include in the presentation? 1. "Ibuprofen is the only effective means to reduce fever." 2. "If the child requires more than one dose of acetaminophen antibiotics are needed." 3. "Purchase a new bottle of acetaminophen for your newborn because it will have recommended medication concentration." 4. "It is not necessary to follow the recommendations on the bottle of ibuprofen as this will not prevent an overdose for your child."

3. "Purchase a new bottle of acetaminophen for your newborn because it will have recommended medication concentration." The recommendation to purchase a new bottle of acetaminophen due to recommended medication concentrations is an appropriate statement for the nurse to include in the teaching session. The other statements are inaccurate or inappropriate for the nurse to include in the teaching session.

A nurse is providing information to a group of new mothers. Which statement best explains why newborns and young infants are more susceptible to infection? 1. "They have high levels of maternal antibodies to diseases to which the mother has been exposed." 2. "They have passive transplacental immunity from maternal immunoglobulin G." 3. "They have immune systems that are not fully mature at birth." 4. "They have been exposed to microorganisms during the birth process."

3. "They have immune systems that are not fully mature at birth." Newborns have a limited storage pool of neutrophils and plasma proteins to defend against infection. Newborns' and young infants' high levels of maternal antibodies, passive transplacental immunity, and exposure to microorganisms during the birth process are all true but are incorrect answers because they do not explain the susceptibility of newborns and young infants to infection.

Which client in the pediatric intensive care unit (PICU) would most benefit from palliative care? 1. A child with end-stage leukemia 2. A child with a broken arm after a motor vehicle accident 3. A child with burn injuries to the legs 4. A child with recurrent asthma

3. A child with burn injuries to the legs A child with burn injuries to the legs will benefit most from palliative care to help control pain, anxiety, sleep disturbances, and so on. The child with end-stage leukemia will benefit from hospice care. The child with a broken arm or recurrent asthma will not need palliative care.

The nurse is caring for a child who is in a sickle cell crisis and has severe pain. Which nursing intervention is the most appropriate for this child? 1. Giving comfort measures, such as back rubs 2. Suggesting diversional activities, such as coloring 3. Administering pain medication 4. Preparing the child for painful procedures

3. Administering pain medication Severe pain requires administration of pain medication for pain relief. Comfort measures and diversional activities are not effective against severe pain in children. Comfort measures should be given to every child and can be used after pain medication is given. A child in severe pain is not capable of participating in or enjoying diversional activities. Preparing the child for painful procedures is not appropriate when the child is already in pain.

A school-age child with hemophilia falls on the playground and goes to the nurse's office with superficial bleeding above the knee. Which action by the nurse is the most appropriate? 1. Apply a warm, moist pack to the area. 2. Perform some passive range of motion to the affected leg. 3. Apply pressure to the area for at least 15 minutes. 4. Keep the affected extremity in a dependent position.

3. Apply pressure to the area for at least 15 minutes. If a hemophiliac child experiences a bleeding episode, superficial bleeding should be controlled by applying pressure to the area for at least 15 minutes. Ice should be applied, not heat. The extremity should be immobilized and elevated, so passive range of motion and keeping the extremity in a dependent position would not be appropriate interventions at this time.

A school-age child with congenital heart block codes in the emergency department (ED). The parents witness this and stare at the resuscitation scene unfolding before them. Which nursing intervention is most appropriate in this situation? 1. Ask the parents to leave until the child has stabilized. 2. Ask the parents to call the family to come into watch the resuscitation. 3. Ask the parents to sit near the child's face and hold her hand. 4. Ask the parents to stand at the foot of the cart to watch.

3. Ask the parents to sit near the child's face and hold her hand. Parents should be helped to support their child through emergency procedures, if they are able. Parents should never be asked to take part in emergency efforts unless absolutely necessary. Merely watching the resuscitation serves no purpose for the child. If the parents interfere with resuscitation efforts or they are unable to tolerate the situation, they can be asked to leave later.

The antiemetic drug ondansetron (Zofran) is administered to a child receiving chemotherapy. When should the nurse administer this medication? 1. Only if the child experiences nausea 2. After the chemotherapy has been administered 3. Before chemotherapy administration as a prophylactic measure 4. Never; this antiemetic is not effective for controlling nausea and vomiting associated with chemotherapy

3. Before chemotherapy administration as a prophylactic measure The antiemetic ondansetron (Zofran) should be administered before chemotherapy as a prophylactic measure. Giving it after the child has nausea or at the end of chemotherapy treatment does not help with preventing nausea. It is the drug of choice for controlling nausea caused by chemotherapy agents.

During the recovery-management phase of burn treatment, which is the most common complication seen in children? 1. Shock 2. Metabolic acidosis 3. Burn-wound infection 4. Asphyxia

3. Burn-wound infection Infection of the burned area is a frequent complication in the recovery—management phase. A goal of burn-wound care is protection from infection.

The nurse is providing discharge teaching to a school-age client who was recently diagnosed with a latex allergy. Which product will the nurse educate the client and family to avoid? 1. Plastic bottles 2. Footballs 3. Chewing gum 4. Paper bags

3. Chewing gum When a child is diagnosed with a latex allergy, it is essential for the nurse to educate both the child and the family regarding sources of latex within the home and the community. The child and family should be educated to avoid chewing gum as it contains latex. The other items do not contain latex and do not pose a risk for this child in the community.

A toddler client with a fever is prescribed amoxicillin clavulanate 250 mg/5 cc three times daily by mouth × 10 days for otitis media. Which teaching point will guard against antibiotic resistance to the disease process? 1. Administer a loading dose for the first dose. 2. Measure the prescribed dose in a household teaspoon. 3. Give the antibiotic for the full 10 days. 4. Stop the antibiotic if the child is afebrile.

3. Give the antibiotic for the full 10 days. Antibiotics must be administered for the full number of days ordered to prevent mutation of resistant strains of bacteria. A loading dose was not ordered. A household teaspoon may contain less than 5 cc, and the full dose must be given. Stopping the antibiotic before the prescribed time will permit remaining bacteria to reproduce, and the otitis media will return, possibly with antibiotic-resistant organisms. The absence of a fever is not an indication that all bacteria are killed or not reproducing.

Which athletic activity can the nurse recommend for a school-age client with pulmonary-artery hypertension? 1. Cross-country running 2. Soccer 3. Golf 4. Basketball

3. Golf A child with pulmonary-artery hypertension should have exercise tailored to avoid dyspnea. Golf would require less exertion than soccer, basketball, or cross-country running.

A nurse notes blue sclera during a newborn assessment. Which item will the newborn require further assessment for based on this finding? 1. Marfan syndrome 2. Achondroplasia 3. Osteogenesis imperfecta 4. Muscular dystrophy

3. Osteogenesis imperfecta Clinical manifestations of osteogenesis imperfecta include blue sclera. This is not present in Marfan syndrome, achondroplasia, or muscular dystrophy.

The school nurse is trying to prevent the spread of a flu virus through the school. Which infection-control strategies can be employed to prevent the spread of the flu virus? Select all that apply. 1. Teaching parents safe food preparation and storage 2. Withholding immunizations for children with compromised immune systems 3. Sanitizing toys, telephones, and door knobs to kill pathogens 4. Separating children with infections from children who are well 5. Teaching children to wash their hands after using the bathroom

3. Sanitizing toys, telephones, and door knobs to kill pathogens 4. Separating children with infections from children who are well 5. Teaching children to wash their hands after using the bathroom To prevent the spread of communicable diseases, microorganisms must be killed or their growth controlled. Sanitizing toys and all contact surfaces, separating children with infections, and teaching children to wash their hands all control the growth and spread of microorganisms. Teaching parents safe food preparation and storage is another tool to prevent the spread of microorganisms but is not related to the flu virus. Immunizations should not be withheld from immunocompromised children; this is not an infection-control strategy.

The nurse can instruct parents to expect children in which age group to begin to assume more independent responsibility for their own management of a chronic condition, such as blood-glucose monitoring, insulin administration, intermittent self-catheterization, and appropriate inhaler use? 1. Toddlers 2. Preschool-age 3. School-age 4. Adolescents

3. School-age School-age children are developing a sense of industry and can begin assuming responsibility for self-care. Toddlers and preschool-age children do not have the cognitive and psychomotor skills for these tasks. Adolescents should already be well accomplished at self-care.

The nurse is teaching a 10-year-old and family about the diagnosis of Ewing sarcoma. The nurse knows that instruction has been successful when the child and family indicate which is a common site? 1. Bone marrow 2. Head 3. Shaft 4. Growth plate 5. Bursae

3. Shaft Ewing sarcoma is a malignant, tumor involving the diaphyseal (shaft) portion of the long bones. Common sites include: femur, pelvis, tibia, fibula, ribs, humerus, scapula, and clavicle.

The nurse is providing care to an adolescent client diagnosed with systemic lupus erythematosus (SLE). Which action by the client indicates acceptance of body changes associated with SLE? 1. She refuses to attend school. 2. She doesn't want to attend any social functions. 3. She discusses the body changes with a peer. 4. She discusses the body changes with healthcare personnel only.

3. She discusses the body changes with a peer. Peer interaction is important to the teen. Being able to discuss the changes to her body with a peer indicates acceptance of the change in body image. Discussing changes only with healthcare personnel does not indicate the teen has adjusted to body-image changes. Refusing to go to school or not going to social functions indicates nonacceptance of the changes to body image.

A school-age child diagnosed with rheumatoid arthritis asks the nurse to recommend an exercise activity. Which activity is most appropriate for this child? 1. Softball 2. Football 3. Swimming 4. Basketball

3. Swimming Swimming helps to exercise all of the extremities without putting undue stress on joints. Softball, football, and basketball could exacerbate joint discomfort.

A child has been admitted to the hospital unit in congestive heart failure (CHF). Which symptom would the nurse anticipate upon assessment of the child? 1. Weight loss 2. Bradycardia 3. Tachycardia 4. Increased blood pressure

3. Tachycardia Tachycardia is a sign of congestive heart failure because the heart attempts to improve cardiac output by beating faster. Bradycardia is a serious sign and can indicate impending cardiac arrest. Blood pressure does not increase in CHF, and the weight, instead of decreasing, increases because of retention of fluids.

A child returns from spinal-fusion surgery. Which item is the priority assessment for this child? 1. Increased intracranial pressure 2. Seizure activity 3. Impaired pupillary response during neurological checks 4. Impaired color, sensitivity, and movement to lower extremities

4. Impaired color, sensitivity, and movement to lower extremities When the spinal column is manipulated, there is a risk for impaired color, sensitivity, and movement to lower extremities. The other signs are neurological impairment and are not high risk with spinal surgery.

The charge nurse on a pediatric unit is making a room assignment for a school-age child diagnosed with sickle cell disease, who is in splenic sequestration crisis. Which room assignment is most appropriate for this client? 1. Semiprivate room 2. Reverse-isolation room 3. Contact-isolation room 4. Private room

4. Private room Splenic sequestration can be life-threatening, and there is profound anemia. The child does not need an isolation room but should not be placed in a room with any child who may have an infectious illness. The private room is appropriate for this child.

The nurse finds that an infant has stronger pulses in the upper extremities than in the lower extremities, and higher blood pressure readings in the arms than in the legs. Which assessment will the nurse perform next on this infant? 1. Pedal pulses 2. Pulse oximetry level 3. Hemoglobin and hematocrit values 4. Blood pressure of the four extremities

4. Blood pressure of the four extremities Coarctation of the aorta can present with stronger pulses in the upper extremities than in the lower extremities and higher blood pressure readings in the arms than in the legs because of obstruction of circulation to the lower extremities. Blood pressure values of the four limbs should be the next assessment data collected. Pedal pulses, pulse oximetry, and labs themselves will not provide the data needed.

The nurse is working with a group of parents who have children with chronic conditions. Which statement by a parent would indicate a risk for a caregiver burden that could become overwhelming? 1. "My mother moved in and helped us take our quadruplets home." 2. "Our health insurance sent us a rejection letter for my child's brand-name medication, and we must fill out forms to get the generic." 3. "I chose to quit my job to be home with my child, and my husband helps in the evening when he can." 4. "I have to care for my child day and night, which leaves little time for me."

4. "I have to care for my child day and night, which leaves little time for me." No respite time from caregiving responsibilities may lead to overwhelming caregiver burden. The family's pitching in to help indicates family support. Substituting generic for brand-name medications will not result in caregiver burden. The mother's choosing to care for the child and receiving help from the husband indicates family support.

Parents of a child who experienced a moderately severe allergic reaction after eating peanuts ask the nurse what they can do to help if it happens again. Which response by the nurse is the most appropriate? 1. "If it happens again, I will teach you what to do." 2. "You should have an antihistamine like Benadryl with you at all times." 3. "We can start a desensitization process to take the allergy away." 4. "I will teach you how to use an Epi-Pen."

4. "I will teach you how to use an Epi-Pen." An Epi-Pen is the appropriate treatment if this reaction occurs again. Benadryl is fine, but most likely is not strong enough in light of the serious reaction the child had. Desensitization is not the appropriate instruction at this time. Telling the parents that they will be taught if it happens again is brushing off the seriousness of the situation.

The nurse is providing education to the parents of a pediatric client who is diagnosed with tinea capitis (ringworm of the scalp). Which statement made the parents indicates an appropriate understanding of the teaching session? 1. "We will give the griseofulvin on an empty stomach." 2. "We're glad ringworm isn't transmitted from person to person." 3. "Once the lesion is gone, we can stop the griseofulvin." 4. "We will give the griseofulvin with milk or peanut butter."

4. "We will give the griseofulvin with milk or peanut butter." Parents are advised to give oral griseofulvin with fatty foods such as milk or peanut butter to enhance absorption. The medication must be used for the entire prescribed period even if the lesions are gone. All members of the family and household pets should be assessed for fungal lesions because person-to-person transmission is common.

The nurse must prepare parents to see their adolescent daughter in the pediatric intensive-care unit (PICU). The child arrived by life flight after experiencing multiple traumas in a car accident involving a suspected drunk driver. At this time, which statement by the nurse to the family is the most appropriate? 1. "Don't worry; everything will be okay. We will take excellent care of your child." 2. "You should press charges against the drunk driver." 3. "Your child's leg was crushed and may have to be amputated." 4. "Your child's condition is very critical; her face is swollen, and she may not look like herself."

4. "Your child's condition is very critical; her face is swollen, and she may not look like herself." The priority is to prepare the parents for the child's changed appearance. The nurse must not offer false reassurance nor project future stressful events. Truthful statements about the child's condition can be introduced after the parents have seen the child and grasped the situation. The nurse supports the family but remains nonjudgmental about accident details.

The nurse is checking peripheral perfusion to a child's extremity following a cardiac catheterization. Which assessment finding indicates adequate peripheral circulation to the affected extremity? 1. A capillary refill of greater than three seconds 2. A palpable dorsalis pedis pulse but a weak posterior tibial pulse 3. A decrease in sensation with a weakened dorsalis pedis pulse 4. A capillary refill of less than three seconds with palpable warmth

4. A capillary refill of less than three seconds with palpable warmth The nurse checks the extremity to determine adequacy of circulation following a cardiac catheterization. An extremity that is warm with capillary refill of less than three seconds has adequate circulation. Other indicators of adequate circulation include palpable pedal (dorsalis and posterior tibial) pulses, adequate sensation, and pinkness of skin color. If the capillary refill is over three seconds; if any of the pedal pulses are absent and/or weakened; or if the extremity is cool, cyanotic, or lacking sensation, circulation may not be adequate.

A school bus carrying children in grades K-12 crashed into a ravine. The critically injured children were transported by ambulance and admitted to the pediatric intensive-care unit (PICU). The nurse is concerned about calming the frightened children. Which nursing intervention is most appropriate to achieve the goal of calming the frightened children? 1. Tell the children that the physicians are competent. 2. Assure the children that the nurses are caring. 3. Explain that the PICU equipment is state of the art. 4. Call the children's parents to come into the PICU.

4. Call the children's parents to come into the PICU. A sense of physical and psychological security is best achieved by the presence of parents. Children at all developmental levels look first to their parents or whoever acts as their parents for safety and security. Healthcare providers, no matter how competent or caring, cannot substitute for parents. Children often neither recognize nor care about state-of-the-art equipment.

A mother refuses to have her child be immunized with measles, mumps, and rubella (MMR) vaccine because she believes that letting her infant get these diseases will help him fight off other diseases later in life. Which response by the nurse is most appropriate? 1. Honor her request because she is the parent. 2. Explain that antibodies can fight many diseases. 3. Tell her that not immunizing her infant may protect pregnant women. 4. Explain that if her child contracts measles, mumps, or rubella, there could be very serious and permanent complications from these diseases.

4. Explain that if her child contracts measles, mumps, or rubella, there could be very serious and permanent complications from these diseases. Explaining that if her child contracts measles, mumps, or rubella, he could have very serious and permanent complications from these diseases is correct because measles, mumps, and rubella all have potentially serious sequelae, such as encephalitis, brain damage, and deafness. Honoring her request is not correct because the nurse has a professional duty to explain that the mother's belief about immunizations is erroneous and may result in harm to her infant. Explaining that antibodies can fight many diseases is not correct because the body makes antibodies that are specific to antigens of each disease. Antibodies for one disease cannot fight another disease. Telling her that not immunizing her infant may protect pregnant women is not correct because immunizing the infant with MMR vaccine will help protect pregnant women from contracting rubella by decreasing the transmission. If a pregnant woman contracts rubella, her fetus can be severely damaged with congenital rubella syndrome.

The parents of a toddler-age child who sustained severe head trauma from falling out a second-story window are arguing in the pediatric intensive-care unit (PICU) and blaming each other for the child's accident. Which nursing diagnosis is most appropriate for this family? 1. Parental Role Conflict Related to Protecting the Child 2. Hopelessness Related to the Child's Deteriorating Condition 3. Anxiety Related to the Critical-Care-Unit Environment 4. Family Coping: Compromised, Related to the Child's Critical Injury

4. Family Coping: Compromised, Related to the Child's Critical Injury The parents are displaying ineffective coping behaviors as a family. Parental role conflict does not refer to the parents' argument in the PICU, but means a parent is conflicted or confused about some aspect of the parental role. Each parent may be experiencing hopelessness, frustration, and anxiety, but they are not coping well as a family unit.

A parent reports that her school-age child, who has had all recommended immunizations, had a mild fever one week ago and now has bright red cheeks and a lacy red maculopapular rash on the trunk and arms. Which disease process does the nurse suspect based on the parent's description? 1. Chicken pox (varicella) 2. German measles (rubella) 3. Roseola (exanthem subitum) 4. Fifth disease (erythema infectiosum)

4. Fifth disease (erythema infectiosum) Fifth disease manifests first with a flulike illness, followed by a red "slapped-cheek" sign. Then a lacy maculopapular erythematous rash spreads symmetrically from the trunk to the extremities, sparing the soles and palms. Varicella (chicken pox) and rubella (German measles) are unlikely if the child has had all recommended immunizations. The rash of varicella progresses from papules to vesicles to pustules. The rash of rubella is a pink maculopapular rash that begins on the face and progresses downward to the trunk and extremities. Roseola typically occurs in infants and begins abruptly with a high fever followed by a pale pink rash starting on the trunk and spreading to the face, neck, and extremities.

The nurse prepares the second diphtheria, tetanus toxoid, and acellular pertussis (DTaP) and second inactivated polio vaccine (IPV) immunization injections for an infant who is 4 months old. The nurse may also give which of immunizations during the same well-child-care appointment? 1. Var (varicella) 2. TIV (influenza) 3. MMR (measles, mumps, rubella) 4. Haemophilus influenza type B (HIB)

4. Haemophilus influenza type B (HIB) Haemophilus influenza type B (HIB) vaccine is given at 2, 4, 6, and 12 to 15 months of age (four doses). None of the other vaccines can be given to a 4-month-old infant. Influenza (TIV) vaccine may be given yearly to infants between 6 months and 3 years of age. Measles, mumps, and rubella (MMR) vaccine is given at 12 to 15 months and 4 to 6 years of age (two doses). Varicella (Var) is given at 12 to 18 months or any time up to 12 years for one dose; for 13 years and older two doses are given, 4 to 8 weeks apart.

Reducing the number of preventable childhood illnesses is a major national goal in Healthy People 2020. What will the school nurse teach families regarding immunizations in order to reach this goal? 1. A minor illness with a low-grade fever is a contraindication to receiving an immunization according to Healthy People 2020. 2. Vaccines should be given one at a time for optimum active immunity in the prevention of illness and disease. 3. Premature infants and low-birth-weight infants should receive half doses of vaccines for protection from communicable diseases. 4. It is important to maintain vaccination coverage for recommended vaccines in early childhood and to maintain them through kindergarten.

4. It is important to maintain vaccination coverage for recommended vaccines in early childhood and to maintain them through kindergarten. The benefits of vaccines far outweigh the risks from communicable diseases and resulting complications. A minor illness is not a contraindication to immunization. Giving vaccines one at a time will result in many missed opportunities. Half doses of vaccines should not be given routinely to premature and low-birth-weight infants.

A child who is diagnosed with leukemia has a sibling who is expressing feelings of anger and guilt. How would the nurse characterize this reaction by the sibling? 1. Abnormal; the sibling should be referred to a psychologist. 2. Normal; the illness doesn't affect the sibling. 3. Unexpected; the cancer is easily treated. 4. Normal; the sibling is affected too, and anger and guilt are expected feelings.

4. Normal; the sibling is affected too, and anger and guilt are expected feelings. A diagnosis of cancer affects the whole family, and initial feelings experienced by the sibling may be anger and guilt. Seldom will the sibling be unaffected; however, the response is not abnormal.

The hospital admitting nurse is taking a history of a child's illness from the parents. The nurse concludes that the parents treated their 6-year-old child appropriately for a fever related to otitis media. Which action by the parents brought the nurse to this conclusion? 1. Used aspirin every four hours to reduce the fever 2. Alternated acetaminophen with ibuprofen every two hours 3. Put the child in a tub of cold water to reduce the fever 4. Offered generous amounts of fluids frequently

4. Offered generous amounts of fluids frequently The body's need for fluids increases during a febrile illness. Aspirin has been associated with Reye syndrome and should not be given to children with a febrile illness. Alternating acetaminophen with ibuprofen every two hours may result in an overdose. Pediatric medication doses are more accurately calculated using the child's weight, not age. Putting the child in a tub of cold water will chill the child and cause shivering, a response that will increase body temperature.

An adolescent with cystic fibrosis is intubated with an endotracheal tube. Which nursing diagnosis is most appropriate for this adolescent? 1. Potential for Imbalanced Nutrition, More Than Body Requirements Related to Inactivity 2. Anxiety Related to Leaving Chores Undone at Home 3. Potential for Fear of Future Pain Related to Medical Procedures 4. Powerlessness (Moderate) Related to Inability to Speak to or Communicate with Friends

4. Powerlessness (Moderate) Related to Inability to Speak to or Communicate with Friends The adolescent values communication with peers and may feel frustrated that he cannot speak to them while intubated. The adolescent is present-oriented and is unlikely to worry about household chores or future unknown procedures. The adolescent with cystic fibrosis is likely to be underweight and is unlikely to take in more calories than needed while intubated.

The clinic nurse is working with a child with multiple disabilities. The parents have asked the nurse to help them in meeting with the school board to develop an Individualized Education Plan (IEP) and an Individualized Health Plan (IHP). Which nursing intervention is most appropriate? 1. Providing a written list of the child's medical diagnoses for the IEP meeting. 2. Offering to wait with the child while the parents attend the IEP meeting. 3. Listening to the parents' concerns and complaints about the school district. 4. Presenting verbally the child's cognitive, physical, and social skills to school officials at the IEP meeting.

4. Presenting verbally the child's cognitive, physical, and social skills to school officials at the IEP meeting. As an advocate for the child and a partner with the family, the nurse attends the IEP meeting and presents the child's functional skills to develop a comprehensive IEP. A list of medical diagnoses does not accurately inform school officials about the child's skills or needs. Waiting with the child and listening to parents' concerns may be kind and empathetic but does not contribute to an action plan for the child's educational needs.

A child with meningococcemia is being admitted to the pediatric intensive-care unit. Which room assignment is the most appropriate for this child? 1. Semiprivate room 2. Private room, but not in isolation 3. Private room, in protective isolation 4. Private room, in respiratory isolation

4. Private room, in respiratory isolation Meningococcemia follows an infection with Neisseria meningitidis. N. meningitidis is transmitted through airborne droplets; thus, the child should be placed in a private room in respiratory isolation. A private room with protective isolation (child is essentially kept in a "bubble") would not be appropriate.

The nurse is monitoring the urine specific gravity and pH on a child receiving chemotherapy. Which urinalysis result is the goal for this child? 1. Spec gravity 1.030; pH 6 2. Spec gravity 1.030; pH 7.5 3. Spec gravity 1.005; pH 6 4. Spec gravity 1.005; pH 7.5

4. Spec gravity 1.005; pH 7.5 Because the breakdown of malignant cells releases intracellular components into the blood and electrolyte imbalance causes metabolic acidosis, the urine specific gravity should remain at less than 1.010 and the pH at 7 to 7.5. A specific gravity higher than 1.010 can mean fluid intake is not high enough, and a pH of less than 7 means acidosis.

A young child diagnosed with leukemia is experiencing anorexia and severe stomatitis. The nurse should suggest that the parents try which intervention? a. Relax any eating pressures b. Firmly insist that child eat normally c. Begin gavage feedings to supplement diet d. Serve foods that are either hot or cold

ANS: A A multifaceted approach is necessary for children with severe stomatitis and anorexia. First, the parents should relax eating pressures. The nurse should suggest that the parents try soft, bland foods; normal saline or bicarbonate mouthwashes; and local anesthetics. The stomatitis is a temporary condition. The child can resume good food habits as soon as the condition resolves.

Which condition can result from the bone demineralization associated with immobility? a. Osteoporosis b. Urinary retention c. Pooling of blood d. Susceptibility to infection

ANS: A Bone demineralization leads to a negative calcium balance, osteoporosis, pathologic fractures, extraosseous bone formation, and renal calculi. Urinary retention is secondary to the effect of immobilization on the urinary tract. Pooling of blood is a result of the cardiovascular effects of immobilization. Susceptibility to infection can result from the effects of immobilization on the respiratory and renal systems.

The nurse, providing support to parents of a child newly diagnosed with a chronic disability, notices that they keep asking the same questions. How should the nurse respond to best meet their needs? a. Patiently continue to answer questions. b. Kindly refer them to someone else to answer their questions. c. Recognize that some parents cannot understand explanations. d. Suggest that they ask their questions when they are not upset.

ANS: A Diagnosis is one of the anticipated stress points for parents. The parents may not hear or remember all that is said to them. The nurse should continue to provide the kind of information that they desire. This is a particularly stressful time for the parents; the nurse can play a key role in providing necessary information. Parents should be provided with oral and written information. The nurse needs to work with the family to ensure understanding of the information. The parents require information at the time of diagnosis. Other questions will arise as they adjust to the information.

An adolescent with osteosarcoma is scheduled for a leg amputation in 2 days. The nurse's approach should include which action? a. Answering questions with straightforward honesty b. Avoiding discussing the seriousness of the condition c. Explaining that, although the amputation is difficult, it will cure the cancer d. Assisting the adolescent in accepting the amputation as better than a long course of chemotherapy

ANS: A Honesty is essential to gain the child's cooperation and trust. The diagnosis of cancer should not be disguised with falsehoods. The adolescent should be prepared for the surgery so he or she has time to reflect on the diagnosis and subsequent treatment. This allows questions to be answered. To accept the need for radical surgery, the child must be aware of the lack of alternatives for treatment. Amputation is necessary, but it will not guarantee a cure. Chemotherapy is an integral part of the therapy with surgery. The child should be informed of the need for chemotherapy and its side effects before surgery.

An adolescent is scheduled for a leg amputation in 2 days for treatment of osteosarcoma. The nurse's approach should include what intervention? a. Answering questions with straightforward honesty. b. Avoiding discussing the seriousness of the condition. c. Explaining that, although the amputation is difficult, it will cure the cancer. d. Assisting the adolescent in accepting the amputation as better than a long course of chemotherapy.

ANS: A Honesty is essential to gain the cooperation and trust of the child. The diagnosis of cancer should not be disguised with falsehoods. The adolescent should be prepared in advance for the surgery so that there is time for reflection about the diagnosis and subsequent treatment. This allows questions to be answered. To accept the need for radical surgery, the child must be aware of the lack of alternatives for treatment. Amputation is necessary, but it will not guarantee a cure. Chemotherapy is an integral part of the therapy with surgery. The child should be informed of the need for chemotherapy and its side effects before surgery.

The nurse is administering an IV chemotherapeutic agent to a child diagnosed with leukemia. The child suddenly begins to wheeze and have severe urticaria. Which is the most appropriate nursing action? a. Stop drug infusion immediately. b. Recheck rate of drug infusion. c. Observe child closely for next 10 minutes. d. Explain to child that this is an expected side effect.

ANS: A If an allergic reaction is suspected, the drug should be immediately discontinued. Any drug in the line should be withdrawn, and a normal saline infusion begun to keep the line open. Rechecking the rate of drug infusion, observing the child closely for next 10 minutes, and explaining to the child that this is an expected side effect can all be done after the drug infusion is stopped and the child is evaluated.

A child with growth hormone (GH) deficiency is receiving GH therapy. What is the best time for the GH to be administered? a. At bedtime b. After meals c. Before meals d. On arising in the morning

ANS: A Injections are best given at bedtime to more closely approximate the physiologic release of GH. Before or after meals and on arising in the morning are times that do not mimic the physiologic release of the hormone.

A common parental reaction to a child with special needs is parental overprotection. Parental behavior suggestive of this includes which behavior? a. Attempting to avoid frustrating situations. b. Providing consistent, strict discipline. c. Forcing child to help self, even when not capable. d. Encouraging social and educational activities not appropriate to child's level of capability.

ANS: A Parental overprotection is manifested by the parents' fear of letting the child achieve any new skill, avoiding all discipline, and catering to the child's every desire to prevent frustration. The overprotective parents usually do not set limits and or institute discipline, and they usually prefer to remain in the role of total caregiver. They do not allow the child to perform self-care or encourage the child to try new activities.

Four year old, placed in Buck's extension traction for Legg-Calvé-Perthes disease, is crying with pain as the nurse assesses that the skin of the right foot is pale with an absence of pulse. What should the nurse do first? a. Notify the practitioner of the changes noted. b. Give the child medication to relieve the pain. c. Reposition the child and notify the physician. d. Chart the observations and check the extremity again in 15 minutes.

ANS: A The absence of a pulse and change in color of the foot must be reported immediately for evaluation by the practitioner. Pain medication and repositioning should be addressed after the practitioner is notified. This is an emergency condition; immediate reporting is indicated. The findings should be documented with ongoing assessment.

Which is the most descriptive of a school-age child's reaction to death? a. Is very interested in funerals and burials b. Has little understanding of words such as forever c. Imagines the deceased person to be still alive d. Has an idealistic view of the world and criticizes funerals as barbaric

ANS: A The school-age child is very interested in postdeath services and may be inquisitive about what happens to the body. School-age children have an established concept of forever and have a deeper understanding of death in a concrete manner. Toddler may imagine the deceased person to be still alive. Adolescents may respond to death with an idealistic view of the world and criticize funerals as barbaric.

Which is the most appropriate nursing intervention to promote normalization in a school-age child with a chronic illness? a. Give child as much control as possible. b. Ask child's peer to make child feel normal. c. Convince child that nothing is wrong with him or her. d. Explain to parents that family rules for the child do not need to be the same as for healthy siblings.

ANS: A The school-age child who is ill may be forced into a period of dependency. To foster normalcy, the child should be given as much control as possible. It is unrealistic to expect one individual to make the child feel normal. The child has a chronic illness. It would be unacceptable to convince the child that nothing is wrong. The family rules should be similar for each of the children in a family. Resentment and hostility can arise if different standards are applied to each child.

Discharge planning for the child diagnosed with juvenile arthritis includes the need for which intervention? a. Routine ophthalmologic examinations to assess for visual problems. b. A low-calorie diet to decrease or control weight in the less mobile child. c. Avoiding the use of aspirin to decrease gastric irritation. d. Immobilizing the painful joints, which are the results of the inflammatory process.

ANS: A The systemic effects of juvenile arthritis can result in visual problems, making routine eye examinations important. Children with juvenile arthritis do not have problems with increased weight and often are anorexic and in need of high-calorie diets. Children with arthritis are often treated with aspirin. Children with arthritis are able to immobilize their own joints. Range-of-motion exercises are important for maintaining joint flexibility and preventing restricted movement in the affected joints.

The nurse is talking with the parents of a child who died 6 months ago. They sometimes still "hear" the child's voice and have trouble sleeping. They describe feeling "empty" and depressed. The nurse should recognize that: a. these are normal grief responses. b. the pain of the loss is usually less by this time. c. these grief responses are more typical of the early stages of grief. d. this grieving is essential until the pain is gone and the child is gradually forgotten.

ANS: A These are normal grief responses. The process of grief work is lengthy and resolution of grief may take years, with intensification during the early years. The child will never be forgotten by the parents.

The nurse is implementing care for a school-age child admitted to the pediatric intensive care experiencing symptomology associated with diabetic ketoacidosis (DKA). Which prescribed intervention should the nurse implement first? a. Begin 0.9% saline solution intravenously as prescribed. b. Administer regular insulin intravenously as prescribed. c. Place child on a cardiac monitor. d. Place child on a pulse oximetry monitor.

ANS: A All patients with DKA experience dehydration (10% of total body weight in severe ketoacidosis) because of the osmotic diuresis, accompanied by depletion of electrolytes (sodium, potassium, chloride, phosphate, and magnesium). The initial hydrating solution is 0.9% saline solution. Insulin therapy should be started after the initial rehydration bolus because serum glucose levels fall rapidly after volume expansion. The child should be placed on the cardiac and pulse oximetry monitors after the rehydrating solution has been initiated.

A child is upset because, when the leg cast is removed, the skin surface is caked with desquamated skin and sebaceous secretions. What should the nurse suggest to remove this material? a. Soak in a bathtub b. Vigorously scrub the leg c. Apply powder to absorb material d. Carefully pick material off of the leg

ANS: A Simple soaking in the bathtub is usually sufficient for the removal of the desquamated skin and sebaceous secretions. It may take several days to eliminate the accumulation completely. The parents and child should be advised not to scrub the leg vigorously or forcibly remove this material because it may cause excoriation and bleeding. Oil or lotion, but not powder, may provide comfort for the child.

The nurse is caring for an infant with developmental dysplasia of the hips (DDH). Which clinical manifestations should the nurse expect to observe? (Select all that apply.) a. Positive Ortolani sign b. Unequal gluteal folds c. Negative Babinski's sign d. Trendelenburg's sign e. Telescoping of the affected limb f. Lordosis

ANS: A, B A positive Ortolani sign and unequal gluteal folds are clinical manifestations of developmental dysplasia of the hips (DDH) seen from birth to 2 to 3 months. Trendelenburg's sign is noted in a child capable of standing alone. Negative Babinski's sign, telescoping of the affected limb, and lordosis are not clinical manifestations of developmental dysplasia of the hips (DDH).

A school-age child is diagnosed with systemic lupus erythematosus (SLE). The nurse should plan to implement which interventions for this child? (Select all that apply.) a. Instructions to avoid exposure to sunlight b. Teaching about body changes associated with SLE c. Preparation for home schooling d. Restricted activity

ANS: A, B Key issues for a child with SLE include therapy compliance; body-image problems associated with rash, hair loss, and steroid therapy; school attendance; vocational activities; social relationships; sexual activity; and pregnancy. Specific instru ctions for avoiding exposure to the sun and ultraviolet B light, such as using sunscreens, wearing sun-resistant clothing, and altering outdoor activities, must be provided with great sensitivity to ensure compliance while minimizing the associated feeling of being different from peers. The child should continue school attendance in order to gain interaction with peers and activity should not be restricted, but promoted.

The treatment of brain tumors in children consists of which therapies? (Select all that apply.) a. Surgery b. Bone marrow transplantation c. Chemotherapy d. Stem cell transplantation e. Radiation f. Myelography

ANS: A, C, E Treatment for brain tumors in children may consist of surgery, chemotherapy, and radiotherapy alone or in combination. Bone marrow and stem cell transplantation therapies are used for leukemia, lymphoma, and other solid tumors where myeloablative therapies are used. Myelography is a radiographic examination after an intrathecal injection of contrast medium. It is not a treatment.

Which are appropriate statements the nurse should make to parents after the death of their child? (Select all that apply.) a. "We feel so sorry that we couldn't save your child." b. "Your child isn't suffering anymore." c. "I know how you feel." d. "You're feeling all the pain of losing a child." e. "You are still young enough to have another baby."

ANS: A, D By saying, "We feel so sorry that we couldn't save your child," the nurse is expressing personal feeling of loss or frustration, which is therapeutic. Stating, "You're feeling all the pain of losing a child," focuses on a feeling, which is therapeutic. The statement, "Your child isn't suffering anymore," is a judgmental statement, which is nontherapeutic. "I know how you feel" and "You're still young enough to have another baby" are statements that give artificial consolation and are nontherapeutic.

Nursing care of a child diagnosed with syndrome of inappropriate antidiuretic hormone (SIADH) should include which interventions? (Select all that apply.) a. Weigh daily b. Encourage fluids c. Turn frequently d. Maintain nothing by mouth e. Restrict fluids

ANS: A, E Increased secretion of ADH causes the kidney to resorb water, which increases fluid volume and decreases serum osmolarity with a progressive reduction in sodium concentration. The immediate management of the child is to restrict fluids. The child should also be weighed at the same time each day. Encouraging fluids, turning frequently, and maintaining nothing by mouth are not associated with SIADH since they are not associated with managing/monitoring for fluid retention.

What should a nurse advise the parents of a child with type 1 diabetes mellitus who is not eating as a result of a minor illness? a. Give the child half his regular morning dose of insulin. b. Substitute simple carbohydrates or calorie-containing liquids for solid foods. c. Give the child plenty of unsweetened, clear liquids to prevent dehydration. d. Take the child directly to the emergency department.

ANS: B A sick-day diet of simple carbohydrates or calorie-containing liquids will maintain normal serum glucose levels and decrease the risk of hypoglycemia. The child should receive his regular dose of insulin even if he does not have an appetite. If the child is not eating as usual, he needs calories to prevent hypoglycemia. During periods of minor illness, the child with type 1 diabetes mellitus can be managed safely at home.

Chronic adrenocortical insufficiency is also referred to as what? a. Graves' disease b. Addison's disease c. Cushing's syndrome d. Hashimoto's disease

ANS: B Addison's disease is chronic adrenocortical insufficiency. Graves' and Hashimoto's diseases involve the thyroid gland. Cushing's syndrome is a result of excessive circulation of free cortisol.

A youngster will receive a bone marrow transplant (BMT) made possible because an older siblings is a histocompatible donor. Which is this type of BMT called? a. Syngeneic b. Allogeneic c. Monoclonal d. Autologous

ANS: B Allogeneic transplants are from another individual. Because he and his sibling are histocompatible, the BMT can be done. Syngeneic marrow is from an identical twin. There is no such thing as a monoclonal BMT. Autologous refers to the individual's own marrow.

Which behavior is considered an approach behavior in parents of chronically ill children? a. Inability to adjust to a progression of the disease or condition. b. Anticipation of future problems and seeking guidance and answers. c. Looking for new cures without a perspective toward possible benefit. d. Failing to recognize seriousness of child's condition despite physical evidence.

ANS: B Approach behaviors are coping mechanisms that result in a family's movement toward adjustment and resolution of the crisis of having a child with a chronic illness or disability. The parents who anticipate future problems and seek guidance and answers are demonstrating approach behaviors. They are demonstrating positive actions in caring for their child. Avoidance behaviors include being unable to adjust to a progression of the disease or condition, looking for new cures without a perspective toward possible benefit, and failing to recognize the seriousness of the child's condition despite physical evidence. These behaviors would suggest that the parents are moving away from adjustment or adaptation in the crisis of a child with chronic illness or disability.

An 8 year old will soon be able to return to school after an injury that resulted in several severe, chronic disabilities. What is the most appropriate action by the school nurse to help assure a smooth transition back to school? a. Recommending that the child's parents attend school at first to prevent teasing b. Preparing the child's classmates and teachers for changes they can expect c. Referring the child to a school where the children have chronic disabilities similar to hers d. Discussing with both the child and the parents the fact that classmates will not likely be as accepting as before

ANS: B Attendance at school is an important part of normalization for the child. The school nurse should prepare teachers and classmates about her condition, abilities, and special needs. A visit by the parents can be helpful, but unless the classmates are prepared for the changes, it alone will not prevent teasing. The child's school experience should be normalized as much as possible. Children need the opportunity to interact with healthy peers and engage in activities with groups or clubs composed of similarly affected persons. Children with special needs are encouraged to maintain and reestablish relationships with peers and participate according to their capabilities.

A 10 year old sustained a fracture in the epiphyseal plate of the right fibula when falling from a tree. When discussing this injury with the child's parents, the nurse should consider which statement? a. Healing is usually delayed in this type of fracture. b. Growth can be affected by this type of fracture. c. This is an unusual fracture site in young children. d. This type of fracture is inconsistent with a fall.

ANS: B Detection of epiphyseal injuries is sometimes difficult, but fractures involving the epiphysis or epiphyseal plate present special problems in determining whether bone growth will be affected. Healing of epiphyseal injuries is usually prompt. The epiphysis is the weakest point of the long bones. This is a frequent site of damage during trauma.

Which intervention will encourage a sense of autonomy in a toddler with disabilities? a. Avoiding separation from family during hospitalization b. Encouraging age appropriate independence in as many areas as possible c. Exposing child to pleasurable experiences as much as possible d. Helping parents learn special care needs of their child

ANS: B Encouraging the toddler to be independent encourages a sense of autonomy. The child can be given choices about feeding, dressing, and diversional activities, which will provide a sense of control. Avoiding separation from family during hospitalization and helping parents learn special care needs of their child should be practiced as part of family-centered care. They do not particularly foster autonomy. Exposing the child to pleasurable experiences, especially sensory ones, is a supportive intervention. It does not particularly support autonomy.

An adolescent is being seen in the clinic for evaluation of acromegaly. The nurse understands that which event occurs with acromegaly? a. There is a lack of growth hormone (GH) being produced. b. There is excess GH after closure of the epiphyseal plates. c. There is an excess of GH before the closure of the epiphyseal plates. d. There is a lack of thyroid hormone being produced.

ANS: B Excess GH after closure of the epiphyseal plates results in acromegaly. A lack of growth hormone results in delayed growth or even dwarfism. Gigantism occurs when there is hypersecretion of GH before the closure of the epiphyseal plates. Cretinism is associated with hypothyroidism.

Exophthalmos may occur in children with what diagnosis? a. Hypothyroidism b. Hyperthyroidism c. Hypoparathyroidism d. Hyperparathyroidism

ANS: B Exophthalmos (protruding eyeballs) is a clinical manifestation of hyperthyroidism. Hypothyroidism, hypoparathyroidism, and hyperparathyroidism are not associated with exophthalmos.

The nurse is discussing various sites used for insulin injections with a child and her family. Which site usually has the fastest rate of absorption? a. Arm b. Leg c. Buttock d. Abdomen

ANS: D The abdomen has the fastest rate of absorption but the shortest duration. The arm has a fast rate of absorption but short duration. The leg has a slow rate of absorption but a long duration. The buttock has the slowest rate of absorption and the longest duration.

Families progress through various stages of reactions when a child is diagnosed with a chronic illness or disability. After the shock phase, a period of adjustment usually follows that may be characterized by what reaction? a. Anger b. Overprotectiveness c. Social reintegration d. Guilt

ANS: B For most families, the adjustment phase is accompanied by several responses that are normally part of the adjustment process. Overprotectiveness, rejection, denial, or gradual acceptance are common reactions. The initial diagnosis of a chronic illness or disability often is often met with intense emotion and characterized by guilt and anger. Social reintegration is the culmination of the adjustment process.

The nurse is caring for a 4-year-old child immobilized by a fractured hip. Which complication should the nurse monitor for? a. Hypocalcemia b. Decreased metabolic rate c. Positive nitrogen balance d. Increased production of stress hormones

ANS: B Immobilization causes a decreased metabolic rate with slowing of all systems and a decreased food intake, leads to hypercalcemia, and causes a negative nitrogen balance secondary to muscle atrophy. A decreased production of stress hormones occurs with decreased physical and emotional coping capacity.

Which represents a common best practice in the provision of services to children with chronic or complex conditions? a. Care is focused on the child's chronologic age. b. Children with complex conditions are integrated into regular classrooms. c. Disabled children are less likely to be cared for by their families. d. Children with complex conditions are placed in residential treatment facilities.

ANS: B Normalization refers to behaviors and interventions for people with disabilities to integrate into society by living life as people without a disability would. For children, normalization includes attending school and being integrated into regular classrooms. This affords the child the advantages of learning with a wide group of peers. Care is necessarily focused on the child's developmental age. Home care by the family is considered best practice. The nurse can assist families by assessing social support systems, coping strategies, family cohesiveness, and family and community resources.

A nurse is planning palliative care for a child with severe pain. Which should the nurse expect to be prescribed for pain relief? a. Opioids as needed b. Opioids on a regular schedule c. Distraction and relaxation techniques d. Nonsteroidal antiinflammatory drugs

ANS: B Pain medications for children in palliative care should be given on a regular schedule, and extra doses for breakthrough pain should be available to maintain comfort. Opioid drugs such as morphine should be given for severe pain, and the dose should be increased as necessary to maintain optimal pain relief. Techniques such as distraction, relaxation techniques, and guided imagery should be combined with drug therapy to provide the child and family strategies to control pain. Nonsteroidal antiinflammatory drugs are not sufficient to manage severe pain for children in palliative care.

The nurse comes into the room of a child who was just diagnosed with a chronic disability. The child's parents begin to yell at the nurse about a variety of concerns. What is the nurse's best response? a. "What is really wrong?" b. "Being angry is only natural." c. "Yelling at me will not change things." d. "I will come back when you settle down."

ANS: B Parental anger after the diagnosis of a child with a chronic disability is a common response. One of the most common targets for parental anger is members of the staff. The nurse should recognize the common response of anger to the diagnosis and allow the family to express their feelings and emotions. "What is really wrong?" "Yelling at me will not change things," and "I will come back when you settle down" are all possible responses, but they are not addressing the parent's need to express their anger effectively.

Lindsey, age 5 years with a diagnosis of cerebral palsy, will be starting kindergarten next month and will be placed in a special education classroom. The parents are tearful when telling the nurse about this and state that they did not realize that their child's disability was so severe. How should the nurse interpret this parental response? a. This is a sign that parents are in denial. b. This is a normal anticipated time of parental stress. c. The parents need to learn more about cerebral palsy. d. The parents are used to having expectations that are too high.

ANS: B Parenting a child with a chronic illness can be very stressful for parents. There are anticipated times that parental stress increases. One of these identified times is when the child begins school. Nurses can help parents recognize and plan interventions to work through these stressful periods. The parents are not in denial; they are responding to the child's placement in school. The parents are not exhibiting signs of a knowledge deficit or expectations that are too high; this is their first interaction with the school system with this child.

Which interaction is part of the discharge plan for a school-age child with osteomyelitis who is receiving home antibiotic therapy? a. Instructions for a low-calorie diet b. Arrangements for tutoring and schoolwork c. Instructions for a high-fat, low-protein diet d. Instructions for the parent to return the child to team sports immediately

ANS: B Promoting optimal growth and development in the school-age child is important. It is important to continue schoolwork and arrange for tutoring if indicated. The child with osteomyelitis should be on a high-calorie, high-protein diet. The child with osteomyelitis may need time for the bone to heal before returning to full activities.

The nurse is caring for an 11-year-old boy who has recently been diagnosed with diabetes. What should be included in the teaching plan for daily injections? a. The parents do not need to learn the procedure. b. He is old enough to give most of his own injections. c. Self-injections will be possible when he is closer to adolescence. d. He can learn about self-injections when he is able to reach all injection sites.

ANS: B School-age children are able to give their own injections. Parents should participate in learning and giving the insulin injections. He is already old enough to administer his own insulin. The child is able to use thighs, abdomen, part of the hip, and arm. Assistance can be obtained if other sites are used.

Which laboratory finding confirms that a child with type 1 diabetes is experiencing diabetic ketoacidosis? a. No urinary ketones b. Low arterial pH c. Elevated serum carbon dioxide d. Elevated serum phosphorus

ANS: B Severe insulin deficiency produces metabolic acidosis, which is indicated by a low arterial pH. Urinary ketones, often in large amounts, are present when a child is in diabetic ketoacidosis. Serum carbon dioxide is decreased in diabetic ketoacidosis. Serum phosphorus is decreased in diabetic ketoacidosis.

A 5 year old is being prepared for surgery to remove a brain tumor. Nursing actions should be based on which statement? a. Removal of tumor will stop the various symptoms. b. Usually the postoperative dressing covers the entire scalp. c. He is not old enough to be concerned about his head being shaved. d. He is not old enough to Comprehension the significance of the brain.

ANS: B The child should be told what he will look and feel like after surgery. This includes the size of the dressing. The nurse can demonstrate on a doll the expected size and shape of the dressing. Some of the symptoms may be alleviated by the removal of the tumor, but postsurgical headaches and cerebellar symptoms such as ataxia may be aggravated. Children should be prepared for the loss of their hair, and it should be removed in a sensitive, positive manner if the child is awake. Children at this age have poorly defined body boundaries and little knowledge of internal organs. Intrusive experiences are frightening, especially those that disrupt the integrity of the skin.

At the time of a child's death, the nurse tells his mother, "We will miss him so much." What does this statement indicate about the nurse? a. Pretending to be experiencing grief. b. Expressing personal feelings of loss. c. Denying the mother's sense of loss. d. Talking when listening would be better.

ANS: B The death of a patient is one of the most stressful aspects of a critical care or oncology nurse. Nurses experience reactions similar to those of family members because of their involvement with the child and family during the illness. Nurses often have feelings of personal loss when a patient dies. The nurse is experiencing a normal grief response to the death of a patient. There is no implication that the mother's loss is minimized. The nurse is validating the worth of the child.

A nurse is reviewing the laboratory results on a school-age child diagnosed with hypoparathyroidism. Which results are consistent with this condition? a. Decreased serum phosphorus b. Decreased serum calcium c. Increased serum glucose d. Decreased serum cortisol

ANS: B The diagnosis of hypoparathyroidism is made on the basis of clinical manifestations associated with decreased serum calcium and increased serum phosphorus. Decreased serum phosphorus would be seen in hyperparathyroidism, elevated glucose in diabetes, and decreased serum cortisol in adrenocortical insufficiency (Addison's disease).

A neonate born with ambiguous genitalia is diagnosed with congenital adrenogenital hyperplasia. Therapeutic management includes administration of which medication? a. Vitamin D b. Cortisone c. Stool softeners d. Calcium carbonate

ANS: B The most common biochemical defect with congenital adrenal hyperplasia is partial or complete 21-hydroxylase deficiency. With complete deficiency, insufficient amounts of aldosterone and cortisol are produced, so circulatory collapse occurs without immediate replacement. Vitamin D, stool softeners, and calcium carbonate have no role in the therapy of adrenogenital hyperplasia.

Which statement made by the nurse would indicate a correct understanding of palliative care? a. "Palliative care serves to hasten death and make the process easier for the family." b. "Palliative care provides pain and symptom management for the child." c. "The goal of palliative care is to place the child in a hospice setting at the end of life." d. "The goal of palliative care is to act as the liaison between the family, child, and other health care professionals."

ANS: B The primary goal of palliative care is to provide pain and symptom management, not to hasten death or place the child in a hospice setting. Palliative care is provided by a multidisciplinary team whose goal it is to provide active total care for patients whose disease is no longer responding to curative treatment.

What is the most appropriate intervention for the parents of a 6-year-old girl with precocious puberty? a. Advise the parents to consider birth control for their daughter. b. Explain the importance of having the child foster relationships with same-age peers. c. Assure the child's parents that there is no increased risk for sexual abuse because of her appearance. d. Counsel parents that there is no treatment currently available for this disorder.

ANS: B Despite the child's appearance, the child needs to be treated according to her chronologic age and to interact with children in the same age-group. An expected outcome is that the child will adjust socially by exhibiting age-appropriate behaviors and social interactions. Advising the parents of a 6 year old to put their daughter on birth control is not appropriate and will not reverse the effects of precocious puberty. Parents need to be aware that there is an increased risk of sexual abuse for a child with precocious puberty. Treatment for precocious puberty is the administration of gonadotropin-releasing hormone blocker, which slows or reverses the development of secondary sexual characteristics and slows rapid growth and bone aging.

Which children admitted to the pediatric unit would the nurse monitor closely for development of syndrome of inappropriate antidiuretic hormone (SIADH)? (Select all that apply.) a. A newly diagnosed preschooler with type 1 diabetes. b. A school-age child returning from surgery for removal of a brain tumor. c. An infant with suspected meningitis. d. An adolescent with blunt abdominal trauma following a car accident. e. A school-age child with head trauma.

ANS: B, C, E The disorder that results from hypersecretion of ADH from the posterior pituitary hormone. Childhood SIADH usually is caused by disorders affecting the central nervous system, such as infections (meningitis), head trauma, and brain tumors. Type 1 diabetes and blunt abdominal trauma are not likely to cause SIADH since do not affect secretion of this hormone.

A clinic nurse is conducting a staff in-service for other clinic nurses about signs and symptoms of a rhabdomyosarcoma tumor. Which should be included in the teaching session? (Select all that apply.) a. Bone fractures b. Abdominal mass c. Sore throat and ear pain d. Headache e. Ecchymosis of conjunctiva

ANS: B, C, E The initial signs and symptoms of rhabdomyosarcoma tumors are related to the site of the tumor and compression of adjacent organs. Some tumor locations, such as the orbit, manifest early in the course of the illness. Other tumors, such as those of the retroperitoneal area, only produce symptoms when they are relatively large and compress adjacent organs. Unfortunately, many of the signs and symptoms attributable to rhabdomyosarcoma are vague and frequently suggest a common childhood illness, such as "earache" or "runny nose." An abdominal mass, sore throat and ear pain, and ecchymosis of conjunctiva are signs of a rhabdomyosarcoma tumor. Bone fractures would be seen in osteosarcoma, and a headache is a sign of a brain tumor.

Which expected appearance will the nurse explain to parents of an infant returning from surgery after an enucleation was performed to treat retinoblastoma? (Select all that apply.) a. A lot of drainage will come from the affected socket. b. The face may be edematous or ecchymotic. c. The eyelids will be sutured shut for the first week. d. There will be an eye pad dressing taped over the surgical site. e. The implanted sphere is covered with conjunctiva and resembles the lining of the mouth.

ANS: B, D, E After enucleation surgery, the parents are prepared for the child's facial appearance. An eye patch is in place, and the child's face may be edematous or ecchymotic. Parents often fear seeing the surgical site because they imagine a cavity in the skull. A surgically implanted sphere maintains the shape of the eyeball, and the implant is covered with conjunctiva. When the eyelids are open, the exposed area resembles the mucosal lining of the mouth. The dressing, consisting of an eye pad taped over the surgical site, is changed daily. The wound itself is clean and has little or no drainage. So expecting a lot of drainage is not accurate to tell parents. The eyelids are not sutured shut after enucleation surgery.

A nurse is caring for a child who is near death. Which physical signs indicate the child is approaching death? (Select all that apply.) a. Body feels warm b. Tactile sensation decreasing c. Speech becomes rapid d. Change in respiratory pattern e. Difficulty swallowing

ANS: B, D, E Physical signs of approaching death include tactile sensation beginning to decrease, a change in respiratory pattern, and difficulty swallowing. Even though there is a sensation of heat, the body feels cool, not warm, and speech becomes slurred, not rapid.

The nurse case manager is planning a care conference about a young child who has complex health care needs and will soon be discharged home. Whom should the nurse invite to the conference? a. Family and nursing staff b. Social worker, nursing staff, and primary care physician c. Family and key health professionals involved in child's care d. Primary care physician and key health professionals involved in child's care

ANS: C A multidisciplinary conference is necessary for coordination of care for children with complex health needs. The family and key health professionals who are involved in the child's care are included. The nursing staff can address the nursing care needs of the child with the family, but other involved disciplines must be included. The family must be included in the discharge conferences, which allow them to determine what education they will require and the resources needed at home. A member of the nursing staff must be included to review the nursing needs of the child.

A parent asks the nurse why self-monitoring of blood glucose is being recommended for her child with diabetes. The nurse should base the explanation on what information? a. It is a less expensive method of testing. b. It is not as accurate as laboratory testing. c. Children need to learn to manage their diabetes. d. The parents are better able to manage the disease.

ANS: C Blood glucose self-management has improved diabetes management and can be used successfully by children from the time of diagnosis. Insulin dosages can be adjusted based on blood sugar results. Blood glucose monitoring is more expensive but provides improved management. It is as accurate as equivalent testing done in laboratories. The ability to self-test allows the child to balance diet, exercise, and insulin. The parents are partners in the process, but the child should be taught how to manage the disease.

At what age do most children have an adult concept of death as being inevitable, universal, and irreversible? a. 4 to 5 years b. 6 to 8 years c. 9 to 11 years d. 12 to 16 years

ANS: C By age 9 to 11 years, children have an adult concept of death. They realize that it is inevitable, universal, and irreversible. Preschoolers and young school-age children are too young to have an adult concept of death. Adolescents have a mature understanding of death.

The nurse is teaching the parents of a child who is receiving propylthiouracil for the treatment of hyperthyroidism (Graves' disease). Which statement made by the parent indicates a correct understanding of the teaching? a. "I would expect my child to gain weight while taking this medication." b. "I would expect my child to experience episodes of ear pain while taking this medication." c. "If my child develops a sore throat and fever, I should contact the physician immediately." d. "If my child develops the stomach flu, my child will need to be hospitalized."

ANS: C Children being treated with propylthiouracil must be carefully monitored for the side effects of the drug. Parents must be alerted that sore throat and fever accompany the grave complication of leukopenia. These symptoms should be immediately reported. Weight gain, episodes of ear pain, and stomach flu are not usually associated with leukopenia.

What is the priority nursing goal for a 14 year old diagnosed with Graves' disease? a. Relieving constipation b. Allowing the adolescent to make decisions about whether or not to take medication c. Verbalizing the importance of monitoring for medication side effects d. Developing alternative educational goals

ANS: C Children being treated with propylthiouracil or methimazole must be carefully monitored for side effects of the drug. Because sore throat and fever accompany the grave complication of leukopenia, these children should be seen by a health care practitioner if such symptoms occur. Parents and children should be taught to recognize and report symptoms immediately. The adolescent with Graves' disease is not likely to be constipated. Adherence to the medication schedule is important to ensure optimal health and wellness. Medications should not be skipped and dose regimens should not be tapered by the child without consultation with the child's medical provider. The management of Graves' disease does not interfere with school attendance and does not require alternative educational plans.

The nurse is admitting a toddler with the diagnosis of juvenile hypothyroidism. Which is a common clinical manifestation of this disorder? a. Insomnia b. Diarrhea c. Dry skin d. Accelerated growth

ANS: C Dry skin, mental decline, and myxedematous skin changes are associated with juvenile hypothyroidism. Children with hypothyroidism are usually sleepy. Constipation is associated with hypothyroidism. Decelerated growth is common in juvenile hypothyroidism.

When infants are seen for fractures, which nursing intervention is a priority? a. No intervention is necessary. It is not uncommon for infants to fracture bones. b. Assess the family's safety practices. Fractures in infants usually result from falls. c. Assess for child abuse. Fractures in infants are often nonaccidental. d. Assess for genetic factors.

ANS: C Fractures in infants warrant further investigation to rule out child abuse. Fractures in children younger than 1 year are unusual because of the cartilaginous quality of the skeleton; a large amount of force is necessary to fracture their bones. Infants should be cared for in a safe environment and should not be falling. Fractures in infancy are usually nonaccidental rather than related to a genetic factor.

What is the most common clinical manifestation(s) of brain tumors in children? a. Irritability b. Seizures c. Headaches and vomiting d. Fever and poor fine motor control

ANS: C Headaches, especially on awakening, and vomiting that is not related to feeding are the most common clinical manifestation(s) of brain tumors in children. Irritability, seizures, and fever and poor fine motor control are clinical manifestations of brain tumors, but headaches and vomiting are the most common.

When does idiopathic scoliosis become most noticeable? a. Newborn period b. When child starts to walk c. During preadolescent growth spurt d. Adolescence

ANS: C Idiopathic scoliosis is most noticeable during the preadolescent growth spurt and is seldom apparent before age 10 years.

Which statement best describes idiopathic hypopituitarism? a. Growth is normal during the first 3 years of life. b. Weight is usually more retarded than height. c. Skeletal proportions are normal for age. d. Most of these children have subnormal intelligence.

ANS: C In children with idiopathic hypopituitarism, the skeletal proportions are normal. Growth is within normal limits for the first year of life. Height is usually more delayed than weight. Intelligence is not affected by hypopituitarism.

Which is most descriptive of the pathophysiology of leukemia? a. Increased blood viscosity occurs. b. Thrombocytopenia (excessive destruction of platelets) occurs. c. Unrestricted proliferation of immature white blood cells (WBCs) occurs. d. First stage of coagulation process is abnormally stimulated.

ANS: C Leukemia is a group of malignant disorders of the bone marrow and lymphatic system. It is defined as an unrestricted proliferation of immature WBCs in the blood-forming tissues of the body. Increased blood viscosity may occur secondary to the increased number of WBCs. Thrombocytopenia may occur secondary to the overproduction of WBCs in the bone marrow. The coagulation process is unaffected by leukemia.

At what age is sexual development in boys and girls considered to be precocious? a. Boys, 11 years; girls, 9 years b. Boys, 12 years; girls, 10 years c. Boys, 9 years; girls, 8 years d. Boys, 10 years; girls, 9.5 years

ANS: C Manifestations of sexual development before age 9 in boys and age 8 in girls are considered precocious and should be investigated. Boys older than 9 years of age and girls older than 8 years of age fall within the expected range of pubertal onset.

A neonate is born with mild clubfeet. When the parents ask the nurse how this will be corrected, the nurse should base the explanation on what fact? a. Traction is tried first. b. Surgical intervention is needed. c. Frequent, serial casting is tried first. d. Children outgrow this condition when they learn to walk.

ANS: C Serial casting, the preferred treatment, is begun shortly after birth before discharge from the nursery. Successive casts allows for gradual stretching of skin and tight structures on the medial side of the foot. Manipulation and casting of the leg are repeated frequently (every week) to accommodate the rapid growth of early infancy. Surgical intervention is done only if serial casting is not successful. Children do not improve without intervention.

A school-age child is diagnosed with a life-threatening illness. The parents want to protect their child from knowing the seriousness of the illness. What information should the nurse provide to these parents? a. This will help the child cope effectively by denial. b. This attitude is helpful to give parents time to cope. c. Terminally ill children know when they are seriously ill. d. Terminally ill children usually choose not to discuss the seriousness of their illness.

ANS: C The child needs honest and accurate information about the illness, treatments, and prognosis. Children, even at a young age, realize that something is seriously wrong and that it involves them. The nurse should help parents understand the importance of honesty. The child will know that something is wrong because of the increased attention of health professionals. This would interfere with denial as a form of coping. Parents may need professional support and guidance from a nurse or social worker in this process. Children will usually tell others how much information they want about their condition.

The father, of a 9 year old diagnosed with several physical disabilities, explains to the nurse that his child concentrates on what he/she can do rather than cannot do and is as independent as possible. How should the nurse's best interpret this statement? a. The father is experiencing denial. b. The father is expressing his own views. c. The child is using an adaptive coping style. d. The child is using a maladaptive coping style.

ANS: C The father is describing a well-adapted child who has learned to accept physical limitations. These children function well at home, at school, and with peers. They have an understanding of their disorder that allows them to accept their limitations, assume responsibility for care, and assist in treatment and rehabilitation. The father is not denying the child's limitations or expressing his own views. This is descriptive of an adaptive coping style.

The nurse is teaching a family how to care for their infant in a Pavlik harness to treat developmental dysplasia of the hips (DDH). What information should be included? a. Apply lotion or powder to minimize skin irritation. b. Remove the harness several times a day to prevent contractures. c. Hip stabilization usually occurs within 12 weeks. d. Place a diaper over harness, preferably using a superabsorbent disposable diaper that is relatively thin.

ANS: C The harness is worn continuously until the hip is proved stable on both clinical and ultrasound examination, usually within 6 to 12 weeks. Lotions and powders should not be used with the harness. The harness should not be removed, except as directed by the practitioner. A thin disposable diaper can be placed under the harness.

What is the characteristic of the immune-mediated type 1 diabetes mellitus? a. Ketoacidosis is infrequent b. Onset is gradual c. Age at onset is usually younger than 18 years d. Oral agents are often effective for treatment

ANS: C The immune-mediated type 1 diabetes mellitus typically has its onset in children or young adults. Peak incidence is between the ages of 10 and 15 years. Infrequent ketoacidosis, gradual onset, and treatment with oral agents are more consistent with type 2 diabetes.

What nursing consideration is especially important when caring for a child diagnosed with juvenile idiopathic arthritis (JIA)? a. Apply ice packs to relieve stiffness and pain. b. Administer acetaminophen to reduce inflammation. c. Teach child and family the correct administration of medications. d. Encourage range-of-motion exercises during periods of inflammation.

ANS: C The management of JIA is primarily pharmacologic. The family should be instructed regarding administration of medications and the value of a regular schedule of administration to maintain a satisfactory blood level in the body. They need to know that nonsteroidal antiinflammatory drugs should not be given on an empty stomach and to be alert for signs of toxicity. Warm, moist heat is best for relieving stiffness and pain. Acetaminophen does not have antiinflammatory effects. Range-of-motion exercises should not be done during periods of inflammation.

A nurse is conducting discharge teaching for parents of an infant diagnosed with osteogenesis imperfecta (OI). Further teaching is indicated if the parents make which statement? a. "We will be very careful handling the baby." b. "We will lift the baby by the buttocks when diapering." c. "We're glad there is a cure for this disorder." d. "We will schedule follow-up appointments as instructed."

ANS: C The treatment for OI is primarily supportive. Although patients and families are optimistic about new research advances, there is no cure. The use of bisphosphonate therapy with IV pamidronate to promote increased bone density and prevent fractures has become standard therapy for many children with OI; however, long bones are weakened by prolonged treatment. Infants and children with this disorder require careful handling to prevent fractures. They must be supported when they are being turned, positioned, moved, and held. Even changing a diaper may cause a fracture in severely affected infants. These children should never be held by the ankles when being diapered but should be gently lifted by the buttocks or supported with pillows. Follow-up appointments for treatment with bisphosphonate can be expected.

Glucocorticoids, mineralocorticoids, and sex steroids are secreted by which organ? a. Thyroid gland b. Parathyroid glands c. Adrenal cortex d. Anterior pituitary

ANS: C These hormones are secreted by the adrenal cortex. The thyroid gland produces thyroid hormone and thyrocalcitonin. The parathyroid glands produce parathyroid hormone. The anterior pituitary produces hormones such as growth hormone, thyroid-stimulating hormone, adrenocorticotropic hormone, gonadotropin, prolactin, and melanocyte-stimulating hormone.

A child diagnosed with hypopituitarism is being started on growth hormone (GH) therapy. Nursing considerations should be based on which information? a. Treatment is most successful if it is started during adolescence. b. Treatment is considered successful if children attain full stature by adulthood. c. Replacement therapy requires daily subcutaneous injections. d. Replacement therapy will be required throughout the child's lifetime.

ANS: C Additional support is required for children who require hormone replacement therapy, such as preparation for daily subcutaneous injections and education for self-management during the school-age years. Young children, obese children, and those who are severely GH deficient have the best response to therapy. When therapy is successful, children can attain their actual or near-final adult height at a slower rate than their peers. Replacement therapy is not needed after attaining final height. They are no longer GH deficient.

The nurse is caring for a preschool child with a cast applied recently for a fractured tibia. Which assessment findings indicate possible compartment syndrome? (Select all that apply.) a. Palpable distal pulse b. Capillary refill to extremity of <3 seconds c. Severe pain not relieved by analgesics d. Tingling of extremity e. Inability to move extremity

ANS: C, D, E Indications of compartment syndrome are severe pain not relieved by analgesics, tingling of extremity, and inability to move extremity. A palpable distal pulse and capillary refill to the extremity of <3 seconds are expected findings.

A nurse is planning care for a school-age child diagnosed with type 1 diabetes. Which insulin preparations are either rapid or short acting? (Select all that apply.) a. Novolin N b. Lantus c. NovoLog d. Novolin R

ANS: C, D Rapid-acting insulin (e.g., NovoLog) reaches the blood within 15 minutes after injection. The insulin peaks 30 to 90 minutes later and may last as long as 5 hours. Short-acting (regular) insulin (e.g., Novolin R) usually reaches the blood within 30 minutes after injection. The insulin peaks 2 to 4 hours later and stays in the blood for about 4 to 8 hours. Intermediate-acting insulins (e.g., Novolin N) reach the blood 2 to 6 hours after injection. The insulins peak 4 to 14 hours later and stay in the blood for about 14 to 20 hours. Long-acting insulin (e.g., Lantus) takes 6 to 14 hours to start working. It has no peak or a very small peak 10 to 16 hours after injection. The insulin stays in the blood between 20 and 24 hours.

The nurse is conducting discharge teaching about signs and symptoms of heart failure to parents of an infant with a repaired tetralogy of Fallot. Which signs and symptoms should the nurse include? (Select all that apply.) a. Warm flushed extremities b. Weight loss c. Decreased urinary output d. Sweating (inappropriate) e. Anorexia

ANS: C, D, E The signs and symptoms of heart failure include decreased urinary output, sweating, and poor feeding. Other signs include pale, cool extremities, not warm and flushed, and weight gain, not weight loss.

Which is a common clinical manifestation of Hodgkin disease? a. Petechiae b. Bone and joint pain c. Painful, enlarged lymph nodes d. Enlarged, firm, nontender lymph nodes

ANS: D Asymptomatic, enlarged, cervical or supraclavicular lymphadenopathy is the most common presentation of Hodgkin disease. Petechiae are usually associated with leukemia. Bone and joint pain are not likely in Hodgkin disease. The enlarged nodes are rarely painful.

At what developmental period do children have the most difficulty coping with death, particularly if it is their own? a. Toddlerhood b. Preschool c. School-age d. Adolescence

ANS: D Because of their mature understanding of death, remnants of guilt and shame, and issues with deviations from normal, adolescents have the most difficulty coping with death. Toddlers and preschoolers are too young to have difficulty coping with their own death. They will fear separation from parents. School-age children will fear the unknown, such as the consequences of the illness and the threat to their sense of security.

Which clinical manifestation may occur in the child who is prescribed methimazole for the treatment of hyperthyroidism (Graves' disease)? a. Seizures b. Enlargement of all lymph glands c. Pancreatitis or cholecystitis d. Sore throat or fever

ANS: D Children being treated with propylthiouracil or methimazole must be carefully monitored for side effects of the drug. Because sore throat and fever accompany the grave complication of leukopenia, these children should be seen by a health care practitioner if such symptoms occur. Neither seizures, cholecystitis nor pancreatitis are associated with the administration of methimazole. Enlargement of the salivary and cervical lymph glands may occur.

Most parents of children with special needs tend to experience chronic sorrow. How may chronic sorrow be characterized? a. Lack of acceptance of the child's limitation. b. Lack of available support to prevent sorrow. c. Periods of intensified sorrow when experiencing anger and guilt. d. Periods of intensified sorrow and loss that occur in waves over time.

ANS: D Chronic sorrow is manifested by feelings of sorrow and loss that recur in waves over time. The sorrow is in response to the recognition of the child's limitations. The family should be assessed in an ongoing manner to provide appropriate support as the needs of the family change. The sorrow is not preventable. The chronic sorrow occurs during the reintegration and acknowledgment stage.

The nurse is caring for a child with suspected diabetes insipidus. Which clinical manifestation would be observable? a. Oliguria b. Glycosuria c. Nausea and vomiting d. Polydipsia

ANS: D Excessive urination accompanied by insatiable thirst is the primary clinical manifestation of diabetes. These symptoms may be so severe that the child does little other than drink and urinate. Oliguria is decreased urine production and is not associated with diabetes insipidus. Glycosuria is associated with diabetes mellitus. Nausea and vomiting are associated with inappropriate antidiuretic hormone secretion.

What would cause a nurse to suspect that an infection has developed under a cast? a. Complaint of paresthesia b. Cold toes c. Increased respirations d. "Hot spots" felt on cast surface

ANS: D If hot spots are felt on the cast surface, they usually indicate infection beneath the area. This should be reported so a window can be made in the cast to observe the site. The "five Ps" of ischemia from a vascular injury include pain, pallor, pulselessness, paresthesia, and paralysis. Paresthesia is an indication of vascular injury, not infection. Cold toes may be indicative of too tight a cast and need further evaluation. Increased respirations may indicate a respiratory infection or pulmonary emboli. This should be reported, and the child should be evaluated.

Which term is used to describe an abnormally increased convex angulation in the curvature of the thoracic spine? a. Scoliosis b. Ankylosis c. Lordosis d. Kyphosis

ANS: D Kyphosis is an abnormally increased convex angulation in the curve of the thoracic spine. Scoliosis is a complex spinal deformity usually involving lateral curvature, spinal rotation causing rib asymmetry, and thoracic hypokyphosis. Ankylosis is the immobility of a joint. Lordosis is an accentuation of the cervical or lumbar curvature beyond physiologic limits.

Which medication is usually tried first when a child is diagnosed with juvenile idiopathic arthritis (JIA)? a. Aspirin b. Corticosteroids c. Cytotoxic drugs such as methotrexate d. Nonsteroidal antiinflammatory drugs (NSAIDs)

ANS: D NSAIDs are the first drugs used in JIA. Naproxen, ibuprofen, and tolmetin are approved for use in children. Aspirin, once the drug of choice, has been replaced by the NSAIDs because they have fewer side effects and easier administration schedules. Corticosteroids are used for life-threatening complications, incapacitating arthritis, and uveitis. Methotrexate is a second-line therapy for JIA.

Type 1 diabetes mellitus is suspected in an adolescent. Which clinical manifestation may be present? a. Moist skin b. Weight gain c. Fluid overload d. Poor wound healing

ANS: D Poor wound healing is often an early sign of type 1 diabetes mellitus. Dry skin, weight loss, and dehydration are clinical manifestations of type 1 diabetes mellitus.

A child will start treatment for precocious puberty. This involves injections of which synthetic medication? a. Thyrotropin b. Gonadotropins c. Somatotropic hormone d. Luteinizing hormone-releasing hormone

ANS: D Precocious puberty of central origin is treated with monthly subcutaneous injections of luteinizing hormone-releasing hormone. Thyrotropin, gonadotropin, and somatotropic hormone are not appropriate therapies for precocious puberty.

What are the manifestations of hypoglycemia? a. Lethargy b. Thirst c. Nausea and vomiting d. Shaky feeling and dizziness

ANS: D Some of the clinical manifestations of hypoglycemia include shaky feelings; dizziness; difficulty concentrating, speaking, focusing, and coordinating; sweating; and pallor. Lethargy, thirst, and nausea and vomiting are manifestations of hyperglycemia.

A child eats some sugar cubes after experiencing symptoms of hypoglycemia. This rapid-releasing sugar should be followed by: a. saturated and unsaturated fat. b. fruit juice. c. several glasses of water. d. complex carbohydrate and protein.

ANS: D Symptoms of hypoglycemia are treated with a rapid-releasing sugar source followed by a complex carbohydrate and protein. Saturated and unsaturated fat, fruit juice, and several glasses of water do not provide the child with complex carbohydrate and protein necessary to stabilize the blood sugar.

Myelosuppression, associated with chemotherapeutic agents or some malignancies such as leukemia can cause bleeding tendencies because of what resulting outcome? a. Decrease in leukocytes b. Increase in lymphocytes c. Vitamin C deficiency d. Decrease in blood platelets

ANS: D The decrease in blood platelets secondary to the myelosuppression of chemotherapy can cause an increase in bleeding. The child and family should be alerted to avoid risk of injury. Decrease in leukocytes, increase in lymphocytes, and vitamin C deficiency would not affect bleeding tendencies.

A 4-year-old child is newly diagnosed with Legg-Calvé-Perthes disease. Nursing considerations should include which action? a. Encouraging normal activity for as long as is possible. b. Explaining the cause of the disease to the child and family. c. Preparing the child and family for long-term, permanent disabilities. d. Teaching the family the care and management of the corrective appliance.

ANS: D The family needs to learn the purpose, function, application, and care of the corrective device and the importance of compliance to achieve the desired outcome. The initial therapy is rest and nonweight bearing, which helps reduce inflammation and restore motion. Legg-Calvé-Perthes is a disease with an unknown etiology. A disturbance of circulation to the femoral capital epiphysis produces an ischemic aseptic necrosis of the femoral head. The disease is self-limiting, but the ultimate outcome of therapy depends on early and efficient therapy and the child's age at onset.

The nurse is teaching the parents of a 7-year-old child who has just had a cast applied for a fractured arm with the wrist and elbow immobilized. Which instructions should be included in the teaching? a. Swelling of the fingers is to be expected for the next 48 hours. b. Immobilize the shoulder to decrease pain in the arm. c. Allow the affected limb to hang down for 1 hour each day. d. Elevate casted arm when resting and when sitting up.

ANS: D The injured extremity should be kept elevated while resting and in a sling when upright. This will increase venous return. Swelling of the fingers may indicate neurovascular damage and should be reported immediately. Permanent damage can occur within 6 to 8 hours. Joints above and below the cast on the affected extremity should be moved. The child should not engage in strenuous activity for the first few days. Rest with elevation of the extremity is encouraged.

Which term best describes a multidisciplinary approach to the management of a terminal illness that focuses on symptom control and support? a. Dying care b. Curative care c. Restorative care d. Palliative care

ANS: D This is one of the definitions of palliative care. The goal of palliative care is the achievement of the highest possible quality of life for patients and their families. Curative care would infer providing a cure for the disease or disorder while restorative care involves measures to regain past abilities. Dying care generally refers to the care of an individual in the final stage of life.

The nurse, talking with the tearful parent of a child newly diagnosed with a chronic illness, asks, "Who do you talk with when something is worrying you?" What is the purpose of this statement? a. Inappropriate, because parent is so upset. b. A diversion of the present crisis to similar situations with which parent has dealt. c. An intervention to find someone to help parent. d. Part of assessing parent's available support system.

ANS: D This question will provide information about the marital relationship (does the parent speak to the spouse?), alternate support systems, and ability to communicate. These are very important data for the nurse to obtain and an appropriate part of an accurate assessment. By assessing these areas, the nurse can facilitate the identification and use of community resources as needed. The nurse is obtaining information to help support the parent through the diagnosis. The parent is not in need of additional parenting help at this time.

The nurse uses the palms of the hands when handling a wet cast to achieve what outcome? a. Assess dryness of the cast b. Facilitate easy turning c. Keep the patient's limb balanced d. Avoid indenting the cast

ANS: D Wet casts should be handled by the palms of the hands, not the fingers, to prevent creating pressure points. Assessing dryness, facilitating easy turning, or keeping the patient's limb balanced are not reasons for using the palms of the hand rather than the fingers when handling a wet cast.

Which nursing intervention is appropriate to assess for neurovascular competency in a child suspected of experiencing compartment syndrome? a. The degree of motion and ability to position the extremity. b. The length, diameter, and shape of the extremity. c. The amount of swelling noted in the extremity and pain intensity. d. The skin color, temperature, movement, sensation, and capillary refill of the extremity

ANS: D A neurovascular evaluation includes assessing skin color and temperature, ability to move the affected extremity, degree of sensation experienced, and speed of capillary refill in the extremity. The degree of motion in the affected extremity and ability to position the extremity are incomplete assessments of neurovascular competency. The length, diameter, and shape of the extremity are not assessment criteria in a neurovascular evaluation. Although the amount of swelling is an important factor in assessing an extremity, it is not a criterion for a neurovascular assessment.

The nurse is talking to a parent of an infant with heart failure about feeding the infant. Which statement about feeding the child is correct? a. "You may need to increase the caloric density of your infant's formula." b. "You should feed your baby every 2 hours." c. "You may need to increase the amount of formula your infant eats with each feeding." d. "You should place a nasal oxygen cannula on your infant during and after each feeding."

a. "You may need to increase the caloric density of your infant's formula." The metabolic rate of infants with heart failure is greater because of poor cardiac function and increased heart and respiratory rates. Their caloric needs are greater than those of the average infants, yet their ability to take in the calories is diminished by their fatigue. Infants with heart failure should be fed every 3 hours; a 2-hour schedule does not allow for enough rest, and a 4-hour schedule is too long. Fluids must be carefully monitored because of the heart failure. Infants do not require supplemental oxygen with feedings.

Which information should the nurse give to a child who is to have magnetic resonance imaging (MRI) of the brain? a. "Your head will be kept from moving during the procedure." b. "You will have to drink a special fluid before the test." c. "You will have to lie flat after the test is finished." d. "You will have electrodes placed on your head with glue."

a. "Your head will be kept from moving during the procedure." To reduce fear and enhance cooperation during the MRI, the child should be made aware that the head will be restricted to obtain accurate information. Drinking fluids is usually done for neurologic procedures. A child should lie flat after a lumbar puncture, not after an MRI. Electrodes are attached to the head for an electroencephalogram.

A toddler, who fell out of a second-story window, had brief loss of consciousness and vomited 4 times. Since admission, the child has been alert and oriented. The mother asks why a computed tomography (CT) scan is required when the child "seems fine." The nurse should base the response on the need to monitor for what possible complication? a. A brain injury b. Coma c. Seizures d. Skull fracture

a. A brain injury The child's history of the fall, brief loss of consciousness, and vomiting 4 times necessitate evaluation of a potential brain injury. The severity of a head injury may not be apparent on clinical examination but will be detectable on a CT scan. All the remaining options are a result of varying degrees of brain injury.

What is the most appropriate nursing diagnosis for a child diagnosed with moderate anemia? a. Activity intolerance related to generalized weakness b. Decreased cardiac output related to abnormal hemoglobin c. Risk for injury related to depressed sensorium d. Risk for Injury related to dehydration and abnormal hemoglobin

a. Activity intolerance related to generalized weakness The basic pathology in anemia is the decreased oxygen-carrying capacity of the blood. The nurse must assess the child's activity level (response to the physiologic state). The nursing diagnosis would reflect the activity intolerance. In generalized anemia no abnormal hemoglobin may be present. Only at a level of very severe anemia does cardiac output become altered. No decreased sensorium exists until profound anemia occurs. Dehydration and abnormal hemoglobin are not usually part of anemia.

The nurse is admitting a child with rheumatic fever. Which therapeutic management should the nurse expect to implement? a. Administering penicillin b. Avoiding salicylates (aspirin) c. Imposing strict bed rest for 4 to 6 weeks d. Administering corticosteroids if chorea develops

a. Administering penicillin The goal of medical management is the eradication of the hemolytic streptococci. Penicillin is the drug of choice. Salicylates can be used to control the inflammatory process, especially in the joints, and reduce the fever and discomfort. Bed rest is recommended for the acute febrile stage, but it does not need to be strict. The chorea is transient and will resolve without treatment.

What is the nurse's first action when planning to teach the parents of an infant with a congenital heart defect (CHD)? a. Assess the parents' anxiety level and readiness to learn b. Gather literature for the parents c. Secure a quiet place for teaching d. Discuss the plan with the nursing team

a. Assess the parents' anxiety level and readiness to learn Any effort to organize the right environment, plan, or literature is of no use if the parents are not ready to learn or have high anxiety. Decreasing their level of anxiety is often needed before new information can be processed. A baseline assessment of prior knowledge should be taken into consideration before developing any teaching plan. Locating a quiet place for meeting with parents is appropriate; however, an assessment should be done before any teaching is done. Discussing a teaching plan with the nursing team is appropriate after an assessment of the parents' knowledge and readiness.

Latex allergy is suspected in a child with spina bifida. What intervention should be included in the child's plan of care? a. Avoiding using any latex product b. Using only nonallergenic latex products c. Administering medication for long-term desensitization d. Teaching the family about long-term management of asthma

a. Avoiding using any latex product Care must be taken that individuals who are at high risk for latex allergies do not come in direct or secondary contact with products or equipment containing latex at any time during medical treatment. There are no nonallergenic latex products. At this time desensitization is not an option. The child does not have asthma. The parents must be taught about allergy and the risk of anaphylaxis.

An adolescent boy is brought to the emergency department after a motorcycle accident. His respirations are deep, periodic, and gasping. There are extreme fluctuations in blood pressure. Pupils are dilated and fixed. What type of head injury should the nurse suspect? a. Brainstem b. Skull fracture c. Subdural hemorrhage d. Epidural hemorrhage

a. Brainstem Signs of brainstem injury include deep, rapid, periodic or intermittent, and gasping respirations. Wide fluctuations or noticeable slowing of the pulse, widening pulse pressure, or extreme fluctuations in blood pressure are consistent with a brainstem injury. Skull fracture and subdural and epidural hemorrhages are not consistent with these signs.

Which clinical manifestations would suggest hydrocephalus in a neonate? a. Bulging fontanel and dilated scalp veins b. Closed fontanel and high-pitched cry c. Constant low-pitched cry and restlessness d. Depressed fontanel and decreased blood pressure

a. Bulging fontanel and dilated scalp veins Bulging fontanel, dilated scalp veins, and separated sutures are clinical manifestations of hydrocephalus in neonates. Closed fontanel and high-pitched cry, constant low-pitched cry and restlessness, and depressed fontanel and decreased blood pressure are not clinical manifestations of hydrocephalus, but all should be referred for evaluation.

Which drug is an angiotensin-converting enzyme (ACE) inhibitor? a. Captopril b. Furosemide c. Spironolactone d. Chlorothiazide

a. Captopril Capoten is an ACE inhibitor. Lasix is a loop diuretic. Aldactone blocks the action of aldosterone. Diuril works on the distal tubules.

Which complication should the nurse asses for when caring for a child post cardiac catheterization? a. Cardiac arrhythmia b. Hypostatic pneumonia c. Congestive heart failure d. Rapidly increasing blood pressure

a. Cardiac arrhythmia Because a catheter is introduced into the heart, a risk exists of catheter-induced arrhythmias occurring during the procedure. These are usually transient. Hypostatic pneumonia, congestive heart failure, and rapidly increasing blood pressure are not risks usually associated with cardiac catheterization.

The nurse is conducting discharge teaching with parents of a preschool child with a myelomeningocele, repaired at birth, who is being discharged from the hospital after a urinary tract infection (UTI). Which should the nurse include in the discharge instructions related to management of the child's genitourinary function? (Select all that apply.) a. Continue to perform the clean intermittent catheterizations (CIC) at home. b. Administer the oxybutynin chloride as prescribed. c. Reduce fluid intake in the afternoon and evening hours. d. Monitor for signs of a recurrent UTI. e. Administer furosemide as prescribed.

a. Continue to perform the clean intermittent catheterizations (CIC) at home. b. Administer the oxybutynin chloride as prescribed. d. Monitor for signs of a recurrent UTI. Discharge teaching to prevent renal complications in a child with myelomeningocele include: (1) regular urologic care with prompt and vigorous treatment of infections; (2) a method of regular emptying of the bladder, such as clean intermittent catheterization (CIC) taught to and performed by parents and self-catheterization taught to children; and (3) medications to improve bladder storage and continence, such as oxybutynin chloride and tolterodine. Fluids should not be limited, and furosemide is not used to improve renal function for children with myelomeningocele.

What beneficial effect is achieved by administering digoxin? a. Decreases edema b. Decreases cardiac output c. Increases heart size d. Increases venous pressure

a. Decreases edema Digoxin has a rapid onset and is useful in increasing cardiac output, decreasing venous pressure, and as a result decreasing edema. Heart size is decreased by digoxin.

Which nursing intervention is appropriate when caring for a child who has experienced a seizure? a. Describe and record the seizure activity observed. b. Restrain the child when seizure occurs to prevent bodily harm. c. Place a tongue blade between the teeth if they become clenched. d. Suction the child during a seizure to prevent aspiration.

a. Describe and record the seizure activity observed. When a child is having a seizure, the priority nursing care is observation of the child and seizure. The nurse then describes and records the seizure activity. The child should not be restrained, and nothing should be placed in his or her mouth. This may cause injury. To prevent aspiration, if possible, the child should be placed on his or her side, facilitating drainage.

A nurse should expect which cerebrospinal fluid (CSF) laboratory results on a child diagnosed with bacterial meningitis? (Select all that apply.) a. Elevated white blood cell (WBC) count b. Decreased protein c. Decreased glucose d. Cloudy in color e. Increase in red blood cells (RBCs)

a. Elevated white blood cell (WBC) count c. Decreased glucose d. Cloudy in color The CSF laboratory results for bacterial meningitis include elevated WBC counts, cloudy or milky in color, and decreased glucose. The protein is elevated and there should be no RBCs present. RBCs are present when the tap was traumatic.

What is the priority nursing intervention when a child is unconscious after a fall? a. Establish an adequate airway b. Perform neurologic assessment c. Monitor intercranial pressure d. Determine whether a neck injury is present

a. Establish an adequate airway Respiratory effectiveness is the primary concern in the care of the unconscious child. Establishing an adequate airway is always the first priority. A neurologic assessment and determination of neck injury are performed after breathing and circulation are stabilized. Intracranial, not intercranial, pressure is monitored if indicated after airway, breathing, and circulation are maintained.

Which problem is most often associated with myelomeningocele? a. Hydrocephalus b. Craniosynostosis c. Biliary atresia d. Esophageal atresia

a. Hydrocephalus Hydrocephalus is an associated anomaly in 80% to 90% of children. Craniosynostosis is the premature closing of the cranial sutures and is not associated with myelomeningocele. Biliary and esophageal atresias are not associated with myelomeningocele.

Spastic cerebral palsy is characterized by what presentation? a. Hypertonicity and poor control of posture, balance, and coordinated motion b. Athetosis and dystonic movements c. Wide-based gait and poor performance of rapid, repetitive movements d. Tremors and lack of active movement

a. Hypertonicity and poor control of posture, balance, and coordinated motion Hypertonicity and poor control of posture, balance, and coordinated motion are part of the classification of spastic cerebral palsy. Athetosis and dystonic movements are part of the classification of dyskinetic/athetoid cerebral palsy. Wide-based gait and poor performance of rapid, repetitive movements are part of the classification of ataxic cerebral palsy. Tremors and lack of active movement may indicate other neurologic disorders.

What is the priority nursing intervention for a child hospitalized with hemarthrosis resulting from hemophilia? a. Immobilization and elevation of the affected joint b. Administration of acetaminophen for pain relief c. Assessment of the child's response to hospitalization d. Assessment of the impact of hospitalization on the family system

a. Immobilization and elevation of the affected joint Immobilization and elevation of the joint will prevent further injury until bleeding is resolved. Although acetaminophen may help with pain associated with the treatment of hemarthrosis, it is not the priority nursing intervention. Assessment of a child's response to hospitalization is relevant to all hospitalized children; however, in this situation, psychosocial concerns are secondary to physiologic concerns. A priority nursing concern for this child is the management of hemarthrosis. Assessing the impact of hospitalization on the family system is relevant to all hospitalized children; however, it is not the priority in this situation.

Which assessment findings should the nurse note in a school-age child diagnosed with Duchenne's muscular dystrophy (DMD)? (Select all that apply.) a. Lordosis b. Gower's sign c. Kyphosis d. Scoliosis e. Waddling gait

a. Lordosis b. Gower's sign e. Waddling gait Difficulties in running, riding a bicycle, and climbing stairs are usually the first symptoms noted in DMD. Typically, affected boys have a waddling gait and lordosis, fall frequently, and develop a characteristic manner of rising from a squatting or sitting position on the floor (Gower's sign). Lordosis occurs as a result of weakened pelvic muscles, and the waddling gait is a result of weakness in the gluteus medius and maximus muscles. Kyphosis and scoliosis are not assessment findings with DMD.

Which drug would be used to treat a child who has increased intracranial pressure (ICP) resulting from cerebral edema? a. Mannitol b. Epinephrine hydrochloride c. Atropine sulfate d. Sodium bicarbonate

a. Mannitol For increased ICP, mannitol, an osmotic diuretic, administered intravenously, is the drug used most frequently for rapid reduction. Epinephrine, atropine sulfate, and sodium bicarbonate are not used to decrease ICP.

Which statement best explains why iron deficiency anemia is common during toddlerhood? a. Milk is a poor source of iron. b. Iron cannot be stored during fetal development. c. Fetal iron stores are depleted by age 1 month. d. Dietary iron cannot be started until age 12 months.

a. Milk is a poor source of iron. Children between the ages of 12 and 36 months are at risk for anemia because cow's milk is a major component of their diet, and it is a poor source of iron. Iron is stored during fetal development, but the amount stored depends on maternal iron stores. Fetal iron stores are usually depleted by age 5 to 6 months. Dietary iron can be introduced by breastfeeding, iron-fortified formula, and cereals during the first 12 months of life.

When caring for the child diagnosed with Reye's syndrome, what is the priority nursing intervention? a. Monitor intake and output b. Prevent skin breakdown c. Observe for petechiae d. Do range-of-motion (ROM) exercises

a. Monitor intake and output Accurate and frequent monitoring of intake and output is essential for adjusting fluid volumes to prevent both dehydration and cerebral edema. Preventing skin breakdown, observing for petechiae, and doing ROM exercises are important interventions in the care of a critically ill or comatose child. Careful monitoring of intake and output is a priority.

A 14 year old is in the intensive care unit after a spinal cord injury 2 days ago. Which nursing care interventions are needed for this child? (Select all that apply.) a. Monitoring and maintaining systemic blood pressure. b. Administering corticosteroids. c. Minimizing environmental stimuli. d. Discussing long-term care issues with the family. e. Monitoring for respiratory complications.

a. Monitoring and maintaining systemic blood pressure. b. Administering corticosteroids. e. Monitoring for respiratory complications. Spinal cord injury patients are physiologically labile, and close monitoring of blood pressure and respirations is required. They may be unstable for the first few weeks after the injury. Corticosteroids are administered to minimize the inflammation present with the injury. Minimizing environmental stimuli and discussing long-term care issues with the family do not apply to providing care for this patient.

What is the most common problem for children born with a myelomeningocele? a. Neurogenic bladder b. Intellectual impairment c. Respiratory compromise d. Cranioschisis

a. Neurogenic bladder Myelomeningocele is one of the most common causes of neuropathic (neurogenic) bladder dysfunction among children. Risk of intellectual impairment is minimized through early intervention and management of hydrocephalus. Respiratory compromise is not a common problem in myelomeningocele. Cranioschisis is a skull defect through which various tissues protrude. It is not associated with myelomeningocele.

An infant diagnosed with hydrocephalus is hospitalized for surgical placement of a ventriculoperitoneal shunt. Which interventions should be included in the child's postoperative care? (Select all that apply.) a. Observe closely for signs of infection b. Pump the shunt reservoir to maintain patency c. Administer sedation to decrease irritability d. Maintain Trendelenburg position to decrease pressure on the shunt e. Maintain an accurate record of intake and output f. Monitor for abdominal distention

a. Observe closely for signs of infection e. Maintain an accurate record of intake and output f. Monitor for abdominal distention Infection is a major complication of ventriculoperitoneal shunts. Observation for signs of infection is a priority nursing intervention. Intake and output should be measured carefully. Abdominal distention could be a sign of peritonitis or a postoperative ileus. Pumping the shunt reservoir, administering sedation, and maintaining Trendelenburg position are not interventions associated with this condition.

The nurse is caring for an infant diagnosed with congestive heart disease (CHD). The nurse should plan which intervention to decrease cardiac demands? a. Organize nursing activities to allow for uninterrupted sleep b. Allow the infant to sleep through feedings during the night c. Wait for the infant to cry to show definite signs of hunger d. Discourage parents from rocking the infant

a. Organize nursing activities to allow for uninterrupted sleep The infant requires rest and conservation of energy for feeding. Every effort is made to organize nursing activities to allow for uninterrupted periods of sleep. Whenever possible, parents are encouraged to stay with their infant to provide the holding, rocking, and cuddling that help children sleep more soundly. To minimize disturbing the infant, changing bed linens and complete bathing are done only when necessary. Feeding is planned to accommodate the infant's sleep and wake patterns. The child is fed at the first sign of hunger, such as when sucking on fists, rather than waiting until he or she cries for a bottle because the stress of crying exhausts the limited energy supply. Because infants with CHD tire easily and may sleep through feedings, smaller feedings every 3 hours may be helpful.

What term is used to describe the painful, tender, pea-sized nodules that may appear on the pads of the fingers or toes in cases of bacterial endocarditis? a. Osler's nodes b. Janeway lesions c. Subcutaneous nodules d. Aschoff's nodules

a. Osler's nodes Osler's nodes are red, painful, intradermal nodes found on pads of the phalanges in bacterial endocarditis. Janeway lesions are painless hemorrhagic areas on palms and soles in bacterial endocarditis. Subcutaneous nodules are nontender swellings located over bony prominences, commonly found in rheumatic fever. Aschoff's nodules are small nodules composed of cells and leukocytes found in the interstitial tissues of the heart in rheumatic myocarditis.

What is the most common mode of transmission of human immunodeficiency virus (HIV) in the pediatric population? a. Perinatal transmission b. Sexual abuse c. Blood transfusions d. Poor hand washing

a. Perinatal transmission Infected women can transmit the virus to their infants across the placenta during pregnancy, at delivery, and through breastfeeding. Cases of HIV infection from sexual abuse have been reported; however, perinatal transmission accounts for most pediatric HIV infections. In the past some children became infected with HIV through blood transfusions; however, improved laboratory screening has significantly reduced the probability of contracting HIV from blood products. Poor hand washing is not an etiology of HIV infection.

What is a major clinical manifestation of rheumatic fever? a. Polyarthritis b. Osler's nodes c. Janeway spots d. Splinter hemorrhages of distal third of nails

a. Polyarthritis Polyarthritis is swollen, hot, red, and painful joints. The affected joints will change every 1 to 2 days. Primarily the large joints are affected. Osler's nodes, Janeway spots, and splinter hemorrhages are characteristic of infective endocarditis.

A child with pulmonary atresia exhibits cyanosis with feeding. On reviewing this child's laboratory values, the nurse is not surprised to notice which abnormality? a. Polycythemia b. Infection c. Dehydration d. Anemia

a. Polycythemia Polycythemia is a compensatory response to chronic hypoxia. The body attempts to improve tissue oxygenation by producing additional red blood cells and thereby increases the oxygen-carrying capacity of the blood. Infection is not a clinical consequence of cyanosis. Although dehydration can occur in cyanotic heart disease, it is not a compensatory mechanism for chronic hypoxia. It is not a clinical consequence of cyanosis. Anemia may develop as a result of increased blood viscosity. Anemia is not a clinical consequence of cyanosis.

Which structural defects constitute tetralogy of Fallot? a. Pulmonic stenosis, ventricular septal defect, overriding aorta, right ventricular hypertrophy b. Aortic stenosis, ventricular septal defect, overriding aorta, right ventricular hypertrophy c. Aortic stenosis, atrial septal defect, overriding aorta, left ventricular hypertrophy d. Pulmonic stenosis, ventricular septal defect, aortic hypertrophy, left ventricular hypertrophy

a. Pulmonic stenosis, ventricular septal defect, overriding aorta, right ventricular hypertrophy Tetralogy of Fallot has these four characteristics: pulmonary stenosis, ventricular septal defect, overriding aorta, and right ventricular hypertrophy. There is pulmonic stenosis but not aortic stenosis in tetralogy of Fallot. Right ventricular hypertrophy, not left ventricular hypertrophy, is present in tetralogy of Fallot. There is a ventricular septal defect, not an atrial septal defect, and overriding aorta, not aortic hypertrophy, is present.

A young boy has just been diagnosed with pseudohypertrophic muscular dystrophy. The management plan should include which intervention? a. Recommending genetic counseling b. Explaining that the disease is easily treated c. Suggesting ways to limit the use of muscles d. Assisting the family in finding a nursing facility to provide his care

a. Recommending genetic counseling Pseudohypertrophic (Duchenne's) muscular dystrophy is inherited as an X-linked recessive gene. Genetic counseling is recommended for parents, female siblings, maternal aunts, and their female offspring. No effective treatment exists at this time for childhood muscular dystrophy. Maintaining optimal function of all muscles for as long as possible is the primary goal. It has been found that children who remain as active as possible are able to avoid wheelchair confinement for a longer time. Assisting the family in finding a nursing facility is inappropriate at the time of diagnosis. When the child becomes increasingly incapacitated, the family may consider home-based care, a skilled nursing facility, or respite care to provide the necessary care.

The nurse is caring for an infant with myelomeningocele who is scheduled for surgical repair in the morning. Which early signs of infection should the nurse monitor on this infant? (Select all that apply.) a. Temperature instability b. Irritability c. Lethargy d. Bradycardia e. Hypertension

a. Temperature instability b. Irritability c. Lethargy The nurse should observe an infant with unrepaired myelomeningocele for early signs of infection, such as temperature instability (axillary), irritability, and lethargy. Bradycardia and hypertension are not early signs of infection in infants.

Parents of a 3-year-old child diagnosed with congenital heart disease are afraid to let their child play with other children because of possible overexertion. The nurse's reply should be based on what knowledge? a. The child needs opportunities to play with peers. b. The child needs to understand that peers' activities are too strenuous. c. Parents can meet all the child's needs. d. Constant parental supervision is needed to avoid overexertion.

a. The child needs opportunities to play with peers. The child needs opportunities for social development. Children usually limit their activities if allowed to set their own pace and regulate their activities. The child will limit activities as necessary. Parents must be encouraged to seek appropriate social activities for the child, especially before kindergarten. The child needs to have activities that foster independence.

How much folic acid is recommended for women of childbearing age? a. 0.1 mg b. 0.4 mg c. 1.5 mg d. 2 mg

b. 0.4 mg It has been estimated that a daily intake of 0.4 mg of folic acid in women of childbearing age will prevent 50% to 70% of cases of neural tube defects. A dose of 0.1 mg is too low, and 1.5 mg and 2 mg are not recommended dosages of folic acid.

Which intervention should be included in the plan of care for an infant with the nursing diagnosis of Excess Fluid Volume related to congestive heart failure? a. Weigh the infant every day on the same scale at the same time. b. Notify the physician when weight gain exceeds more than 20 g/day. c. Put the infant in a car seat to minimize movement. d. Administer digoxin as ordered by the physician.

a. Weigh the infant every day on the same scale at the same time. Excess fluid volume may not be overtly visible. Weight changes may indicate fluid retention. Weighing the infant on the same scale at the same time each day ensures consistency. An excessive weight gain for an infant is an increase of more than 50 g/day. With fluid volume excess, skin will be edematous. The infant's position should be changed frequently to prevent undesirable pooling of fluid in certain areas. Digoxin is used in the treatment of congestive heart failure to improve cardiac function. Diuretics will help the body get rid of excess fluid.

The nurse is preparing an adolescent for discharge after a cardiac catheterization. Which statement by the adolescent would indicate a need for further teaching? a. "I should avoid tub baths but may shower." b. "I have to stay on strict bed rest for 3 days." c. "I should remove the pressure dressing the day after the procedure." d. "I may attend school but should avoid exercise for several days."

b. "I have to stay on strict bed rest for 3 days." Encourage rest and quiet activities for the first 3 days and avoid strenuous exercise. The child does not need to be on strict bed rest for 3 days. Showers are recommended; children should avoid a tub bath. The pressure dressing is removed the day after the catheterization and replaced by an adhesive bandage to keep the area clean.

The nurse is preparing a school-age child for a computed tomography (CT) scan to assess cerebral function. When preparing the child for the scan, which statement should the nurse include? a. "Pain medication will be given." b. "The scan will not hurt." c. "You will be able to move once the equipment is in place." d. "Unfortunately no one can remain in the room with you during the test."

b. "The scan will not hurt." For CT scans, the child will not be allowed to move and must be immobilized. It is important to emphasize to the child that at no time is the procedure painful. Pain medication is not required; however, sedation is sometimes necessary. Someone is able to remain with the child during the procedure.

The nurse has received report on four children. Which child should the nurse assess first? a. A school-age child in a coma with stable vital signs. b. A preschool child with a head injury and decreasing level of consciousness. c. An adolescent admitted after a motor vehicle accident who is oriented to person and place. d. A toddler in a persistent vegetative state with a low-grade fever.

b. A preschool child with a head injury and decreasing level of consciousness. The nurse should assess the child with a head injury and decreasing level of consciousness (LOC) first. Assessment of LOC remains the earliest indicator of improvement or deterioration in neurologic status. The next child the nurse should assess is a toddler in a persistent vegetative state with a low-grade fever. The school-age child in a coma with stable vital signs and the adolescent admitted to the hospital who is oriented to his or her surroundings would be of least worry to the nurse.

The nurse is monitoring an infant for signs of increased intracranial pressure (ICP). Which are late signs of increased ICP in an infant? (Select all that apply.) a. Tachycardia b. Alteration in pupil size and reactivity c. Increased motor response d. Extension or flexion posturing e. Cheyne-Stokes respirations

b. Alteration in pupil size and reactivity d. Extension or flexion posturing e. Cheyne-Stokes respirations Late signs of ICP in an infant or child include bradycardia, alteration in pupil size and reactivity, decreased motor response, extension or flexion posturing, and Cheyne-Stokes respirations.

The nurse is planning care for a school-age child admitted to the hospital with hemophilia. Which interventions should the nurse plan to implement for this child? (Select all that apply.) a. Fingersticks for blood work instead of venipunctures b. Avoidance of intramuscular (IM) injections c. Acetaminophen for mild pain control d. Soft toothbrush for dental hygiene e. Administration of packed red blood cells

b. Avoidance of intramuscular (IM) injections c. Acetaminophen for mild pain control d. Soft toothbrush for dental hygiene Nurses should take special precautions when caring for a child with hemophilia to prevent the use of procedures that may cause bleeding, such as IM injections. The subcutaneous route is substituted for IM injections whenever possible. Venipunctures for blood samples are usually preferred for these children. There is usually less bleeding after the venipuncture than after finger or heel punctures. Neither aspirin nor any aspirin-containing compound should be used. Acetaminophen is a suitable aspirin substitute, especially for controlling mild pain. A soft toothbrush is recommended for dental hygiene to prevent bleeding from the gums. Packed red blood cells are not administered. The primary therapy for hemophilia is replacement of the missing clotting factor. The products available are factor VIII concentrates.

What is one of the most frequent causes of hypovolemic shock in children? a. Myocardial infarction b. Blood loss c. Anaphylaxis d. Congenital heart disease

b. Blood loss Blood loss and extracellular fluid loss are two of the most frequent causes of hypovolemic shock in children. Myocardial infarction is rare in a child; if it occurred, the resulting shock would be cardiogenic, not hypovolemic. Anaphylaxis results in distributive shock from extreme allergy or hypersensitivity to a foreign substance. Congenital heart disease tends to contribute to hypervolemia, not hypovolemia.

What is the primary nursing intervention necessary to prevent bacterial endocarditis? a. Institute measures to prevent dental procedures. b. Counsel parents of high risk children about prophylactic antibiotics. c. Observe children for complications such as embolism and heart failure. d. Encourage restricted mobility in susceptible children.

b. Counsel parents of high risk children about prophylactic antibiotics. The objective of nursing care is to counsel the parents of high risk children about both the need for prophylactic antibiotics for dental procedures and the necessity of maintaining excellent oral health. The child's dentist should be aware of the child's cardiac condition. Dental procedures should be done to maintain a high level of oral health. Prophylactic antibiotics are necessary. Observing for complications and encouraging restricted mobility in susceptible children should be done, but maintaining good oral health and using prophylactic antibiotics are most important.

A young child with human immunodeficiency virus is receiving several antiretroviral drugs. What is the expected outcome of these drug therapies? a. Cure the disease b. Delay disease progression c. Prevent spread of disease d. Treat Pneumocystis jiroveci pneumonia

b. Delay disease progression Although not a cure, these antiviral drugs can suppress viral replication, preventing further deterioration of the immune system, and delay disease progression. At this time cure is not possible. These drugs do not prevent the spread of the disease. Pneumocystis jiroveci prophylaxis is accomplished with antibiotics.

A school-age child has sustained a head injury and multiple fractures after being thrown from a horse. The child's level of consciousness is variable. The parents tell the nurse that they think their child is in pain because of periodic crying and restlessness. What is the most appropriate nursing action in response to the parent's concern? a. Discuss with parents the child's previous experiences with pain. b. Discuss with practitioner what analgesia can be safely administered. c. Explain that analgesia is contraindicated with a head injury. d. Explain that analgesia is unnecessary when child is not fully awake and alert.

b. Discuss with practitioner what analgesia can be safely administered. A key nursing role is to provide sedation and analgesia for the child. Consultation with the appropriate practitioner is necessary to avoid conflict between the necessity to monitor the child's neurologic status and to promote comfort and relieve anxiety. Gathering information about the child's previous experiences with pain should be obtained as part of the assessment, but because of the severity of injury, analgesia should be provided as soon as possible. Analgesia can be used safely in individuals who have sustained head injuries and can decrease anxiety and resultant increased intracranial pressure.

Chelation therapy is begun on a child with â-thalassemia major with what expected result? a. Treatment of the disease. b. Elimination of excess iron. c. Decreasing the risk of hypoxia. d. Managing nausea and vomiting.

b. Elimination of excess iron. A complication of the frequent blood transfusions in thalassemia is iron overload. Chelation therapy with deferoxamine (an iron-chelating agent) is given with oral supplements of vitamin C to increase iron excretion. Chelation therapy treats the side effects of disease management. Decreasing the risk of hypoxia and managing nausea and vomiting are not the purposes of chelation therapy.

The Glasgow Coma Scale consists of an assessment of what functions? a. Pupil reactivity and motor response. b. Eye opening and verbal and motor responses. c. Level of consciousness and verbal response. d. Intracranial pressure (ICP) and level of consciousness.

b. Eye opening and verbal and motor responses. The Glasgow Coma Scale assesses eye opening and verbal and motor responses. Pupil reactivity is not a part of the Glasgow Coma Scale but is included in the pediatric coma scale. Level of consciousness and ICP are not part of the Glasgow Coma Scale.

Which clinical changes occur as a result of septic shock? a. Hypothermia b. Increased cardiac output c. Vasoconstriction d. Angioneurotic edema

b. Increased cardiac output Increased cardiac output, which results in warm, flushed skin, is one of the manifestations of septic shock. Fever and chills are characteristic of septic shock. Vasodilation is more common in septic shock. Angioneurotic edema occurs as a manifestation in anaphylactic shock.

What is the common vector reservoir for agents causing viral encephalitis in the United States? a. Tarantula spiders b. Mosquitoes c. Carnivorous wild animals d. Domestic and wild animals

b. Mosquitoes Viral encephalitis, not attributable to a childhood viral disease, is usually transmitted by mosquitoes and ticks. The vector reservoir for most agents pathogenic for humans and detected in the United States are mosquitoes and ticks; therefore, most cases of encephalitis appear during the hot summer months. Tarantulas, carnivorous wild animals, and domestic animals are not reservoirs for the agents that cause viral encephalitis.

The parents of a child hospitalized with sickle cell anemia tell the nurse that they are concerned about narcotic analgesics causing addiction. The nurse should provide what explanation in response to the parent's concern? a. Narcotics are often ordered but not usually needed. b. Narcotics rarely cause addiction when they are medically indicated. c. Narcotics are given as a last resort because of the threat of addiction. d. Narcotics are used only if other measures such as ice packs are ineffective.

b. Narcotics rarely cause addiction when they are medically indicated. The pain of sickle cell anemia is best treated by a multidisciplinary approach. Mild-to-moderate pain can be controlled by ibuprofen and acetaminophen. When narcotics are indicated, they are titrated to effect and given around the clock. Patient-controlled analgesia reinforces the patient's role and responsibility in managing the pain and provides flexibility in dealing with pain. Few if any patients who receive opioids for severe pain become behaviorally addicted to the drug. Narcotics are often used because of the severe nature of the pain of vaso-occlusive crisis. Ice is contraindicated because of its vasoconstrictive effects.

The nurse, closely monitoring a child who is unconscious after a fall, notices that the child suddenly has a fixed and dilated pupil. The nurse should interpret this as the indication of what occurrence? a. Eye trauma b. Neurosurgical emergency c. Severe brainstem damage d. Indication of brain death

b. Neurosurgical emergency The sudden appearance of a fixed and dilated pupil(s) is a neurosurgical emergency. The nurse should immediately report this finding. Although a dilated pupil may be associated with eye trauma, this child has experienced a neurologic insult. Pinpoint pupils or fixed, bilateral pupils for more than 5 minutes are indicative of brainstem damage. The unilateral fixed and dilated pupil is suggestive of damage on the same side of the brain. One fixed and dilated pupil is not suggestive of brain death.

Which should the nurse teach about prevention of sickle cell crises to parents of a preschool child with sickle cell disease? (Select all that apply.) a. Limit fluids at bedtime. b. Notify the health care provider if a fever of 38.5° C (101.3° F) or greater occurs. c. Give penicillin as prescribed. d. Use ice packs to decrease the discomfort of vaso-occlusive pain in the legs. e. Notify the health care provider if your child begins to develop symptoms of cold.

b. Notify the health care provider if a fever of 38.5° C (101.3° F) or greater occurs. c. Give penicillin as prescribed. e. Notify the health care provider if your child begins to develop symptoms of cold. The most important issues to teach the family of a child with sickle cell anemia are to (1) seek early intervention for problems, such as a fever of 38.5° C (101.3° F) or greater; (2) give penicillin as ordered; (3) recognize signs and symptoms of splenic sequestration, as well as respiratory problems that can lead to hypoxia; and (4) treat the child normally. The nurse emphasizes the importance of adequate hydration to prevent sickling and to delay the adhesion-stasis-thrombosis-ischemia cycle. It is not sufficient to advise parents to "force fluids" or "encourage drinking." They need specific instructions on how many daily glasses or bottles of fluid are required. Many foods are also a source of fluid, particularly soups, flavored ice pops, ice cream, sherbet, gelatin, and puddings. Increased fluids combined with impaired kidney function result in the problem of enuresis. Parents who are unaware of this fact frequently use the usual measures to discourage bed-wetting, such as limiting fluids at night. Enuresis is treated as a complication of the disease, such as joint pain or some other symptom, to alleviate parental pressure on the child. Ice should not be used during a vaso-occlusive pain crisis because it vasoconstricts and impairs circulation even more.

The nurse assessing a premature newborn infant auscultates a continuous machinery-like murmur. This finding is associated with which congenital heart defect? a. Pulmonary stenosis b. Patent ductus arteriosus c. Ventricular septal defect d. Coarctation of the aorta

b. Patent ductus arteriosus The classic murmur associated with patent ductus arteriosus is a machinery-like one that can be heard throughout both systole and diastole. A systolic ejection murmur that may be accompanied by a palpable thrill is a manifestation of pulmonary stenosis. The characteristic murmur associated with ventricular septal defect is a loud, harsh, holosystolic murmur. A systolic murmur that is accompanied by an ejection click may be heard on auscultation when coarctation of the aorta is present.

An 8-month-old infant becomes hypercyanotic while blood is being drawn. What should be the nurse's first action? a. Assess for neurologic defects b. Place the child in the knee-chest position c. Begin cardiopulmonary resuscitation d. Prepare the family for imminent death

b. Place the child in the knee-chest position The first action is to place the infant in the knee-chest position. Blow-by oxygen may be indicated. Neurologic defects are unlikely. The child should be assessed for airway, breathing, and circulation. Often calming the child and administering oxygen and morphine can alleviate the hypercyanotic spell; cardiopulmonary resuscitation is not necessary, and death is unlikely.

As part of the treatment for congestive heart failure, the child takes the diuretic furosemide. As part of teaching home care, the nurse encourages the family to give the child foods such as bananas, oranges, and leafy vegetables. These foods are recommended because they are high in what electrolyte? a. Chlorides b. Potassium c. Sodium d. Zinc

b. Potassium Diuretics that work on the proximal and distal renal tubules contribute to increased losses of potassium. The child's diet should be supplemented with potassium.

The nurse is caring for a child with persistent hypoxia secondary to a cardiac defect. The nurse recognizes that a risk of cerebrovascular accidents (strokes) exists. An important objective to decrease this risk is to achieve what result? a. Minimize seizures b. Prevent dehydration c. Promote cardiac output d. Reduce energy expenditure

b. Prevent dehydration In children with persistent hypoxia, polycythemia develops. Dehydration must be prevented in hypoxemic children because it potentiates the risk of strokes. Minimizing seizures, promoting cardiac output, and reducing energy expenditure will not reduce the risk of cerebrovascular accidents.

What is the leading cause of death after heart transplantation? a. Infection b. Rejection c. Cardiomyopathy d. Congestive heart failure

b. Rejection The posttransplantation course is complex. The leading cause of death after cardiac transplantation is rejection. Infection is a continued risk secondary to the immunosuppression necessary to prevent rejection. Cardiomyopathy is one of the indications for cardiac transplant. Congestive heart failure is not a leading cause of death.

The nurse is caring for a child with aplastic anemia. Which nursing diagnoses are appropriate? (Select all that apply.) a. Acute Pain related to vaso-occlusion b. Risk for Infection related to inadequate secondary defenses or immunosuppression c. Ineffective Protection related to thrombocytopenia d. Ineffective Tissue Perfusion related to anemia e. Ineffective Protection related to abnormal clotting

b. Risk for Infection related to inadequate secondary defenses or immunosuppression c. Ineffective Protection related to thrombocytopenia d. Ineffective Tissue Perfusion related to anemia These are appropriate nursing diagnosis for the nurse planning care for a child with aplastic anemia. Aplastic anemia is a condition in which the bone marrow ceases production of the cells it normally manufactures, resulting in pancytopenia. The child will have varying degrees of the disease depending on how low the values are for absolute neutrophil count (affecting the body's response to infection), platelet count (putting the child at risk for bleeding), and absolute reticulocyte count (causing the child to have anemia). Acute Pain related to vaso-occlusion is an appropriate nursing diagnosis for sickle cell anemia for the child in vaso-occlusive crisis, but it is not applicable to a child with aplastic anemia. Ineffective Protection related to abnormal clotting is an appropriate diagnosis for a child with hemophilia.

What term is used to identify the condition in which the normal adult hemoglobin is partly or completely replaced by abnormal hemoglobin? a. Aplastic anemia b. Sickle cell anemia c. Thalassemia major d. Iron deficiency anemia

b. Sickle cell anemia Sickle cell anemia is one of a group of diseases collectively called hemoglobinopathies, in which normal adult hemoglobin is replaced by abnormal hemoglobin. Aplastic anemia is a lack of cellular elements being produced. Hemophilia refers to a group of bleeding disorders in which there is deficiency of one of the factors necessary for coagulation. Iron deficiency anemia affects size and depth of color of hemoglobin and does not involve abnormal hemoglobin.

Which term is used when a patient remains in a deep sleep, responsive only to vigorous and repeated stimulation? a. Coma b. Stupor c. Obtundation d. Persistent vegetative state

b. Stupor Stupor exists when the child remains in a deep sleep, responsive only to vigorous and repeated stimulation. Coma is the state in which no motor or verbal response occurs to noxious (painful) stimuli. Obtundation describes a level of consciousness in which the child can be aroused with stimulation. Persistent vegetative state describes the permanent loss of function of the cerebral cortex.

When taking the history of a child hospitalized with Reye's syndrome, the nurse should not be surprised that a week ago the child had recovered from infectious illness? a. Measles b. Varicella c. Meningitis d. Hepatitis

b. Varicella Most cases of Reye's syndrome follow a common viral illness such as varicella or influenza. Measles, meningitis, and hepatitis are not associated with Reye's syndrome

Which immunization should be given with caution to children infected with human immunodeficiency virus? a. Influenza b. Varicella c. Pneumococcus d. Inactivated poliovirus

b. Varicella The children should be carefully evaluated before giving live viral vaccines such as varicella, measles, mumps, and rubella. The child must be immunocompetent and not have contact with other severely immunocompromised individuals. Influenza, pneumococcus, and inactivated poliovirus are not live vaccines.

What is a common sign of digoxin toxicity? a. Seizures b. Vomiting c. Bradypnea d. Tachycardia

b. Vomiting Vomiting is a common sign of digoxin toxicity. Seizures are not associated with digoxin toxicity. The child will have a slower heart rate, not respiratory rate.

The nurse is caring for a neonate born with a myelomeningocele. Surgery to repair the defect is scheduled the next day. The most appropriate way to position and feed this neonate is to place him: a. prone and tube feed. b. prone, turn head to side, and nipple feed. c. supine in infant carrier and nipple feed. d. supine, with defect supported with rolled blankets, and nipple feed.

b. prone, turn head to side, and nipple feed. In the prone position, feeding is a problem. The infant's head is turned to one side for feeding. If the child is able to nipple feed, no indication is present for tube feeding. Before surgery, the infant is kept in the prone position to minimize tension on the sac and risk of trauma.

A 5 year old sustained a concussion when falling out of a tree. In preparation for discharge, the nurse is discussing home care with the mother. Which statement made by the mother indicates a correct understanding of the teaching? a. "I should expect my child to have a few episodes of vomiting." b. "If I notice sleep disturbances, I should contact the physician immediately." c. "I should expect my child to have some difficulty concentrating for a while." d. "If I notice diplopia, I will have my child rest for 1 hour."

c. "I should expect my child to have some difficulty concentrating for a while." The parents are advised of probably posttraumatic symptoms that may be expected, including difficulty concentrating, and memory impairment. If the child has episodes of vomiting, sleep disturbances, or diplopia, they should be immediately reported for evaluation.

A 3-year-old child is hospitalized after a near-drowning accident. The child's mother complains to the nurse, "This seems unnecessary when he is perfectly fine." What is the nurse's best reply? a. "He still needs a little extra oxygen." b. "I'm sure he is fine, but the doctor wants to make sure." c. "The reason for this is that complications could still occur." d. "It is important to observe for possible central nervous system problems."

c. "The reason for this is that complications could still occur." All children who have a near-drowning experience should be admitted to the hospital for observation. Although many children do not appear to have suffered adverse effects from the event, complications such as respiratory compromise and cerebral edema may occur up to 24 hours after the incident. Aspiration pneumonia is a frequent complication that occurs about 48 to 72 hours after the episode. Stating that, "He still needs a little extra oxygen" does not respond directly to the mother's concern. Why is her child still receiving oxygen? The nurse should clarify that different complications can occur up to 24 hours later and that observations are necessary. The nurse should not provide statements that provide unfounded information, like "I'm sure he is fine."

A preschool child is scheduled for an echocardiogram. Parents ask the nurse whether they can hold the child during the procedure. The nurse should answer with which response? a. "You will be able to hold your child during the procedure." b. "Your child can be active during the procedure, but can't sit in your lap." c. "Your child must lie quietly; sometimes a mild sedative is administered before the procedure." d. "The procedure is invasive so your child will be restrained during the echocardiogram."

c. "Your child must lie quietly; sometimes a mild sedative is administered before the procedure." Although an echocardiogram is noninvasive, painless, and associated with no known side effects, it can be stressful for children. The child must lie quietly in the standard echocardiographic positions; crying, nursing, being held, or sitting up often leads to diagnostic errors or omissions. Therefore, infants and young children may need a mild sedative; older children benefit from psychologic preparation for the test. The distraction of a video or movie is often helpful.

A child has been seizure-free for 2 years. A father asks the nurse how much longer the child will need to take the antiseizure medications. The nurse includes which intervention in the response? a. Medications can be discontinued at this time. b. The child will need to take the drugs for 5 years after the last seizure. c. A stepwise approach will be used to reduce the dosage gradually. d. Seizure disorders are a lifelong problem. Medications cannot be discontinued.

c. A stepwise approach will be used to reduce the dosage gradually. A predesigned protocol is used to wean a child gradually off antiseizure medications, usually when the child is seizure-free for 2 years and has a normal electroencephalogram. Medications must be gradually reduced to minimize the recurrence of seizures. Seizure medications can be safely discontinued. The risk of recurrence is greatest within the first year.

Which condition is caused by a virus that primarily infects a specific subset of T lymphocytes, the CD4+ T-cells? a. Wiskott-Aldrich syndrome b. Idiopathic thrombocytopenic purpura (ITP) c. Acquired immunodeficiency syndrome (AIDS) d. Severe combined immunodeficiency disease

c. Acquired immunodeficiency syndrome (AIDS) AIDS is caused by the human immunodeficiency virus, which primarily attacks the CD4+ T-cells. Wiskott-Aldrich syndrome, ITP, and severe combined immunodeficiency disease are not viral illnesses.

When teaching the mother of a 9-month-old infant about administering liquid iron preparations, the nurse should include that information? a. They should be given with meals. b. They should be stopped immediately if nausea and vomiting occur. c. Adequate dosage will turn the stools a tarry green color. d. Preparation should be allowed to mix with saliva and bathe the teeth before swallowing.

c. Adequate dosage will turn the stools a tarry green color. The nurse should prepare the mother for the anticipated change in the child's stools. If the iron dose is adequate, the stools will become a tarry green color. The lack of the color change may indicate insufficient iron. The iron should be given in two divided doses between meals, when the presence of free hydrochloric acid is greatest. Iron is absorbed best in an acidic environment. Vomiting and diarrhea may occur with iron administration. If these occur, the iron should be given with meals, and the dosage reduced and gradually increased as the child develops tolerance. Liquid preparations of iron stain the teeth. They should be administered through a straw, and the mouth rinsed after administration.

What intervention should be implemented prior to the removal of a child's chest tubes? a. Explain that it is not painful. b. Explain that only a Band-Aid will be needed. c. Administer analgesics before the procedure. d. Educate the patient to expect bright red drainage for several hours after removal.

c. Administer analgesics before the procedure. It is appropriate to prepare the child for the removal of chest tubes with analgesics. Short-acting medications can be used that are administered through an existing intravenous line. It is not a pain-free procedure. A sharp, momentary pain is felt, and this should not be misrepresented to the child. A petroleum gauze/airtight dressing is needed. Little or no drainage should be found on removal.

Nursing interventions for the child after a cardiac catheterization include which of the following? (Select all that apply.) a. Allow ambulation as tolerated. b. Monitor vital signs every 2 hours. c. Assess the affected extremity for temperature and color. d. Check pulses above the catheterization site for equality and symmetry. e. Remove pressure dressing after 4 hours. f. Maintain a patent peripheral intravenous catheter until discharge.

c. Assess the affected extremity for temperature and color. f. Maintain a patent peripheral intravenous catheter until discharge. The extremity that was used for access for the cardiac catheterization must be checked for temperature and color. Coolness and blanching may indicate arterial occlusion. The child should have a patent peripheral intravenous line to ensure adequate hydration. Allowing ambulation, monitoring vital signs every 2 hours, checking pulses, and removing the pressure dressing after 4 hours are interventions that do not apply to a child after a cardiac catheterization.

Which neurologic diagnostic test gives a visualized horizontal and vertical cross section of the brain at any axis? a. Nuclear brain scan b. Echoencephalography c. Computed tomography (CT) scan d. Magnetic resonance imaging (MRI)

c. Computed tomography (CT) scan A CT scan provides visualization of the horizontal and vertical cross sections of the brain at any axis. A nuclear brain scan uses a radioisotope that accumulates where the blood-brain barrier is defective. Echoencephalography identifies shifts in midline structures of the brain as a result of intracranial lesions. MRI permits visualization of morphologic features of target structures and tissue discrimination that is unavailable with any other techniques.

Which clinical manifestations would the nurse expect to see as shock progresses in a child and decompensated develops? (Select all that apply.) a. Thirst and diminished urinary output b. Irritability and apprehension c. Cool extremities and decreased skin turgor d. Confusion and somnolence e. Normal blood pressure and narrowing pulse pressure f. Tachypnea and poor capillary refill time

c. Cool extremities and decreased skin turgor d. Confusion and somnolence f. Tachypnea and poor capillary refill time Cool extremities, decreased skin turgor, confusion, somnolence, tachypnea, and poor capillary refill time are beginning signs of decompensated shock.

A child is brought to the emergency department experiencing an anaphylactic reaction to a bee sting. While an airway is being established, what medication should the nurse prepare for immediate administration? a. Diphenhydramine b. Dopamine c. Epinephrine d. Calcium chloride

c. Epinephrine After the first priority of establishing an airway, epinephrine is the drug of choice. Diphenhydramine is not a strong enough antihistamine for this severe a reaction. Dopamine and calcium chloride are not appropriate drugs for this type of reaction.

The parents of a child diagnosed with aplastic anemia tell the nurse that a bone marrow transplant (BMT) may be necessary. What should the nurse recognize as important when discussing this with the family? a. BMT should be done at time of diagnosis. b. Parents and siblings of child have a 25% chance of being a suitable donor. c. Finding a suitable donor involves matching antigens from the human leukocyte antigen (HLA) system. d. If BMT fails, chemotherapy or radiotherapy must be continued.

c. Finding a suitable donor involves matching antigens from the human leukocyte antigen (HLA) system. The most successful BMTs come from suitable HLA-matched donors. The timing of a BMT depends on the disease process involved. It usually follows intensive high-dose chemotherapy and/or radiation therapy. Usually parents only share approximately 50% of the genetic material with their children. A one-in-four chance exists that two siblings will have two identical haplotypes and will be identically matched at the HLA loci. Discussing the continuation of chemotherapy or radiotherapy is not appropriate when planning the BMT. That decision will be made later.

A current recommendation to prevent neural tube defects is the administration of what supplement? a. Vitamin A throughout pregnancy b. Multivitamin preparations as soon as pregnancy is suspected c. Folic acid for all women of childbearing age d. Folic acid during the first and second trimesters of pregnancy

c. Folic acid for all women of childbearing age The widespread use of folic acid among women of childbearing age is expected to decrease the incidence of spina bifida significantly. Vitamin A and multivitamin preparations do not have a relation to the prevention of spina bifida. Folic acid supplementation is recommended for the preconceptual period and during the pregnancy. Only 42% of women actually follow these guidelines.

Which type of seizure involves both hemispheres of the brain? a. Focal b. Partial c. Generalized d. Acquired

c. Generalized Clinical observations of generalized seizures indicate that the initial involvement is from both hemispheres. Focal seizures may arise from any area of the cerebral cortex, but the frontal, temporal, and parietal lobes are most commonly affected. Partial seizures are caused by abnormal electrical discharges from epileptogenic foci limited to a circumscribed region of the cerebral cortex. A seizure disorder that is acquired is a result of a brain injury from a variety of factors; it does not specify the type of seizure.

Which term is used to describe a clinical manifestation of the systemic venous congestion that can occur with congestive heart failure? a. Tachypnea b. Tachycardia c. Peripheral edema d. Pale, cool extremities

c. Peripheral edema Peripheral edema, especially periorbital edema, is a clinical manifestation of systemic venous congestion. Tachypnea is a manifestation of pulmonary congestion. Tachycardia and pale, cool extremities are clinical manifestations of impaired myocardial function.

What is best described as the inability of the heart to pump an adequate amount of blood to the systemic circulation at normal filling pressures? a. Pulmonary congestion b. Congenital heart defect c. Heart failure d. Systemic venous congestion

c. Heart failure The definition of heart failure is the inability of the heart to pump an adequate amount of blood to the systemic circulation at normal filling pressures to meet the metabolic demands of the body. Pulmonary congestion is an excessive accumulation of fluid in the lungs. Congenital heart defect is a malformation of the heart present at birth. Systemic venous congestion is an excessive accumulation of fluid in the systemic vasculature.

The nurse is caring for a child after heart surgery. What intervention should the nurse implement immediately if evidence is found of cardiac tamponade? a. Increase analgesia b. Apply warming blankets c. Immediately report this to the physician d. Encourage the child to cough, turn, and breathe deeply

c. Immediately report this to the physician If evidence is noted of cardiac tamponade (blood or fluid in the pericardial space constricting the heart), the physician is notified immediately of this life-threatening complication. Increasing analgesia may be done before the physician drains the fluid, but the physician must be notified. Warming blankets are not indicated at this time. Encouraging the child to cough, turn, and breathe deeply should be deferred until after the evaluation by the physician.

Which statement correctly describes â-thalassemia major (Cooley's anemia)? a. All formed elements of the blood are depressed. b. Inadequate numbers of red blood cells are present. c. Increased incidence occurs in families of Mediterranean extraction. d. Increased incidence occurs in persons of West African descent.

c. Increased incidence occurs in families of Mediterranean extraction. Individuals who live near the Mediterranean Sea and their descendants have the highest incidence of thalassemia. An overproduction of red cells occurs. Although numerous, the red cells are relatively unstable. Sickle cell disease is common in blacks of West African descent.

Which statement most accurately describes the pathologic changes of sickle cell anemia? a. Sickle-shaped cells carry excess oxygen b. Sickle-shaped cells decrease blood viscosity c. Increased red blood cell destruction occurs d. Decreased red blood cell destruction occurs

c. Increased red blood cell destruction occurs The clinical features of sickle cell anemia are primarily the result of increased red blood cell destruction and obstruction caused by the sickle-shaped red blood cells. Sickled red cells have decreased oxygen-carrying capacity and transform into the sickle shape in conditions of low oxygen tension. When the sickle cells change shape, they increase the viscosity in the area where they are involved in the microcirculation.

Iron dextran is ordered for a young child with severe iron deficiency anemia. What nursing consideration should be considered? a. Administering with meals b. Administering between meals c. Injecting deeply into a large muscle d. Massaging injection site for 5 minutes after administration of drug

c. Injecting deeply into a large muscle Iron dextran is a parenteral form of iron. When administered intramuscularly, it must be injected into a large muscle using the Z-track method. Iron dextran is for intramuscular or intravenous administration; it is not taken orally. The site should not be massaged to prevent leakage, potential irritation, and staining of the skin. The administration has no relationship to food since it is not being given orally.

Which statement is most descriptive of a concussion? a. Petechial hemorrhages cause amnesia. b. Visible bruising and tearing of cerebral tissue occur. c. It is a transient, reversible neuronal dysfunction. d. A slight lesion develops remote from the site of trauma.

c. It is a transient, reversible neuronal dysfunction. A concussion is a transient, reversible neuronal dysfunction with instantaneous loss of awareness and responsiveness resulting from trauma to the head. Petechial hemorrhages along the superficial aspects of the brain along the point of impact are a type of contusion but are not necessarily associated with amnesia. A contusion is visible bruising and tearing of cerebral tissue. Contrecoup is a lesion that develops remote from the site of trauma as a result of an acceleration/deceleration injury.

It is appropriate, when caring for an unconscious child, to implement which intervention? a. Change the child's position infrequently to minimize the chance of increased intracranial pressure (ICP). b. Avoid using narcotics or sedatives to provide comfort and pain relief. c. Monitor fluid intake and output carefully to avoid fluid overload and cerebral edema. d. Give tepid sponge baths to reduce fever because antipyretics are contraindicated.

c. Monitor fluid intake and output carefully to avoid fluid overload and cerebral edema. Often comatose patients cannot cope with the quantity of fluids that they normally tolerate. Overhydration must be avoided to prevent fatal cerebral edema. The child's position should be changed frequently to avoid complications such as pneumonia and skin breakdown. Narcotics and sedatives should be used as necessary to reduce pain and discomfort, which can increase ICP. Antipyretics are the method of choice for fever reduction.

Which term is used to describe a child's level of consciousness when the child can be aroused with stimulation? a. Stupor b. Confusion c. Obtundation d. Disorientation

c. Obtundation Obtundation describes a level of consciousness in which the child can be aroused with stimulation. Stupor is a state in which the child remains in a deep sleep, responsive only to vigorous and repeated stimulation. Confusion is impaired decision making. Disorientation is confusion regarding time and place.

A nurse is teaching an adolescent about essential hypertension. The nurse knows that which of the following is correct? a. Primary hypertension should be treated with diuretics as soon as it is detected. b. Congenital heart defects are the most common cause of primary hypertension. c. Primary hypertension may be treated with weight reduction. d. Primary hypertension is not affected by exercise.

c. Primary hypertension may be treated with weight reduction Essential hypertension in children may be treated with weight reduction and exercise programs. If ineffective, pharmacologic intervention may be needed. Primary hypertension is considered an inherited disorder.

Which signs and symptoms are associated with Werdnig-Hoffmann disease? a. Spinal muscular atrophy b. Neural atrophy of muscles c. Progressive weakness and wasting of skeletal muscle d. Pseudohypertrophy of certain muscle groups

c. Progressive weakness and wasting of skeletal muscle Werdnig-Hoffmann disease (spinal muscular atrophy type 1) is the most common paralytic form of floppy infant syndrome (congenital hypotonia). It is characterized by progressive weakness and wasting of skeletal muscle caused by degeneration of anterior horn cells. Kugelberg-Welander syndrome is a juvenile spinal muscular atrophy with a later onset. Charcot-Marie-Tooth disease is a form of progressive neural atrophy of muscles supplied by the peroneal nerves. Progressive weakness of the distal muscles of the arms and feet is found. Duchenne's muscular dystrophy is characterized by muscles, especially in the calves, thighs, and upper arms that become enlarged from fatty infiltration and feel unusually firm or woody on palpation. The term pseudohypertrophy is derived from this muscular enlargement.

The nurse is planning activity for a 4-year-old child with anemia. Which activity should the nurse plan for this child? a. Game of "hide and seek" in the children's outdoor play area b. Participation in dance activities in the playroom c. Puppet play in the child's room d. A walk down to the hospital lobby

c. Puppet play in the child's room Because the basic pathologic process in anemia is a decrease in oxygen-carrying capacity, an important nursing responsibility is to assess the child's energy level and minimize excess demands. The child's level of tolerance for activities of daily living and play is assessed, and adjustments are made to allow as much self-care as possible without undue exertion. Puppet play in the child's room would not be overly tiring. Hide and seek, dancing, and walking to the lobby would not conserve the anemic child's energy.

Which action by the school nurse is important in the prevention of rheumatic fever? a. Encourage routine cholesterol screenings. b. Conduct routine blood pressure screenings. c. Refer children with sore throats for throat cultures. d. Recommend salicylates instead of acetaminophen for minor discomforts.

c. Refer children with sore throats for throat cultures. Nurses have a role in prevention—primarily in screening school-age children for sore throats caused by group A â-hemolytic streptococci. They can achieve this by actively participating in throat culture screening or by referring children with possible streptococcal sore throats for testing. Cholesterol and blood pressure screenings do not facilitate the recognition and treatment of group A â-hemolytic streptococci. Salicylates should be avoided routinely because of the risk of Reye's syndrome after viral illnesses.

When a 10 year old has been hit by a car while riding his bicycle in front of the school, the school nurse immediately assesses airway, breathing, and circulation. What should be the next nursing action? a. Place on side b. Take blood pressure c. Stabilize neck and spine d. Check scalp and back for bleeding

c. Stabilize neck and spine After determining that the child is breathing and has adequate circulation, the next action is to stabilize the neck and spine to prevent any additional trauma. The child's position should not be changed until the neck and spine are stabilized. Blood pressure is a later assessment. Less urgent, but an important assessment, is inspection of the scalp for bleeding.

As related to inherited disorders, which statement is descriptive of most cases of hemophilia? a. Autosomal dominant disorder causing deficiency in a factor involved in the blood-clotting reaction b. X-linked recessive inherited disorder causing deficiency of platelets and prolonged bleeding c. X-linked recessive inherited disorder in which a blood-clotting factor is deficient d. Y-linked recessive inherited disorder in which the red blood cells become moon shaped

c. X-linked recessive inherited disorder in which a blood-clotting factor is deficient The inheritance pattern in 80% of all of the cases of hemophilia is X-linked recessive. The two most common forms of the disorder are factor VIII deficiency (hemophilia A or classic hemophilia), and factor IX deficiency (hemophilia B or Christmas disease). The disorder involves coagulation factors, not platelets. The disorder does not involve red cells or the Y chromosome.

A young child's parents call the nurse after their child was bitten by a raccoon in the woods. The nurse's recommendation should be based on knowing that: a. the child should be hospitalized for close observation. b. no treatment is necessary if thorough wound cleaning is done. c. antirabies prophylaxis must be initiated. d. antirabies prophylaxis must be initiated if clinical manifestations appear.

c. antirabies prophylaxis must be initiated. Current therapy for a rabid animal bite consists of a thorough cleansing of the wound and passive immunization with human rabies immune globulin (HRIG) as soon as possible. Hospitalization is not necessary. The wound cleansing, passive immunization, and immune globulin administration can be done as an outpatient. The child needs to receive both HRIG and rabies vaccine.

Several blood tests are ordered for a preschool child with severe anemia. She is crying and upset because she remembers the venipuncture done at the clinic 2 days ago. The nurse should explain that: a. venipuncture discomfort is very brief. b. only one venipuncture will be needed. c. topical application of local anesthetic can eliminate venipuncture pain. d. most blood tests on children require only a finger puncture because a small amount of blood is needed.

c. topical application of local anesthetic can eliminate venipuncture pain. Preschool children are very concerned about both pain and the loss of blood. When preparing the child for venipuncture, a topical anesthetic will be used to eliminate any pain. This is a very traumatic experience for preschool children. They are concerned about their bodily integrity. A local anesthetic should be used, and a bandage should be applied to maintain bodily integrity. A promise that only one venipuncture will be needed should not be made in case multiple attempts are required. Both finger punctures and venipunctures are traumatic for children. Both require preparation.

The nurse is talking to a parent with a child who has a latex allergy. Which statement by the parent would indicate a correct understanding of the teaching? a. "My child will have an allergic reaction if he comes in contact with yeast products." b. "My child may have an upset stomach if he eats a food made with wheat or barley." c. "My child will probably develop an allergy to peanuts." d. "My child should not eat bananas or kiwis."

d. "My child should not eat bananas or kiwis." There are cross-reactions between latex allergies and a number of foods such as bananas, avocados, kiwi, and chestnuts. Although yeast products, wheat and barley, and peanuts are potential allergens, they are currently not known to cross-react with latex.

What is the most appropriate nursing response to the father of a newborn infant with myelomeningocele who asks about the cause of this condition? a. "One of the parents carries a defective gene that causes myelomeningocele." b. "A deficiency in folic acid in the father is the most likely cause." c. "Offspring of parents who have a spinal abnormality are at greater risk for myelomeningocele." d. "There may be a variety of different causes."

d. "There may be a variety of different causes." The etiology of most neural tube defects is likely multifactorial. There may be a genetic predisposition or a viral origin, and the disorder has been linked to maternal folic acid deficiency; however, the actual cause has not been determined. There is no evidence that children who have parents with spinal problems are at greater risk for neural tube defects.

The nurse is performing a Glasgow Coma Scale (GCS) on a school-age child with a head injury. The child opens eyes spontaneously, obeys commands, and is oriented to person, time, and place. Which is the score the nurse should record? a. 8 b. 11 c. 13 d. 15

d. 15 The GCS consists of a three-part assessment: eye opening, verbal response, and motor response. Numeric values of 1 through 5 are assigned to the levels of response in each category. The sum of these numeric values provides an objective measure of the patient's level of consciousness (LOC). A person with an unaltered LOC would score the highest, 15. The child who opens eyes spontaneously, obeys commands, and is oriented is scored at a 15.

The nurse is caring for an intubated infant with botulism. Which health care provider prescriptions should the nurse clarify with the health care provider before implementing? a. Administer 250 mg botulism immune globulin intravenously (BIG-IV) one time. b. Provide total parenteral nutrition (TPN) at 25 mL/hr intravenously. c. Titrate oxygen to keep pulse oximetry saturations greater than 92. d. Administer gentamicin sulfate 10 mg per intravenous piggyback every 12 hours.

d. Administer gentamicin sulfate 10 mg per intravenous piggyback every 12 hours. The nurse should clarify the administration of an aminoglycoside antibiotic. Antibiotic therapy is not part of the management of infant botulism because the botulinum toxin is an intracellular molecule, and antibiotics would not be effective; aminoglycosides in particular should not be administered because they may potentiate the blocking effects of the neurotoxin. Treatment consists of immediate administration of botulism immune globulin intravenously (BIG-IV) without delaying for laboratory diagnosis. Early administration of BIG-IV neutralizes the toxin and stops the progression of the disease. The human-derived botulism antitoxin (BIG-IV) has been evaluated and is now available nationwide for use only in infant botulism. Approximately 50% of affected infants require intubation and mechanical ventilation; therefore, respiratory support is crucial, as is nutritional support because these infants are unable to feed.

What type of shock is characterized by a hypersensitivity reaction causing massive vasodilation and capillary leaks, which may occur with drug or latex allergy? a. Neurogenic shock b. Cardiogenic shock c. Hypovolemic shock d. Anaphylactic shock

d. Anaphylactic shock Anaphylactic shock results from extreme allergy or hypersensitivity to a foreign substance. Neurogenic shock results from loss of neuronal control, such as the interruption of neuronal transmission that occurs from a spinal cord injury. Cardiogenic shock is decreased cardiac output. Hypovolemic shock is a reduction in the size of the vascular compartment, decreasing blood pressure, and low central venous pressure.

The nurse is caring for a school-age girl who has had a cardiac catheterization. The child tells the nurse that her bandage is "too wet." The nurse finds the bandage and bed soaked with blood. What is the most appropriate initial nursing action? a. Notify the physician. b. Apply a new bandage with more pressure. c. Place the child in the Trendelenburg position. d. Apply direct pressure above the catheterization site.

d. Apply direct pressure above the catheterization site. If bleeding occurs, direct continuous pressure is applied 2.5 cm (1 inch) above the percutaneous skin site to localize pressure over the vessel puncture. Notifying the physician and applying a new bandage with more pressure can be done after pressure is applied. The nurse can have someone else notify the physician while the pressure is being maintained. The Trendelenburg position would not be helpful; it would increase the drainage from the lower extremities.

What action may be beneficial in reducing the risk of Reye's syndrome? a. Immunization against the disease b. Medical attention for all head injuries c. Prompt treatment of bacterial meningitis d. Avoidance of aspirin and ibuprofen for children with varicella or those suspected of having influenza

d. Avoidance of aspirin and ibuprofen for children with varicella or those suspected of having influenza Although the etiology of Reye's syndrome is obscure, most cases follow a common viral illness, either varicella or influenza. A potential association exists between aspirin therapy and the development of Reye's syndrome; thus use of aspirin is avoided. No immunization currently exists for Reye's syndrome. Reye's syndrome is not correlated with head injuries or bacterial meningitis.

What is a common, serious complication of rheumatic fever? a. Seizures b. Cardiac arrhythmias c. Pulmonary hypertension d. Cardiac valve damage

d. Cardiac valve damage Cardiac valve damage is the most significant complication of rheumatic fever. Seizures, cardiac arrhythmias, and pulmonary hypertension are not common complications of rheumatic fever.

An 8-year-old girl is receiving a blood transfusion when the nurse notes that she has developed precordial pain, dyspnea, distended neck veins, slight cyanosis, and a dry cough. These manifestations are most suggestive of what associated adverse reaction? a. Air embolism b. Allergic reaction c. Hemolytic reaction d. Circulatory overload

d. Circulatory overload The signs of circulatory overload include distended neck veins, hypertension, crackles, dry cough, cyanosis, and precordial pain. Signs of air embolism are sudden difficulty breathing, sharp pain in the chest, and apprehension. Allergic reactions are manifested by urticaria, pruritus, flushing, asthmatic wheezing, and laryngeal edema. Hemolytic reactions are characterized by chills, shaking, fever, pain at infusion site, nausea, vomiting, tightness in chest, flank pain, red or black urine, and progressive signs of shock and renal failure.

What is the primary result of anemia? a. Increased blood viscosity. b. Depressed hematopoietic system. c. Presence of abnormal hemoglobin. d. Decreased oxygen-carrying capacity of blood.

d. Decreased oxygen-carrying capacity of blood. Anemia is a condition in which the number of red blood cells or hemoglobin concentration is reduced below the normal values for age. This results in a decreased oxygen-carrying capacity of blood. Increased blood viscosity is usually a function of too many cells or of dehydration, not of anemia. A depressed hematopoietic system or abnormal hemoglobin can contribute to anemia, but the definition depends on the deceased oxygen-carrying capacity of the blood.

What is an expected assessment finding in a child with coarctation of the aorta? a. Orthostatic hypotension b. Systolic hypertension in the lower extremities c. Blood pressure higher on the left side of the body d. Disparity in blood pressure between the upper and lower extremities

d. Disparity in blood pressure between the upper and lower extremities The classic finding in children with coarctation of the aorta is a disparity in pulses and blood pressures between the upper and lower extremities. Orthostatic hypotension is not present with coarctation of the aorta. Systolic hypertension may be detected in the upper extremities. The left arm may not accurately reflect systolic hypertension because the left subclavian artery can be involved in the coarctation.

When discussing hyperlipidemia with a group of adolescents, the nurse should explain that high levels of what substance are thought to protect against cardiovascular disease? a. Cholesterol b. Triglycerides c. Low-density lipoproteins (LDLs) d. High-density lipoproteins (HDLs)

d. High-density lipoproteins (HDLs) HDLs contain very low concentrations of triglycerides, relatively little cholesterol, and high levels of proteins. It is thought that HDLs protect against cardiovascular disease. Cholesterol, triglycerides, and LDLs do not protect against cardiovascular disease.

What name is given to identify an acquired hemorrhagic disorder that is characterized by excessive destruction of platelets? a. Aplastic anemia b. Thalassemia major c. Disseminated intravascular coagulation d. Immune thrombocytopenia

d. Immune thrombocytopenia Immune thrombocytopenia is an acquired hemorrhagic disorder characterized by an excessive destruction of platelets, discolorations caused by petechiae beneath the skin, and a normal bone marrow. Aplastic anemia refers to a bone marrow failure condition in which the formed elements of the blood are simultaneously depressed. Thalassemia major is a group of blood disorders characterized by deficiency in the production rate of specific hemoglobin globin chains. Disseminated intravascular coagulation is characterized by diffuse fibrin deposition in the microvasculature, consumption of coagulation factors, and endogenous generation of thrombin and plasma.

The parents of a child diagnosed with cerebral palsy ask the nurse if any drugs can decrease their child's spasticity. The nurse's response should be based on what knowledge? a. Anticonvulsant medications are sometimes useful for controlling spasticity. b. Medications that would be useful in reducing spasticity are too toxic for use with children. c. Many different medications can be highly effective in controlling spasticity. d. Implantation of a pump to deliver medication into the intrathecal space to decrease spasticity has recently become available.

d. Implantation of a pump to deliver medication into the intrathecal space to decrease spasticity has recently become available. Baclofen given intrathecally is best suited for children with severe spasticity that interferes with activities of daily living and ambulation. Anticonvulsant medications are used when seizures occur in children with cerebral palsy. The intrathecal route decreases the side effects of the drugs that reduce spasticity. Few medications are presently available for the control of spasticity.

What intervention should the nurse share with parents on how to prevent iron deficiency anemia in a healthy, term, breastfed infant? a. Iron (ferrous sulfate) drops after age 1 month b. Iron-fortified commercial formula can be used by ages 4 to 6 months c. Iron-fortified solid foods are introduced at 3 months d. Iron-fortified infant cereal can be introduced at approximately 6 months of age

d. Iron-fortified infant cereal can be introduced at approximately 6 months of age Breast milk supplies inadequate iron for growth and development after age 5 months. Supplementation is necessary at this time. Iron supplementation or the introduction of solid foods in a breastfed baby is not indicated. Introducing iron-fortified infant cereal at 2 months should be done only if the mother is choosing to discontinue breastfeeding.

Which postoperative intervention should be questioned for a child after a cardiac catheterization? a. Continue intravenous (IV) fluids until the infant is tolerating oral fluids. b. Check the dressing for bleeding. c. Assess peripheral circulation on the affected extremity. d. Keep the affected leg flexed and elevated.

d. Keep the affected leg flexed and elevated. The child should be positioned with the affected leg straight for 4 to 6 hours after the procedure. IV fluid administration continues until the child is taking and retaining adequate amounts of oral fluids. The insertion site dressing should be observed frequently for bleeding. The nurse should also look under the child to check for pooled blood. Peripheral perfusion is monitored after catheterization. Distal pulses should be palpable, although they may be weaker than in the contralateral extremity.

A child is unconscious after a motor vehicle accident. The watery discharge from the nose tests positive for glucose. The nurse should recognize that this suggests what complication? a. Diabetic coma b. Brainstem injury c. Upper respiratory tract infection d. Leaking of cerebrospinal fluid (CSF)

d. Leaking of cerebrospinal fluid (CSF) Watery discharge from the nose that is positive for glucose suggests leaking of CSF from a skull fracture and is not associated with diabetes or respiratory tract infection. The fluid is probably CSF from a skull fracture and does not signify whether the brainstem is involved.

The parents of a young child with congestive heart failure tell the nurse that they are "nervous" about giving digoxin. The nurse's response should be based on knowing what information? a. It is a safe, frequently used drug. b. It is difficult to either overmedicate or undermedicate with digoxin. c. Parents lack the expertise necessary to administer digoxin. d. Parents must learn specific, important guidelines for administration of digoxin.

d. Parents must learn specific, important guidelines for administration of digoxin. Digoxin has a narrow therapeutic range. The margin of safety between therapeutic, toxic, and lethal doses is very small. Specific guidelines are available for parents to learn how to administer the drug safely and monitor for side effects. Digoxin is a frequently used drug, but it has a narrow therapeutic range. Very small amounts of the liquid are given to infants, which makes it easy to overmedicate or undermedicate. Parents may lack the necessary expertise to administer the drug at first, but with discharge preparation they should be prepared to administer the drug safely.

The nurse is caring for a child with severe head trauma after a car accident. Which is an ominous sign that often precedes death? a. Papilledema b. Delirium c. Doll's head maneuver d. Periodic and irregular breathing

d. Periodic and irregular breathing Periodic or irregular breathing is an ominous sign of brainstem (especially medullary) dysfunction that often precedes complete apnea. Papilledema is edema and inflammation of the optic nerve. It is commonly a sign of increased intracranial pressure. Delirium is a state of mental confusion and excitement marked by disorientation to time and place. The doll's head maneuver is a test for brainstem or oculomotor nerve dysfunction.

Surgical closure of the ductus arteriosus would bring about what desired effect? a. Stop the loss of unoxygenated blood to the systemic circulation. b. Decrease the edema in legs and feet. c. Increase the oxygenation of blood. d. Prevent the return of oxygenated blood to the lungs.

d. Prevent the return of oxygenated blood to the lungs. The ductus arteriosus allows blood to flow from the higher-pressure aorta to the lower-pressure pulmonary artery, causing a right-to-left shunt. If this is surgically closed, no additional oxygenated blood (from the aorta) will return to the lungs through the pulmonary artery. The aorta carries oxygenated blood to the systemic circulation. Because of the higher pressure in the aorta, blood is shunted into the pulmonary artery and the pulmonary circulation. Edema in the legs and feet is usually a sign of heart failure. This repair would not directly affect the edema. Increasing the oxygenation of blood would not interfere with the return of oxygenated blood to the lungs.

Seventy-two hours after cardiac surgery, a young child has a temperature of 37.7° C (101° F). The nurse should initially implement which intervention? a. Keep the child warm with blankets b. Apply a hypothermia blanket c. Record the temperature on nurses' notes d. Report findings to physician

d. Report findings to physician In the first 24 to 48 hours after surgery, the body temperature may increase to 37.7° C (100° F) as part of the inflammatory response to tissue trauma. If the temperature is higher or an elevated temperature continues after this period, it is most likely a sign of an infection and immediate investigation is indicated. Blankets should be removed from the child to keep the temperature from increasing. A hypothermia blanket is not indicated for this level of temperature. The temperature should be recorded, but the physician must be notified for evaluation.

What is the most common causative agent of bacterial endocarditis? a. Staphylococcus albus b. Streptococcus hemolyticus c. Staphylococcus albicans d. Streptococcus viridans

d. Streptococcus viridans Staphylococcus viridans is the most common causative agent in bacterial (infective) endocarditis. Staphylococcus albus, Streptococcus hemolyticus, and Staphylococcus albicans are not common causative agents.

When caring for the child with Kawasaki disease, the nurse should understand that principle of care? a. The child's fever is usually responsive to antibiotics within 48 hours. b. The principal area of involvement is the joints. c. Aspirin is contraindicated. d. Therapeutic management includes administration of gamma globulin and aspirin.

d. Therapeutic management includes administration of gamma globulin and aspirin. High-dose intravenous gamma globulin and aspirin therapy are indicated to reduce the incidence of coronary artery abnormalities when given within the first 10 days of the illness. The fever of Kawasaki disease is unresponsive to antibiotics and antipyretics. Involvement of mucous membranes and conjunctiva, changes in the extremities, and cardiac involvement are seen.

The mother of a 1-month-old infant tells the nurse that she worries that her baby will get meningitis like her oldest son did when he was an infant. On what information should the nurse's response be based upon? a. Meningitis rarely occurs during infancy. b. Often a genetic predisposition to meningitis is found. c. Vaccination to prevent all types of meningitis is now available. d. Vaccination to prevent Haemophilus influenzae type B meningitis has decreased the frequency of this disease in children.

d. Vaccination to prevent Haemophilus influenzae type B meningitis has decreased H. influenzae type B meningitis has virtually been eradicated in areas of the world where the vaccine is administered routinely. Bacterial meningitis remains a serious illness in children. It is significant because of the residual damage caused by undiagnosed and untreated or inadequately treated cases. The leading causes of neonatal meningitis are the group B streptococci and Escherichia coli organisms. Meningitis is an extension of a variety of bacterial infections. No genetic predisposition exists. Vaccinations are not available for all of the potential causative organisms.

Which statement best describes a myelomeningocele? a. Fissure in the spinal column that leaves the meninges and the spinal cord exposed. b. Herniation of the brain and meninges through a defect in the skull. c. Hernial protrusion of a sac-like cyst of meninges with spinal fluid but no neural elements. d. Visible defect with an external sac-like protrusion containing meninges, spinal fluid, and nerves.

d. Visible defect with an external sac-like protrusion containing meninges, spinal fluid, and nerves. A myelomeningocele is a visible defect with an external sac-like protrusion, containing meninges, spinal fluid, and nerves. Rachischisis is a fissure in the spinal column that leaves the meninges and the spinal cord exposed. Encephalocele is a herniation of brain and meninges through a defect in the skull, producing a fluid-filled sac. Meningocele is a hernial protrusion of a sac-like cyst of meninges with spinal fluid, but no neural elements.

A pediatric client diagnosed with Turner syndrome tells the nurse, "I feel different from my peers." Which response by the nurse is the most appropriate? 1. "Tell me more about the feelings you are experiencing." 2. "These feelings are not unusual and should pass soon." 3. "You'll start to grow soon, so don't worry." 4. "You seem to be upset about your disease."

1. "Tell me more about the feelings you are experiencing." The lack of growth and sexual development associated with Turner syndrome presents problems with psychosocial development. Self-image, self-consciousness, and self-esteem are affected by the girl's perception of her body and how she differs from peers. The nurse should encourage more expression of the girl's feelings. Responding that the feelings will pass, that she'll start to grow, or that she is upset about the disease would not be therapeutic

The nurse has completed discharge teaching for the family of a child diagnosed with Legg-Calve-Perthes disease. Which statement by the family indicates the need for further education? 1. "We're glad this will only take about 6 weeks to correct." 2. "We understand swimming is a good sport for Legg-Calve-Perthes." 3. "We know to watch for areas on the skin the brace may rub." 4. "We understand that abduction of the affected leg is important."

1. "We're glad this will only take about 6 weeks to correct." The treatment generally takes approximately 2 years. Swimming is a good activity to increase mobility. A brace may be worn, so skin irritation should be monitored. The leg should be kept in the abducted position.

The nurse completes parent education related to treatment for a pediatric client with congenital clubfoot. Which statement by the parents indicates the need for further education? 1. "We're happy this is the only cast our baby will need." 2. "We'll watch for any swelling of the feet while the casts are on." 3. "We'll keep the casts dry." 4. "We're getting a special car seat to accommodate the casts."

1. "We're happy this is the only cast our baby will need." Serial casting is the treatment of choice for congenital clubfoot. The cast is changed every one to 2 weeks. Parents should be watching for swelling while the casts are on, keeping the casts dry, and using a car seat to accommodate the casts.

The nurse in the newborn nursery is performing the admission assessment on a neonate. Which assessment finding indicates the neonate may have congenital hip dysplasia? 1. Asymmetry of the gluteal and thigh fat folds 2. Trendelenburg sign 3. Telescoping of the affected limb 4. Lordosis

1. Asymmetry of the gluteal and thigh fat folds A sign of congenital hip dysplasia in the infant would be asymmetry of the gluteal and thigh fat folds. Trendelenburg sign and telescoping of the affected limb are signs that present in an older child with congenital hip dysplasia. Lordosis does not occur with hip dysplasia.

The pediatric nurse educator is conducting an in-service for novice nurses who will begin working on the pediatric oncology unit. The educator wants to include the common clinical manifestations of cancer. Which manifestation will the educator include in the presentation? Select all that apply. 1. Cachexia 2. Anemia 3. Gene abnormalities 4. Palpable mass 5. Chromosomal abnormalities

1. Cachexia 2. Anemia 4. Palpable mass Common clinical manifestations of childhood cancer include cachexia, anemia, and a palpable mass. Gene abnormalities and chromosomal abnormalities are common etiologies to childhood cancer, not clinical manifestations.

A child is diagnosed with a Wilms tumor. Which nursing action is most appropriate prior to surgery? 1. Careful bathing and handling 2. Monitoring of behavioral status 3. Maintenance of strict isolation 4. Administration of packed RBCs

1. Careful bathing and handling The tumor should never be palpated; careful bathing and handling are an important nursing consideration. Palpating the tumor can cause a piece of the tumor to dislodge. The child's behavior will not be affected with a Wilms tumor. The tumor does not cause excessive lowering of WBCs or RBCs, so strict isolation or administration of packed RBCs is not usually a nursing intervention.

A school-age client sustains a basilar skull fracture. Which symptom is a priority for this nurse to assess for when providing care to this client? 1. Cerebral spinal fluid leakage from the nose or ears 2. Headache 3. Transient confusion 4. Periorbital ecchymosis

1. Cerebral spinal fluid leakage from the nose or ears Cerebral spinal fluid leakage could be present from the nose or ears and, if it persists, may indicate that surgical repair will be needed. Headache, transient confusion, and periorbital ecchymosis are findings that commonly present with a basilar skull fracture but do not indicate that surgical repair will be needed.

The nurse is teaching a group of students about wound healing. Which items will the nurse include as occurring during the hemostasis and inflammation stage of wound healing? Select all that apply. 1. Clot formation to seal the wound 2. Production of collagen and granulation tissue 3. Scar formation and strengthening 4. Release of inflammatory mediators by platelets 5. Swelling as a result of increased capillary permeability

1. Clot formation to seal the wound 2. Production of collagen and granulation tissue 5. Swelling as a result of increased capillary permeability During the hemostasis and inflammation stage of wound healing, the nurse would state that clot formation occurs to seal the wound; platelets release inflammatory mediators; and increased capillary permeability results in swelling. Scar formation and strengthening occur during maturation. Collagen and granulation tissue are produced during tissue formation.

The nurse is caring for a pediatric client diagnosed with eczema. Which topical medication order does the nurse anticipate for this client? 1. Corticosteroid 2. Retinoid 3. Antifungal 4. Antibacterial

1. Corticosteroid Topical corticosteroid is used to reduce inflammation when the child has eczema. Topical retinoid is used for acne. Topical antifungal is used for dermatophytoses. Topical antibacterial would be used for problems such as burns

A seasoned nurse is precepting a novice nurse on a pediatric oncology unit. The seasoned nurse would like to review the ongoing physiologic and psychosocial care of the children who survive cancer. Which topics will the seasoned nurse include in the discussion with the novice nurse? Select all that apply. 1. Developing other cancers 2. Recommending regular office visits 3. Encouraging school-age clients to manage their own care 4. Needing weekly laboratory tests 5. Providing educational and psychosocial support

1. Developing other cancers 2. Recommending regular office visits 5. Providing educational and psychosocial support Appropriate topics include discussing the increased risk for these children to develop other cancers; recommending regular office visits for monitoring purposes; and providing educational and psychosocial support. It would be appropriate to encourage the adolescent and young adult clients to manage their own care, not a school-age child. While these clients need regular laboratory examinations, weekly laboratory tests are not appropriate.

A lumbar puncture is performed on an infant suspected of having meningitis. Which finding does the nurse expect in the cerebral spinal fluid if the infant has meningitis? 1. Elevated WBC count 2. Elevated RBC count 3. Normal glucose 4. Decreased WBC count

1. Elevated WBC count The lumbar puncture is done to obtain cerebral spinal fluid (CSF). Elevated WBC count is seen with bacterial meningitis. The RBC count is not elevated, and the glucose is decreased in meningitis.

A child sustains a traumatic brain injury and is monitored in the pediatric intensive-care unit (PICU). The nurse is using the Glasgow Coma Scale to assess the child. Which items will the nurse assess when using this tool? Select all that apply. 1. Eye opening 2. Verbal response 3. Motor response 4. Head circumference 5. Pulse oximetry

1. Eye opening 2. Verbal response 3. Motor response The Glasgow Coma Scale for infants and children scores parameters related to eye opening, verbal response, and motor response. The maximum score is 15, indicating the highest level of neurological functioning. Head circumference and pulse oximetry are not included on the scale.

The parents of a child with Duchenne muscular dystrophy are in the clinic after diagnosis and ask the nurse if the family should have genetic testing completed. Who should the nurse suggest to have genetic testing? Select all that apply. 1. Female cousins 2. Aunts 3. Sisters 4. Brothers 5. Uncles and male cousins

1. Female cousins 2. Aunts 3. Sisters This is an X-linked disorder so all females in the family should be tested.

A child diagnosed with cancer is prescribed chemotherapy. The latest lab value indicates the WBC count is very low. Which medication order does the nurse anticipate? 1. Filgrastim (Neupogen) 2. Ondansetron (Zofran) 3. Oprelvekin (Neumega) 4. Epoetin alfa (human recombinant erythropoietin)

1. Filgrastim (Neupogen) Filgrastim (Neupogen) increases production of neutrophils by the bone marrow. Ondansetron (Zofran) is an antiemetic, oprelvekin (Neumega) increases platelets, and epoetin alfa (human recombinant erythropoietin) stimulates RBC production.

The nurse is providing education to a group of student nurses regarding disorders of the endocrine system that can cause short stature. Which disorders will the nurse include in the educational session? Select all that apply. 1. Hypothyroidism 2. Turner syndrome 3. Type 1 diabetes mellitus 4. Diabetes insipidus 5. Cushing syndrome

1. Hypothyroidism 2. Turner syndrome 5. Cushing syndrome There are many disorders of the endocrine system that can cause short stature including hypothyroidism, Turner syndrome, and Cushing syndrome. Type 1 diabetes mellitus and diabetes insipidus are not endocrine disorders that cause short stature.

The nurse is planning care for a school-age child diagnosed with bacterial meningitis. Which intervention is most appropriate? 1. Keeping environmental stimuli at a minimum 2. Avoiding giving pain medications that could dull sensorium 3. Measuring head circumference to assess developing complications 4. Having the child move the head from side to side at least every two hours

1. Keeping environmental stimuli at a minimum A quiet environment should be maintained because noise can disturb a child with meningitis. Pain medications are appropriate to give and should be used when needed. Measuring head circumference would only be appropriate for a child less than 2 years. Excessive head movement should be avoided because it can increase irritation of the meninges.

Which teaching tips should be included when instructing parents on hydrocortisone administration? Select all that apply. 1. Maintain prescribed administration times. 2. Never discontinue medication abruptly. 3. Injections might be necessary when unable to take by mouth. 4. Lower doses are needed during illness. 5. Keep an emergency kit with the child at all times.

1. Maintain prescribed administration times. 2. Never discontinue medication abruptly. 3. Injections might be necessary when unable to take by mouth. 5. Keep an emergency kit with the child at all times. Maintaining prescribed administration times is important, as they follow the normal body release of cortisol. Abruptly discontinuing a steroid is not recommended. Giving injections when unable to take by mouth and during emergencies is important to maintain cortisol levels. Higher, not lower, doses are needed during illness.

The pediatric nurse is providing care to a school-age child receiving chemotherapy to treat cancer. Which interventions are appropriate to include in the plan of care in order to monitor for oncologic emergencies? Select all that apply. 1. Monitor complete blood count (CBC). 2. Document intake and output. 3. Observe for behavioral changes. 4. Refer for psychosocial support. 5. Implement neutropenic precautions.

1. Monitor complete blood count (CBC). 2. Document intake and output. 3. Observe for behavioral changes. Oncologic emergencies can be organized into three groups: metabolic, hematologic, and those involving space-occupying lesions. Appropriate interventions for the nurse to include in the plan of care to monitor for these emergencies include monitoring the CBC to prevent sepsis and hemorrhage; monitoring intake and output by encouraging hydration to prevent hypercalcemia and observing for signs of water intoxication; and observing for behavioral changes as space-occupying lesions may cause seizures or increased intracranial pressure. Referring for psychosocial support and implementing neutropenia precautions may be appropriate, but these interventions do not address oncologic emergencies.

A child diagnosed with a mild traumatic brain injury is being sedated with a mild sedative so that pain and anxiety are minimized. Which nursing interventions are appropriate for this child? Select all that apply. 1. Place a continuous-pulse oximetry monitor on the child. 2. Place the child in a room near the nurse's station. 3. Allow for several visitors to remain at the child's bedside. 4. Use soft restraints if the child becomes confused. 5. Use sedation around the clock to decrease agitation.

1. Place a continuous-pulse oximetry monitor on the child. 2. Place the child in a room near the nurse's station. When a child is sedated, respiratory status should be monitored with a pulse-oximetry machine. The child should be close to the nurse's station so that frequent monitoring can be done. Several visitors at the bedside would increase the child's anxiety. Soft restraints may increase agitation. Sedation around the clock is not recommended due to the need to evaluate the neurologic system.

A child is diagnosed with rhabdomyosarcoma. Which nursing intervention is most appropriate for this child? 1. Position the child with the head elevated 2. Monitor for hematuria 3. Demonstrate the use of a conformer 4. Administer oxygen

1. Position the child with the head elevated The most common area of the body affected by rhabdomyosarcoma is the bladder. The nursing intervention that is most appropriate is to monitor the child's urine for hematuria. Positioning the child with the head elevated and administering oxygen is appropriate for a child diagnosed with lymphoma. Demonstrating the use of a conformer is appropriate for a child diagnosed with retinoblastoma.

The nurse is planning care for pediatric clients who have diagnoses that impact the endocrine system. Which changes occurring during the school-age and adolescence have a direct impact on the endocrine system? Select all that apply. 1. Puberty 2. Adrenarche 3. Menarche 4. Sexual exploration 5. Risk-taking behavior

1. Puberty 2. Adrenarche 3. Menarche Puberty, adrenarche, and menarche are all changes that occur during the school age and adolescence that have a direct impact on the endocrine system. Sexual exploration and risk-taking behaviors do not have a direct impact on the endocrine system.

A child is diagnosed with thrombocytopenia secondary to chemotherapy treatments. Which action by the nurse is the most appropriate? 1. Refrain from administering any intramuscular injections (IM). 2. Perform oral hygiene. 3. Monitor intake and output. 4. Use palpation as a component of assessment.

1. Refrain from administering any intramuscular injections (IM). When the child is thrombocytopenic (decreased platelets) from chemotherapy, the nurse should not administer IM injections because of the risk of bleeding. Oral hygiene care should be done with a soft toothbrush and intake and output monitored for any abnormalities. Gentle palpation should still be included in physical assessments.

A nurse is planning care for a pediatric client diagnosed with adrenal insufficiency (Addison disease). Which nursing diagnosis is the priority for this client? 1. Risk for Deficient Fluid Volume 2. Risk for Injury Secondary to Hypertension 3. Acute Pain 4. Imbalanced Nutrition: More than Body Requirements

1. Risk for Deficient Fluid Volume Adrenal insufficiency can cause fluid deficit. The goal of care is to maintain fluid and electrolyte balance while normal levels of corticosteroids and mineral corticoids are established. Therefore, Acute Pain and Imbalanced Nutrition: More than Body Requirements are not priority nursing diagnoses. A symptom of adrenal insufficiency is hypotension, not hypertension.

An adolescent client must wear a brace for the correction of scoliosis. Which nursing diagnosis is most appropriate for this client? 1. Risk for Impaired Skin Integrity 2. Risk for Altered Growth and Development 3. Risk for Impaired Mobility 4. Risk for Impaired Gas Exchange

1. Risk for Impaired Skin Integrity The skin should be monitored for breakdown in any area the brace may rub. The other diagnoses would not be a priority and should be corrected by the wearing of the brace.

The nurse educator is describing the pediatric differences associated with the anatomy and physiology of the neurologic system to a group of nursing students. Which statements made by the class indicate appropriate understanding of this topic after the teaching session? Select all that apply. 1. The bones of the cranium are connected by connective tissue to allow for brain growth. 2. The spine of infants is excessively mobile due to immature neck muscles and incompletely developed vertebral bodies. 3. Maturation of the nerves continues until age 10. 4. Myelination is complete at birth, 5. Myelination proceeds in a cephalocaudal direction.

1. The bones of the cranium are connected by connective tissue to allow for brain growth. 2. The spine of infants is excessively mobile due to immature neck muscles and incompletely developed vertebral bodies. 5. Myelination proceeds in a cephalocaudal direction. There are several pediatric differences associated with the anatomy and physiology of the neurological system and include: the bones of the cranium are connected by connective tissue to allow for brain growth; the spine of infants is excessively mobile due to immature neck muscles and incompletely developed vertebral bodies; and myelination proceeds in a cephalocaudal direction. Maturation of the nerves continues until the age of 4, not 10. Myelination is incomplete at birth.

The nurse is providing care to a pediatric client who is diagnosed with psoriasis. Which clinical manifestations does the nurse anticipate upon assessment of this client? Select all that apply. 1. Thick, silvery, scaly erythematous plaque 2. Pruritus 3. Dry skin, likely to crack and fissure 4. Fragile skin and blisters 5. Irregular border surrounded by normal skin

1. Thick, silvery, scaly erythematous plaque 2. Pruritus 5. Irregular border surrounded by normal skin Clinical manifestations that support the diagnosis of psoriasis include thick, silvery, scaly erythematous plaque; pruritis; and irregular border surrounded by normal skin. Dry skin that is likely to crack and fissure is a clinical manifestation of atopic dermatitis. Fragile skin and blisters are clinical manifestations of epidermolysis bullosa.

The child is admitted to the hospital after being diagnosed with retinoblastoma. Which assessment finding does the nurse anticipate for this child? 1. A red reflex 2. Yellow sclera 3. A white pupil 4. Blue-tinged sclera

3. A white pupil The first sign of retinoblastoma is a white pupil. The red reflex is absent. Yellow sclera is a sign of jaundice, not retinoblastoma. Blue-tinged sclera is a sign of osteogenesis imperfecta, not retinoblastoma.

A school health nurse is screening school-age students for scoliosis. Which assessment findings indicate the need for further evaluation for scoliosis? Select all that apply. 1. Uneven shoulders and hips 2. A one-sided rib hump 3. Prominent scapula 4. Lordosis 5. Pain

1. Uneven shoulders and hips 2. A one-sided rib hump 3. Prominent scapula The classic signs of scoliosis include uneven shoulders and hips, a one-sided rib hump, and prominent scapula. Lordosis and pain are not present with scoliosis.

The nurse is teaching a group of adolescents about care for acne vulgaris. Which interventions will the nurse include in the teaching session? Select all that apply. 1. Wash skin with mild soap and water twice a day. 2. Use astringents and vigorous scrubbing. 3. Avoid picking or squeezing the lesions. 4. Apply tretinoin (Retin-A) liberally. 5. Avoid sun exposure if on tetracycline.

1. Wash skin with mild soap and water twice a day. 3. Avoid picking or squeezing the lesions. 5. Avoid sun exposure if on tetracycline. The adolescent should be taught to wash skin with mild soap and water twice a day, to avoid picking or squeezing acne lesions, and to avoid sun exposure if on tetracycline. Using astringents and scrubbing vigorously can exacerbate acne. Tretinoin (Retin-A) should be applied sparingly (pea-size doses).

A child diagnosed with a Wilms tumor is prescribed chemotherapy. Which laboratory test will the nurse monitor prior to administering the chemotherapy to determine the child's infection-fighting capability? 1. Hemoglobin 2. RBC count 3. Absolute neutrophil count (ANC) 4. Platelets

3. Absolute neutrophil count (ANC) The absolute neutrophil count uses both the segmented (mature) and bands (immature) neutrophils as a measure of the body's infection-fighting capability. RBC count, hemoglobin, and platelets cannot determine infection-fighting capabilities.

A school-age client experiences a near-drowning episode and is admitted to the pediatric intensive-care unit (PICU). The parents express guilt over the near drowning of their child. Which response by the nurse is most appropriate? 1. "You will need to watch the child more closely." 2. "Tell me more about your feelings related to the accident." 3. "The child will be fine, so don't worry." 4. "Why did you let the child almost drown?"

2. "Tell me more about your feelings related to the accident." In near-drowning cases, the nurse should be nonjudgmental and provide a forum for parents to express guilt. Telling the parents to watch the child more closely or asking them why they let the child almost drown is judgmental. Saying the child will be fine may not be true. The nurse should reassure the parents that the child is receiving all possible medical treatment.

The nurse is teaching the parent of a type 1 diabetic preschool-age client about management of the disease. Which teaching point is appropriate for the nurse to include in this session? 1. Allowing the client to administer all the insulin injections 2. Allowing the client to choose which finger to stick for glucose testing 3. Allowing the client to draw up the insulin dose 4. Allowing the client to test blood glucose

2. Allowing the client to choose which finger to stick for glucose testing The preschool-age client's need for autonomy and control can be met by allowing the client to pick which finger to stick for glucose testing. Administering the insulin, drawing up the dose, and testing blood glucose should not be done by the client until he or she is middle-school age or older.

The nurse is caring for the newborn with bilateral clubfoot. What nursing diagnoses would the nurse address? Select all that apply. 1. Activity intolerance 2. Impaired physical mobility 3. Risk for impaired skin integrity 4. Ineffective breathing pattern 5. Impaired parenting

2. Impaired physical mobility 3. Risk for impaired skin integrity 5. Impaired parenting Nursing diagnoses that may apply to the newborn with bilateral clubfoot are impaired physical mobility, risk for impaired skin integrity, impaired parenting, and ineffective health maintenance.

A preschool-age child is brought to the clinic by the mother, who says the child has been lethargic and anorexic lately and complains of bone pain. On exam, the nurse notes petechiae, joint pain, and an enlarged liver. Which diagnosis does the nurse anticipate for this child? 1. Hodgkin disease 2. Leukemia 3. Rhabdomyosarcoma 4. Ewing sarcoma

2. Leukemia Hodgkin disease, rhabdomyosarcoma, and Ewing sarcoma are all childhood cancers, but they do not have the clinical manifestations listed. Leukemia is one of the most common childhood cancers, and has those clinical symptoms.Leukemia

A pediatric client is admitted to the hospital unconscious. The client has a history of type 1 diabetes, and according to the client's mother, has been to two birthday parties in the last few days and has resisted taking the prescribed insulin. At school the client had two more pieces of birthday cake and some ice cream at a class birthday party. What is the likely reason for this client's unconscious state? 1. Metabolic alkalosis 2. Metabolic ketoacidosis 3. Insulin shock 4. Insulin reaction

2. Metabolic ketoacidosis Metabolic acidosis or ketoacidosis could have occurred because of the excessive intake of sugar with no additional insulin. The body burns fat and protein stores for energy when no insulin is available to metabolize glucose. Altered consciousness occurs as symptoms progress. Metabolic alkalosis, insulin shock, or insulin reaction would not be happening in this case.

A nurse is conducting a daily weight on a pediatric client diagnosed with diabetes insipidus and notes the child has lost 2 pounds in 24 hours. Which action by the nurse is the most appropriate? 1. Continue to monitor the child. 2. Notify the healthcare provider regarding the weight loss. 3. Chart the weight and report the loss to the next shift. 4. Do nothing more than chart the weight, as this would be a normal finding.

2. Notify the healthcare provider regarding the weight loss. With diabetes insipidus, the child may have severe fluid-volume deficit. A weight loss of 2 pounds indicates a loss of 1 liter of fluid, so the healthcare provider should be notified and fluids replaced either orally or intravenously. This is a significant loss in a 24-hour period, so continuing to monitor, charting the weight and reporting to the next shift, and doing nothing would prolong treatment.

The nurse is administering a dose of rapid-acting insulin at 0800 to an insulin-dependent pediatric client. Based on when the insulin peaks, when will the client be at greatest risk for a hypoglycemic episode? 1. At about noon 2. Between bedtime and breakfast the next morning 3. Between lunch and dinner 4. Around 0930

4. Around 0930 Rapid-acting insulin peaks 30-90 minutes after administration. An injection given at 0800 would peak around 0930.

A nurse is caring for a toddler client who is diagnosed with scabies and prescribed a 5 percent permethrin lotion. How will the nurse apply this lotion when administering it to the toddler? 1. To the scalp only 2. Over the entire body from the chin down, as well as on the scalp and forehead 3. Only on the areas with evidence of scabies activity 4. Only on the hands

2. Over the entire body from the chin down, as well as on the scalp and forehead Treatment of scabies involves application of a scabicide, such as 5 percent permethrin lotion, over the entire body from the chin down. The scabicide is also applied to the scalp and forehead of younger children, avoiding the rest of the face.

A nurse is assessing a child after an open reduction of a fractured femur. Which assessment findings would indicate that the child is experiencing compartment syndrome? Select all that apply. 1. Pink, warm extremity 2. Pain not relieved by pain medication 3. Dorsalis pedis pulse present 4. Prolonged capillary-refill time with paresthesia 5. Skin appears tense.

2. Pain not relieved by pain medication 4. Prolonged capillary-refill time with paresthesia 5. Skin appears tense. The major serious complication post-fracture reduction is compartment syndrome. A prolonged capillary-refill time with loss of paresthesia, pain not relieved by medication, and skin that appears tense are signs of compartment syndrome. Pink, warm extremity; pain relieved by medication; and a present dorsalis pedis pulse would all be normal findings post-fracture reduction.

A child is diagnosed with epilepsy and is prescribed daily phenytoin (Dilantin). Which topic is most appropriate for the nurse to include in the discharge teaching? 1. Increasing fluid intake 2. Performing good dental hygiene 3. Decreasing intake of vitamin D 4. Taking the medication with milk

2. Performing good dental hygiene Because phenytoin (Dilantin) can cause gingival hyperplasia, good dental hygiene should be encouraged. Fluid intake does not affect the drug's effectiveness, an adequate intake of vitamin D should be encouraged, and phenytoin (Dilantin) should not be taken with dairy products.

A school-age child is being assessed for syndrome of inappropriate antidiuretic hormone (SIADH). The nurse should watch the child for which symptoms? Select all that apply. 1. Polyphagia 2. Retention of fluid 3. Hypernatremia 4. Hyponatremia 5. Hyperglycemia

2. Retention of fluid 3. Hypernatremia ADH helps the body retain fluid. Serum osmolality is increased (greater than 300 mOsm/kg) and urine osmolality is decreased (less than 300 mOsm/kg). Urine specific gravity is decreased (less than 1.005) and serum sodium is elevated.

Which action by the parents demonstrates an understanding of the nurse's teaching with regard to prevention of iron-deficient anemia? 1. Feeding their infant with a formula that is not iron fortified 2. Starting iron-fortified infant cereal at 4 to 6 months of age 3. Introducing cow's milk at 6 months of age 4. Limiting vitamin C consumption after 1 year of age

2. Starting iron-fortified infant cereal at 4 to 6 months of age Starting iron-fortified infant cereal at 4 to 6 months of age is recommended for prevention of iron deficiency in children. Infants who are not breast-fed should get iron-fortified formula. Cow's milk should not be introduced until 12 months of age. Vitamin C should be started at 6 to 9 months of age and continued, because foods rich in vitamin C improve iron absorption.

The nurse is caring for a pediatric client diagnosed with syndrome of inappropriate antidiuretic hormone (SIADH) disorder. Which interventions should the nurse implement for this child? Select all that apply. 1. Encouragement of fluids 2. Strict intake and output 3. Administration of ordered diuretics 4. Specific gravity of urine 5. Weight only on admission but not daily

2. Strict intake and output 3. Administration of ordered diuretics 4. Specific gravity of urine SIADH results from an excessive amount of serum antidiuretic hormone, causing water intoxication and hyponatremia. Intake and output should be monitored strictly. Diuretics such as furosemide (Lasix) are administered to eliminate excess body fluid, and urine specific gravity is monitored. Fluids are restricted to prevent further hemodilution. Daily weights should be obtained to monitor fluid balance.

A preschool child is seen in the clinic, and the nurse anticipates a diagnosis of leukemia. Which reaction does the nurse anticipate this child will exhibit upon diagnosis? 1. Acceptance, especially if able to discuss the disease with children their own age 2. Thoughts that they caused their illness and are being punished 3. Understanding of what cancer is and how it is treated 4. Unawareness of the illness and its severity

2. Thoughts that they caused their illness and are being punished Preschool-age children may think they caused their illness. Adolescents find contact with others who have gone through their experience helpful. School-age children can understand a diagnosis of cancer. Infants and toddlers are unaware of the severity of the disease.

An adolescent is receiving methotrexate chemotherapy after undergoing limb-salvage surgery for osteogenic sarcoma. Which statement by the adolescent indicates understanding of the purpose of leucovorin therapy after the methotrexate? 1. "I'm glad I only need one dose of the leucovorin." 2. "I don't have any pain so I won't need to take the leucovorin this time." 3. "I know I will be taking the leucovorin every 6 hours for about the next 3 days." 4. "I don't have any nausea so I won't need the leucovorin."

3. "I know I will be taking the leucovorin every 6 hours for about the next 3 days." Leucovorin (citrovorum factor) is a form of folic acid that helps to protect normal cells from the destructive action of methotrexate. It is started within 24 hours of methotrexate administration and is given along with hydration therapy. Usual administration is every 6 hours for 72 hours or until serum methotrexate is at the desired level.

A pediatric client is seen in the clinic with a possible diagnosis of type 2 diabetes. The mother asks what the healthcare provider uses to make the diagnosis. The nurse explains that type 2 diabetes is suspected if the child has obesity, acanthosis nigricans, and two non-fasting blood-glucose levels above which level? 1. 120 2. 80 3. 200 4. 50

3. 200 Blood-glucose levels at or above 200 mg/dL without fasting is diagnostic of type 2 diabetes.

An infant has a severe case of oral thrush (Candida albicans). Which nursing diagnosis is the priority for this infant? 1. Activity Intolerance Related to Oral Thrush 2. Ineffective Airway Clearance Related to Mucus 3. Ineffective Infant Feeding Pattern Related to Discomfort 4. Ineffective Breathing Pattern Related to Oral Thrush

3. Ineffective Infant Feeding Pattern Related to Discomfort An infant with oral thrush may refuse to nurse or feed because of discomfort and pain. Prompt treatment is necessary so the infant can resume a normal feeding pattern. Activity intolerance, ineffective airway clearance, and ineffective breathing patterns are not usual associated problems.

The nurse is providing teaching to a community group regarding preventative strategies to reduce the risk of burn injury. Which topics will the nurse include in the teaching session? Select all that apply. 1. Avoid contact with unknown animals and wild animals. 2. Layer children's clothing for warmth. 3. Keep infants and toddlers off the lap when drinking hot beverages or eating soup. 4. Lower the temperature settings for hot water heaters. 5. Wear light-colored clothes and avoid eating sweetened foods and beverages when outside.

3. Keep infants and toddlers off the lap when drinking hot beverages or eating soup. 4. Lower the temperature settings for hot water heaters. In order to decrease the risk of burn injury, the nurse would tell the group to keep infants and toddlers off the lap while drinking hot beverages or eating soup and to lower the temperature settings for the hot water heaters. Avoiding contact with unknown animals and wild animals along with wearing light-colored clothes and avoiding eating sweetened foods and beverages when outside are strategies to prevent bites and stings. Layering children's clothing for warmth is a strategy to prevent hypothermia.

Which nursing intervention is most appropriate when caring for an infant with a myelomeningocele in the preoperative stage? 1. Placing infant supine to decrease pressure on the sac 2. Appling a heat lamp to facilitate drying and toughening of the sac 3. Measuring head circumference every shift to identify developing hydrocephalus 4. Appling a diaper to prevent contamination of the sac

3. Measuring head circumference every shift to identify developing hydrocephalus The infant should be monitored for developing hydrocephalus, so the head circumference should be monitored daily. The infant will be placed prone, not supine, and the defect will be protected from trauma or infection. Therefore, applying heat and a diaper around the defect would not be recommended. A sterile saline dressing may be used to cover the sac to maintain integrity.

The nurse is planning care for a 3-month-old infant diagnosed with eczema. Which should be the focus of the nurse's care for this infant? 1. Maintaining adequate nutrition 2. Keeping the baby content 3. Preventing infection of lesions 4. Applying antibiotics to lesions

3. Preventing infection of lesions Nursing care should focus on preventing infection of lesions. Due to impaired skin-barrier function and cutaneous immunity, an infant with eczema is at greater risk for the development of skin infections by organisms. Maintaining adequate nutrition and keeping the infant content are not as high a priority. Antibiotics are not routinely applied to the lesions.

A child with myelomeningocele, corrected at birth, is now 5 years old. Which is the priority nursing diagnosis for a child with corrected spina bifida at this age? 1. Risk for Altered Nutrition 2. Risk for Impaired Tissue Perfusion-Cranial 3. Risk for Altered Urinary Elimination 4. Risk for Altered Comfort

3. Risk for Altered Urinary Elimination A child with spina bifida will continue to have a risk for altered urinary elimination because the bowel and bladder sphincter controls are affected. Urinary retention is a problem, so bladder interventions are initiated early to prevent kidney damage. Risk for Altered Nutrition, Impaired Tissue Perfusion, and Altered Comfort are not problems once surgery has been performed to close the defect.

An adolescent client diagnosed with Graves' disease is admitted to the hospital. Which clinical manifestations would the nurse expect on assessment? 1. Weight gain, hirsutism, and muscle weakness 2. Dehydration, metabolic acidosis, and hypertension 3. Tachycardia, fatigue, and heat intolerance 4. Hyperglycemia, ketonuria, and glucosuria

3. Tachycardia, fatigue, and heat intolerance Graves' disease occurs when thyroid hormone levels are increased, resulting in excessive levels of circulating thyroid hormones. Clinical manifestations include tachycardia, fatigue, and heat intolerance. Weight gain, hirsutism, and muscle weakness are signs of Cushing syndrome. Dehydration, metabolic acidosis, and hypertension are signs of congenital adrenal hyperplasia. Hyperglycemia, ketonuria, and glucosuria are signs of diabetes.

A pediatric client is diagnosed with type 1 diabetes. The nurse teaches the client the difference between insulin shock and diabetic hyperglycemia. The nurse evaluates that the client understands the teaching when the client states which characteristics of diabetic hyperglycemia? 1. Tremors and lethargy 2. Hunger and hypertension 3. Thirst and flushed skin 4. Shakiness and pallor

3. Thirst and flushed skin Thirst and flushed skin are characteristic of diabetic hyperglycemia. Tremors, lethargy, hunger, shakiness, and pallor are characteristic of hypoglycemia. Hypertension is not a sign associated with hyperglycemia or hypoglycemia.

A school-age client is admitted to the hospital with osteomyelitis. Which statement regarding the treatment of osteomyelitis is most appropriate for the nurse to share with the parents? 1. "Cultures should be done immediately after the first dose of antibiotic infuses." 2. "Antibiotics are ineffective against this virus." 3. "Methicillin is the antibiotic of choice." 4. "Antibiotic therapy should continue for 3 to 6 weeks."

4. "Antibiotic therapy should continue for 3 to 6 weeks." Medical management of osteomyelitis begins with intravenous administration of a broad-spectrum antibiotic. Antibiotic therapy should continue for 3 to 6 weeks. Cultures are always done before an antibiotic is started. Methicillin is not the drug of choice.

A nurse is conducting a postoperative assessment on an infant who has just had a ventriculoperitoneal shunt placed for hydrocephalus. Which assessment finding would indicate a malfunction in the shunt? 1. Incisional pain 2. Movement of all extremities 3. Negative Brudzinski sign 4. Bulging fontanel

4. Bulging fontanel A bulging fontanel would be an abnormal finding and could indicate that the shunt is malfunctioning. Incisional pain, movement of all extremities, and negative Brudzinski sign are all normal findings after a ventriculoperitoneal shunt has been placed.

The nurse is evaluating the activity tolerance of a 9-month-old with iron deficiency anemia. Which finding indicates that the infant is not tolerating activity? 1. Heart rate of 138 2. Increased alertness 3. Respiratory rate less than 40 with activity 4. Muscle weakness

4. Muscle weakness Iron deficiency anemia can result in less oxygen reaching the cells and tissues, causing activity intolerance. An indication that a 9-month-old child is not tolerating activity and that iron deficiency anemia is worsening would be the presence of muscle weakness during activity. A heart rate of 138, increased alertness, and a respiratory rate of less than 40 with activity are all signs that iron deficiency anemia is resolving and activity tolerance is improving.

A 2-month-old client has a candidal diaper rash. Which medication does the nurse anticipate will be prescribed for this client? 1. Bacitracin ointment 2. Hydrocortisone ointment 3. Desitin 4. Nystatin given topically and orally

4. Nystatin given topically and orally Diaper candidiasis is treated with an antifungal cream (Nystatin). An oral antifungal agent may be given to clear the candidiasis from the intestines. Bacitracin is for an infection caused by staphylococcus. Mild diaper rash is treated with a barrier such as Desitin. Moderate diaper rash is treated with hydrocortisone ointment.

A nurse is caring for a child who is diagnosed with cerebral palsy. Which goal of therapy is most appropriate for the nurse to include in the plan of care? 1. Reversing the degenerative processes that have occurred 2. Curing the underlying defect causing the disorder 3. Preventing the spread to individuals in close contact with the child 4. Promoting optimum development

4. Promoting optimum development Recognition of the disorder is important so that optimal development can be maintained. Cerebral palsy cannot be reversed or cured. It is not caused by a contagious process, so there is no risk of spread.

The nurse is planning care for a pediatric client diagnosed with adrenal hyperplasia. Which nursing diagnosis is most appropriate for this client? 1. Impaired Social Interaction Related to Unnatural Facial Features 2. Nutrition: Less than Body Requirements due to Nausea and Vomiting 3. Depression Related to Inability to Take in Oral Fluids 4. Risk for Deficient Fluid Volume Related to Failure of Regulatory Mechanisms

4. Risk for Deficient Fluid Volume Related to Failure of Regulatory Mechanisms Adrenal hyperplasia alters the regulatory mechanisms, creating a fluid volume deficit. There is no major nutritional deficit, social interaction, or depression related directly to the diagnosis of adrenal hyperplasia.

A school-age client is transported to the emergency department by ambulance from the scene of a car accident. The client is alert and oriented × 3; pulse, respirations, and blood pressure are stable; and the neck and back are immobilized on a backboard. The nurse sees no obvious bleeding. The client states, "I can't feel or move my legs." Which injury does the nurse suspect? 1. Traumatic brain injury 2. Ruptured spleen 3. Traumatic shock 4. Spinal cord injury

4. Spinal cord injury Spinal cord injury results in paralysis and anesthesia of the affected body parts below the level of the lesion. Altered levels of consciousness may indicate traumatic brain injury. The child may have a ruptured spleen, but it is not evident from the data given. Traumatic shock results in initially increasing then decreasing pulse and respirations, and falling blood pressure.

The school nurse is conducting pediculosis capitis (head lice) checks. Which findings would indicate a "positive" head check? 1. White, flaky particles throughout the entire scalp region 2. Maculopapular lesions behind the ears 3. Lesions in the scalp that extend to the hairline or neck 4. White sacs attached to the hair shafts in the occipital area

4. White sacs attached to the hair shafts in the occipital area Evidence of pediculosis capitis includes white sacs (nits) that are attached to the hair shafts, frequently in the occiput area. Lesions may be present from itching, but the positive sign is evidence of nits. Lice and nits must be distinguished from dandruff, which appears as white, flaky particles.


Ensembles d'études connexes

Ch. 14: Everyday Theology (THEO 104 LUO)

View Set

Chapter 3 Accounting cycle end of period

View Set

Unit 4 - Matter & Energy in Ecosystems

View Set

11: Stream Processes, Landscapes, Mass Wastage, and Flood Hazards

View Set